Re: [obm-l] off topic - calculo numerico

2013-11-04 Por tôpico Johann Dirichlet

MatLab, C++, Fortran, Octave... É só achar uma biblioteca do gênero por aí!

Em 04-11-2013 20:16, regis barros escreveu:

Olá Herman
Aqui na Unicamp usa dependendo do professor C++ ou Matlab; Na UFSCar, 
em física, em fortran.


Regis



Em Segunda-feira, 4 de Novembro de 2013 17:26, Hermann 
ilhadepaqu...@bol.com.br escreveu:

OFF-TOPIC
Por favor, alguém sabe qual o programa que usam para estudar, 
atualmente, o cálculo numérico?

abraços
Herman

--
Esta mensagem foi verificada pelo sistema de antivírus e
acredita-se estar livre de perigo.



--
Esta mensagem foi verificada pelo sistema de antivírus e
acredita-se estar livre de perigo. 



--
Esta mensagem foi verificada pelo sistema de antivírus e
acredita-se estar livre de perigo.



Re: [obm-l] Quadrados em PA

2013-11-01 Por tôpico Johann Dirichlet

Em 18-10-2013 09:19, marcone augusto araújo borges escreveu:

Determine três números inteiros distintos cujos quadrados estão em PA
[Upload Photo to Facebook]
[Google+]
[Twitt]
[Send by Gmail]
[Upload Video to Facebook]
[Google+]
[Twitt]
[Send by Gmail]


Infintas, infinitas soluções.


x^2+y^2=2z^2 é equivalente a 'quadrados em PA'.

Como x e y têm a mesma paridade, podemos escrever x=a+b, y=a-b.

(a+b)^2+(a-b)^2=2(a^2+b^2)
Assim, a^2+b^2=z^2

E as boas e velhas trincas pitagóricas voltam com tudo!


--
Esta mensagem foi verificada pelo sistema de antivírus e
acredita-se estar livre de perigo. 



--
Esta mensagem foi verificada pelo sistema de antivírus e
acredita-se estar livre de perigo.



Re: [obm-l] Álgebra

2013-10-05 Por tôpico Johann Dirichlet

Em 22-09-2013 21:31, marcone augusto araújo borges escreveu:

Sejam x,y inteiros positivos tais que 3x^2 + x = 4y^2 + y.Mostre que
[Upload Photo to Facebook]
[Google+]
[Twitt]
[Send by Gmail]
[Upload Video to Facebook]
[Google+]
[Twitt]
[Send by Gmail]
x - y é um quadrado perfeito.
Estou tentando.Uma ajuda?



Que tal substituir x=d+y e provar que d é quadrado perfeito? 
Provavelmente você precisará resolver uma equação de Pell, mas é bem 
mais provável que um teste simples de divisibilidade seja mais rápido...







--
Esta mensagem foi verificada pelo sistema de antivírus e
acredita-se estar livre de perigo. 



--
Esta mensagem foi verificada pelo sistema de antivírus e
acredita-se estar livre de perigo.



Re: [obm-l] Teoria dos números

2013-08-31 Por tôpico Johann Dirichlet

Em 28-08-2013 21:45, marcone augusto araújo borges escreveu:

Eu já postei a questão aqui,mas infelizmente não obtive resposta.
Sei que vão aparecendo outras questões interessantes e por isso peço 
licença para reapresentá-la

Determine todos os inteiros positivos x,y tais que 7^x - 3^y = 4

Claro que x = 1 e y = 1 satisfaz(desconfio que seja a unica solução)
Eu só consegui concluir que x e y são ímpares,analisando módulo 4.
Desde já agradeço.
[Upload Photo to Facebook]
[Google+]
[Twitt]
[Send by Gmail]
[Upload Video to Facebook]
[Google+]
[Twitt]
[Send by Gmail]



Módulo 7, temos o quê?

-3^y = 4-7

3^y = 3
3^(y-1) = 1

Temos 3^3=27=-1, logo 6|(y-1).

Supondo y1, vamos tentar um módulo 9?

7^x=4
(-2)^x=4

Vou confiar em tu - x é ímpar:

-2^x=4
-2^(x-2)=1
2^(x-2)=-1=9-1=8=2^3
2^(x-5)=1

Temos
2^1=2
2^2=4
2^3=8
2^4=7
2^5=5
2^6=1

Logo 6|x+1


Que coisa! Ainda falta...






--
Esta mensagem foi verificada pelo sistema de antivírus e
acredita-se estar livre de perigo. 



--
Esta mensagem foi verificada pelo sistema de antivírus e
acredita-se estar livre de perigo.



[obm-l] Re: [obm-l] Re: [obm-l] RE: [obm-l] Re: [obm-l] Teoria dos números

2013-08-31 Por tôpico Johann Dirichlet

Em 30-08-2013 10:29, Ralph Teixeira escreveu:


Ah, droga, bobeei. Nao ajudou tanto quanto eu achava... :-( :-(



Mas o caminho deve ser este. Que tal o famigerado módulo 49? Afinal esse 
monte de primos incita raízes primitivas...


On Aug 29, 2013 12:23 PM, marcone augusto araújo borges 
marconeborge...@hotmail.com mailto:marconeborge...@hotmail.com wrote:


7^x modulo 9 dá 1,7 e 4 e 3^y dá quase sempre 0
O que interessa  para 7^x modulo 9 é 4,o que ocorre apenas quando
x é da forma 3.k + 2
Como x tambem é ímpar,só pode ser da forma 6.n + 5,mas...


Date: Thu, 29 Aug 2013 09:21:24 -0300
Subject: [obm-l] Re: [obm-l] Teoria dos números
From: ralp...@gmail.com mailto:ralp...@gmail.com
To: obm-l@mat.puc-rio.br mailto:obm-l@mat.puc-rio.br

Tente agora modulo 9.

On Aug 28, 2013 9:50 PM, marcone augusto araújo borges
marconeborge...@hotmail.com mailto:marconeborge...@hotmail.com
wrote:

Eu já postei a questão aqui,mas infelizmente não obtive resposta.
Sei que vão aparecendo outras questões interessantes e por
isso peço licença para reapresentá-la
Determine todos os inteiros positivos x,y tais que 7^x - 3^y = 4

Claro que x = 1 e y = 1 satisfaz(desconfio que seja a unica
solução)
Eu só consegui concluir que x e y são ímpares,analisando módulo 4.
Desde já agradeço.
[Upload Photo to Facebook]
[Google+]
[Twitt]
[Send by Gmail]
[Upload Video to Facebook]
[Google+]
[Twitt]
[Send by Gmail]


-- 
Esta mensagem foi verificada pelo sistema de antivírus e

acredita-se estar livre de perigo.


-- 
Esta mensagem foi verificada pelo sistema de antivírus e

acredita-se estar livre de perigo.

-- 
Esta mensagem foi verificada pelo sistema de antivírus e

acredita-se estar livre de perigo.


--
Esta mensagem foi verificada pelo sistema de antivírus e
acredita-se estar livre de perigo. 



--
Esta mensagem foi verificada pelo sistema de antivírus e
acredita-se estar livre de perigo.



Re: [obm-l] Contagem(?)

2013-08-31 Por tôpico Johann Dirichlet

Em 30-08-2013 21:58, marcone augusto araújo borges escreveu:

De quantas maneiras podemos escrever 2010 como soma de dois
[Upload Photo to Facebook]
[Google+]
[Twitt]
[Send by Gmail]
[Upload Video to Facebook]
[Google+]
[Twitt]
[Send by Gmail]
inteiros positivos primos entre si?

Eu tirei todas as possíveis parcelas que fossem um múltiplo dos 
divisores de 2010,mas achei 528 e o gabarito dá 264.





Se x+y=2010 e d|x e d|y, então d|2010. Basta então contar todos os 
números primos com 2010 e menores que 2010 - e dividir o total por 2.


Isto é a infame função phi de Euler!



--
Esta mensagem foi verificada pelo sistema de antivírus e
acredita-se estar livre de perigo. 



--
Esta mensagem foi verificada pelo sistema de antivírus e
acredita-se estar livre de perigo.



Re: [obm-l] Re: [obm-l] Problema de lógica

2013-08-18 Por tôpico Johann Dirichlet

Em 18-08-2013 08:58, Artur Costa Steiner escreveu:


Eu apontaria para uma carta qualquer e perguntaria:

Se eu lhe perguntasse se esta carta é um ás, você diria que sim?



O problema é se esta carta for um ás. Aí você não tem como saber qual é 
a outra carta - pode ser valete ou ás.
Realmente, perguntar sobre outra carta é a estratégia. Eu apontaria a 
primeira e perguntaria sobre a carta da outra ponta.



Artur

Em 17/08/2013 21:30, Mauricio de Araujo 
mauricio.de.ara...@gmail.com mailto:mauricio.de.ara...@gmail.com 
escreveu:


Eu disponho de três cartas de baralho, dois ases e um valete, e as
disponho sobre uma mesa com as faces voltadas para baixo, uma ao
lado da outra. Antes de virar as faces, eu anotei a posição de
cada uma das cartas, de maneira que eu sei onde os ases e o valete
estão.
Tua missão é identificar um dos ases me fazendo apenas uma
pergunta que admita apenas SIM-NÃO como resposta. Para isso, você
deverá apontar para uma das cartas. Se a carta para a qual você
apontar for um ás, a resposta à tua pergunta será sincera (eu não
mentirei); se a carta for um valete, a resposta poderá ser sim ou
não, aleatoriamente.

-- 
Abraços


oɾnɐɹɐ ǝp oıɔıɹnɐɯ
/momentos excepcionais pedem ações excepcionais./
/Os cemitérios estão cheios de pessoas insubstituíveis em seus
ofícios./

-- 
Esta mensagem foi verificada pelo sistema de antivírus e
acredita-se estar livre de perigo. 



--
Esta mensagem foi verificada pelo sistema de antiv�rus e
acredita-se estar livre de perigo. 



--
Esta mensagem foi verificada pelo sistema de antiv�rus e
acredita-se estar livre de perigo.



Re: [obm-l] Problema de lógica

2013-08-17 Por tôpico Johann Dirichlet

Em 17-08-2013 21:24, Mauricio de Araujo escreveu:
Eu disponho de três cartas de baralho, dois ases e um valete, e as 
disponho sobre uma mesa com as faces voltadas para baixo, uma ao lado 
da outra. Antes de virar as faces, eu anotei a posição de cada uma das 
cartas, de maneira que eu sei onde os ases e o valete estão.
Tua missão é identificar um dos ases me fazendo apenas uma pergunta 
que admita apenas SIM-NÃO como resposta. Para isso, você deverá 
apontar para uma das cartas. Se a carta para a qual você apontar for 
um ás, a resposta à tua pergunta será sincera (eu não mentirei); se a 
carta for um valete, a resposta poderá ser sim ou não, aleatoriamente.




O apontar para a carta é independente da pergunta, certo? No seguinte 
sentido: eu aponto para a carta do meio, mas pergunto se a primeira 
carta é um valete. Pode ser?



--
Abraços

oɾnɐɹɐ ǝp oıɔıɹnɐɯ
/momentos excepcionais pedem ações excepcionais./
/Os cemitérios estão cheios de pessoas insubstituíveis em seus ofícios./

--
Esta mensagem foi verificada pelo sistema de antiv�rus e
acredita-se estar livre de perigo. 



--
Esta mensagem foi verificada pelo sistema de antiv�rus e
acredita-se estar livre de perigo.



Re: [obm-l] Re: [obm-l] Problema de lógica

2013-08-17 Por tôpico Johann Dirichlet

Em 17-08-2013 21:47, Mauricio de Araujo escreveu:
Sim, a pergunta não precisa ter relação com a carta para a qual você 
apontou... esta apenas vai orientar se a resposta vai ser sincera ou 
aleatória...




Então a resposta vai ter que, pelo menos, forçar o cabra a falar a 
verdade a qualquer custo. Eu me lembro de um problema parecido:


Mulheres de óculos escuros respondiam perguntas. Se os olhos fossem 
azuis, elas falavam a verdade. Se fossem castanhos, mentira. Como saber 
a cor dos olhos de cada uma?


A pergunta para fazer a elas era algo como Se eu te perguntasse qual a 
cor dos olhos de cada uma de vós, qual seria a sua resposta?.






2013/8/17 Johann Dirichlet peterdirich...@gmail.com 
mailto:peterdirich...@gmail.com


Em 17-08-2013 21:24, Mauricio de Araujo escreveu:

Eu disponho de três cartas de baralho, dois ases e um valete, e
as disponho sobre uma mesa com as faces voltadas para baixo, uma
ao lado da outra. Antes de virar as faces, eu anotei a posição de
cada uma das cartas, de maneira que eu sei onde os ases e o
valete estão.
Tua missão é identificar um dos ases me fazendo apenas uma
pergunta que admita apenas SIM-NÃO como resposta. Para isso, você
deverá apontar para uma das cartas. Se a carta para a qual você
apontar for um ás, a resposta à tua pergunta será sincera (eu não
mentirei); se a carta for um valete, a resposta poderá ser sim ou
não, aleatoriamente.



O apontar para a carta é independente da pergunta, certo? No
seguinte sentido: eu aponto para a carta do meio, mas pergunto se
a primeira carta é um valete. Pode ser?

-- 
Abraços


oɾnɐɹɐ ǝp oıɔıɹnɐɯ
/momentos excepcionais pedem ações excepcionais./
/Os cemitérios estão cheios de pessoas insubstituíveis em seus
ofícios./

-- 
Esta mensagem foi verificada pelo sistema de antiv�rus e
acredita-se estar livre de perigo. 



-- 
Esta mensagem foi verificada pelo sistema de antivírus e

acredita-se estar livre de perigo.




--
Abraços

oɾnɐɹɐ ǝp oıɔıɹnɐɯ
/momentos excepcionais pedem ações excepcionais./
/Os cemitérios estão cheios de pessoas insubstituíveis em seus ofícios./

--
Esta mensagem foi verificada pelo sistema de antiv�rus e
acredita-se estar livre de perigo. 



--
Esta mensagem foi verificada pelo sistema de antiv�rus e
acredita-se estar livre de perigo.



[obm-l] Re: [obm-l] RE: [obm-l] Números inteiros

2011-09-23 Por tôpico Johann Dirichlet
Talvez a pergunta dele tenha sido
Determine o numero de soloçoes de 1/x + 1/y = 1/1998 com x e y
inteiros positivos.

E é fácil:

(x+y)*1998 = xy
1998x-xy+1998y=0
x(1998-y)+1998y-1998^2=-1998^2
x(1998-y)+1998(y-1998)=-1998^2
(1998-y)(x-1998)=-1998^2
(1998-y)(1998-x)=1998^2


Em 22/09/11, João Maldonadojoao_maldona...@hotmail.com escreveu:



 1) É impossível que  1/x +  1/y seja maior que 2 né?
 2)   4m²   +m(4n  -49) + 4n²  - 49n = 0
 delta  = 2401 + 392 n - 48 n   ²
 delta=0,  -4=n=12Testando  achamos( 6,10)(10,6)
 []'s
 João

 From: marconeborge...@hotmail.com
 To: obm-l@mat.puc-rio.br
 Subject: [obm-l] Números inteiros
 Date: Thu, 22 Sep 2011 21:23:47 +








 1) Determine o numero de soloçoes de 1/x + 1/y = 1998 com x e y inteiros
 positivos.



 2) Se m e n sao naturais tais que (m + n)/(m^2 + mn + n^2) = 4/49,determinar
 m + n



 Agradeço a quem puder ajudar.



 Abraço,



 Marcone.
   


-- 
/**/
神が祝福

Torres

=
Instru��es para entrar na lista, sair da lista e usar a lista em
http://www.mat.puc-rio.br/~obmlistas/obm-l.html
=


Re: [obm-l] Problema das Quatro Cores (Teoria dos Grafos)

2011-09-17 Por tôpico Johann Dirichlet
Existe uma demonstração fácil de que 5 cores bastam para pintar um grafo planar.
Acho que este é seu problema: tentar provar por absurdo algo que se
provaria diretamente.
Certamente, se você usa 4 cores para piontar, alguém que tem um
estoque de 5,6,7,2002 cores também consegue.

Mas o salto lógico é este:
Para que seja preciso 5 cores para pintar o grafo, eu teria que ter 5
nós ligados entre si.

Se isto não for corretamente demonstrado, adeus demonstração!



Em 17/09/11, Hugo Fernando Marques Fernandeshfernande...@gmail.com escreveu:
 Olá, Lista.

 Seguinte, estava lendo sobre o problema das quatro cores, que segundo
 entendi é um teorema da teoria dos grafos que afirma que se pode colorir
 qualquer grafo planar com quatro cores de modo que nós adjacentes (ou seja,
 que possuam aresta ligando-os) não sejam pintados da mesma cor.

 Consta que tal fato permaneceu por séculos sem demonstração, e a que existe
 hoje depende de recursos computacionais para ser completada, o que levanta
 dúvidas sobre a mesma.

 Minha pergunta então, é a seguinte:

 Para que seja preciso 5 cores para pintar o grafo, eu teria que ter 5 nós
 ligados entre si, isto é, eu teria que ter um sub-grafo do meu grafo inicial
 que fosse um grafo completo de 5 nós (K5). Ora, sabemos (é fácil demonstrar,
 já vi em vários livros a demonstração) que K5 não tem realizações
 planares... logo, o teorema segue.

 Sei que isso está errado (afinal de contas, se não estivesse, alguém teria
 visto de cara e o problema não teria ficado séculos em aberto...) mas não
 consigo ver onde está o erro desse raciocínio. Alguém pode me ajudar?

 Obrigado.



-- 
/**/
神が祝福

Torres

=
Instru��es para entrar na lista, sair da lista e usar a lista em
http://www.mat.puc-rio.br/~obmlistas/obm-l.html
=


[obm-l] Re: [obm-l] (e + pi) e (e.pi) são irracionais?

2011-09-13 Por tôpico Johann Dirichlet
Como diria Capitão Nascimento,  'Nunca saberão!'Mas é fácil provar que pelo 
menos um deles é irracional.
2011/9/13, Paulo  Argolo pauloarg...@bol.com.br: Caros Colegas,| Sabemos 
que os famosos números e e pi são irracionais. A soma (e + pi) e o 
produto (e.pi) são também irracionais? Abraços do Paulo! 
= 
Instruções para entrar na lista, sair da lista e usar a lista em 
http://www.mat.puc-rio.br/~obmlistas/obm-l.html 
=

-- /**/神が祝福
Torres
=
Instru��es para entrar na lista, sair da lista e usar a lista em
http://www.mat.puc-rio.br/~obmlistas/obm-l.html
=


[obm-l] Re: [obm-l] RE: [obm-l] Solução da série \sum_{i=1}^n 2^{n-i}i^2

2011-09-13 Por tôpico Johann Dirichlet
Na Eureka! 21, creio eu, tem o artigo Integrais Discretas. Ele ensina
a fazer este somatório de uma maneira bem automatizada.

Em 13/09/11, Luís Lopesqed_te...@hotmail.com escreveu:

 Sauda,c~oes, oi André,

 Comece calculando a série \sum_{i=1}^n i x^i.

 E depois \sum_{i=1}^n i^2 x^i.

 Para os detalhes, ver os exercícios 47 e 54 do Manual
 de Seq. e Series Vol I em http://www.escolademestres.com

 Abs,
 Luís



 From: andrerc...@gmail.com
 Date: Mon, 12 Sep 2011 20:06:35 -0300
 Subject: [obm-l] Solução da série \sum_{i=1}^n 2^{n-i}i^2
 To: obm-l@mat.puc-rio.br

 Pessoal,Alguém pode me dar alguma dica como se consegue obter a fórmula
 fecha da série \sum_{i=1}^n 2^{n-i}i^2O Wolfram Alpha indica que a fórmula
 fechada da mesma é 3 * 2^{n+1} - n^2 - 4n - 6:



 http://www.wolframalpha.com/input/?i=%5Csum_{i%3D1}^n+2^{n-i}i^2
 Porém, até então não obtive êxito.



 Abraço,__
 André




   


-- 
/**/
神が祝福

Torres

=
Instru��es para entrar na lista, sair da lista e usar a lista em
http://www.mat.puc-rio.br/~obmlistas/obm-l.html
=


[obm-l] Re: [obm-l] Teoria do Caos - Análise combinatória

2011-09-12 Por tôpico Johann Dirichlet
Procure no Google por permutaçAo caótica ou desarranjo.

Em 12/09/11, João Maldonadojoao_maldona...@hotmail.com escreveu:

 Olá, pra todo mundo
 Hoje meu professor me passou um problema sobre teoria do caos como desafio,
 a pergunta era
 Cinco livros caem de uma pratileira, quantas possibilidades existem de todos
 os cinco livros serem repostos,  um do lado do outro, de modo  que nenhum
 deles ocupe a mesma posição  de quando estavam na pratileira.
 Eu fiz  por recursão.
 Para  1 livro, P = 0Para dois livros ab, temos p = ba = 1Para 3 livros, abc,
 temos bac e bca = 2Para 4 livros,  temos 9...
 Para n livros temos:TOTAL  -  1 no mesmo lugar - 2 no mesmo lugar -...- n no
 mesmo lugar,
 f(n)  - n! -  C(n, 1)f(n-1)  - C(n, 2)f(n-2)...-C(n, n)f(0),  tal que f(0) =
 1
 Para 5 livros:  5! -  5.9 - 10.2 - 10.1 - 0 - 1 = 44
 No final meu professor me disse que havia uma fórmula  direta para f(n),
 mas eu não consegui acharComo acho essa fórmula?

 []'sJoão  


-- 
/**/
神が祝福

Torres

=
Instru��es para entrar na lista, sair da lista e usar a lista em
http://www.mat.puc-rio.br/~obmlistas/obm-l.html
=


[obm-l] Re: [obm-l] Questão de geometria plana!! ajuda em nova solução

2011-09-10 Por tôpico Johann Dirichlet
Ué, deslizar o triângulo pra baixo já é sintético. E é a única que eu
imagino agora.

Em 10/09/11, 
douglas.olive...@grupoolimpo.com.brdouglas.olive...@grupoolimpo.com.br
escreveu:


 Olá boa tarde, estou com uma questão de geometria plana, que diz
 assim: Em um triângulo equilátero, um ponto P interno dista de sues
 vértices 5 , 7, e 8 de sues vértices, achar o lado.

 gostaria de uma
 ajudinha, para elaborar uma nova solução, pois conheco a do oswaldo
 dolce, que transporta um triângulo( esta solucao encontra-se no gabarito
 do livro de matematica elementar numero 9), tambem conheco uma fazendo
 duas leis dos cossenos, e tambem elaborei uma em geometria analitica
 fazendo distancia de ponto a ponto , gostaria de uma ajuda para elaborar
 outra mas totalmente voltada para geometria plana, dede ja agradeco.



-- 
/**/
神が祝福

Torres

=
Instru��es para entrar na lista, sair da lista e usar a lista em
http://www.mat.puc-rio.br/~obmlistas/obm-l.html
=


Re: [obm-l] Divisor central de n

2011-09-08 Por tôpico Johann Dirichlet
Mole!Já foi resolvido por mim aqui, mas a ideia é que os produtos dosextremos 
do conjunto dos divisores são iguais a n
Em 08/09/11, Paulo  Argolopauloarg...@bol.com.br escreveu: Caros Colegas, 
Sabe-se que o número natural n1 tem uma quantidade ímpar de divisores e d 
ocupa a posição central, quando eles estão dispostos em ordem crescente. 
Mostrar que n é o quadrado de d. Grato, Paulo 
= 
Instru�ões para entrar na lista, sair da lista e usar a lista em 
http://www.mat.puc-rio.br/~obmlistas/obm-l.html 
=

-- /**/神が祝福
Torres
=
Instru��es para entrar na lista, sair da lista e usar a lista em
http://www.mat.puc-rio.br/~obmlistas/obm-l.html
=


[obm-l] Re: [obm-l] Sem soluções racionais

2011-09-07 Por tôpico Johann Dirichlet
Coloca cada cara na forma x =x/A. teremos:

x^2+y^2+z^2=7A^2, com todas as variáveis inteiras positivas.
Tentando um módulo 8, acho que sai...

Em 06/09/11, Vitor Alvesvitor__r...@hotmail.com escreveu:

 Não estou conseguinodo resolver o seguinte problema: Prove que não existem
 racionais x,y e z tais que x^{2} + y^{2} + z^{2}=7. Qualquer sugestão será
 bem vinda,abraços.
   


-- 
/**/
神が祝福

Torres

=
Instru��es para entrar na lista, sair da lista e usar a lista em
http://www.mat.puc-rio.br/~obmlistas/obm-l.html
=


[obm-l] Re: [obm-l] Re: [obm-l] Re: [obm-l] RE: Mas alguém deu resultado negativo no quinto problema da OBM hoje?

2011-09-03 Por tôpico Johann Dirichlet
Nem. É só fuçar nos sites de matemática mais obscuros da Internet -
OBM, mathlinks...

Em 03/09/11, Bruno França dos Reisbfr...@gmail.com escreveu:
 Nossa, essa é uma regra um tanto quanto difícil de se aplicar, não?

 --
 Bruno FRANÇA DOS REIS

 msn: brunoreis...@hotmail.com
 skype: brunoreis666
 tel: +55 11 9961-7732

 http://brunoreis.com
 http://brunoreis.com/tech (en)
 http://brunoreis.com/blog (pt)

 GPG Key: http://brunoreis.com/bruno-public.key

 e^(pi*i)+1=0


 2011/9/3 Carlos Yuzo Shine cysh...@yahoo.com

 Caros, até terça, quando sai o gabarito, não é permitido discutir a prova
 pela Internet. Isso está escrito na ficha de inscrição da prova.

 []'s
 Shine


 
 From: João Maldonado joao_maldona...@hotmail.com
 To: obm-l@mat.puc-rio.br
 Sent: Saturday, September 3, 2011 1:48 PM
 Subject: [obm-l] RE: Mas alguém deu resultado  negativo no quinto problema
 da OBM hoje?






 Esquece, já descobri no que errei.
 O problema mais difícil  da prova e eu justo vou errar na coisa mais tosca

 []'s
 João




 
 From: joao_maldona...@hotmail.com
 To: obm-l@mat.puc-rio.br
 Subject: Mas alguém deu resultado  negativo no quinto problema da OBM
 hoje?
 Date: Sat, 3 Sep 2011 13:40:06 -0300


  Mais alguém achou um resultado negativo no 5 problema de hoje ou foi só
  eu??

 []'s
 João

 =
 Instruções para entrar na lista, sair da lista e usar a lista em
 http://www.mat.puc-rio.br/~obmlistas/obm-l.html
 =




-- 
/**/
神が祝福

Torres

=
Instru��es para entrar na lista, sair da lista e usar a lista em
http://www.mat.puc-rio.br/~obmlistas/obm-l.html
=


Re: [obm-l] Desafio limite.

2011-09-02 Por tôpico Johann Dirichlet
limite de x^x, x tende a 0+

lim log x^x=lim (x*log x)
lim log (x*log x) = lim log x + lim log log x

lim log x x tende a 0

O que eu fiz ajuda?

Em 29/08/11, Felippe Coulbert Balbifelippeba...@hotmail.com escreveu:

 Que legal... não sabia que já tinha uma definição de algo assim... Mas
 enfim... eu escreve errado é 1 se n é par e 0 se n é impar.

 Date: Mon, 29 Aug 2011 20:50:12 -0300
 Subject: Re: [obm-l] Desafio limite.
 From: wgapetre...@gmail.com
 To: obm-l@mat.puc-rio.br

 http://en.wikipedia.org/wiki/Knuth's_up-arrow_notation

 2011/8/29 Felippe Coulbert Balbi felippeba...@hotmail.com






 Meu amigo Lucas Colucci e eu resolvemos esse problema que surgiu de uma aula
 de calculo.
 Espero que gostem bastante dele.
 Definição: Dado um x pertencendo ao conjunto dos numeros reais e um número
 n pertencendo ao conjunto dos numeros naturais.
 definimos: x|||n= e^(ln(x).x|||n-1)

 definimos: x|||0= 1 (ao invés de x|||n, meu amigo Lucas sugeriu x flecha pra
 cima n, mas enfim, não faz muita diferença)

 Por exemplo:

 x|||3= x^(x^x)
 x|||5= x^(x^(x^(x^x)))

 Prove que


 Lim x|||n =
 x-0+
 =

 1, se n é impar
 0 se n é par

 Grato.Coulbert
   

   


-- 
/**/
神が祝福

Torres

=
Instru��es para entrar na lista, sair da lista e usar a lista em
http://www.mat.puc-rio.br/~obmlistas/obm-l.html
=


Re: [obm-l] Logica

2011-08-18 Por tôpico Johann Dirichlet
O mais matemático que eu conseguiria pensar é lógica booleana de
circuitos. Mas é lógica do mesmo jeito: analisar as premissas e obter
uma conclusão.

-- 
/**/
神が祝福

Torres

=
Instru��es para entrar na lista, sair da lista e usar a lista em
http://www.mat.puc-rio.br/~obmlistas/obm-l.html
=


Re: [obm-l] Logica

2011-08-18 Por tôpico Johann Dirichlet
Não sei se é isso, mas ele prefere algo mais sistemático, menos ad
hoc. Mas isto é meio difícil - creio que impossível: parece um
problema P-NP.

Em 18/08/11, Bernardo Freitas Paulo da Costabernardo...@gmail.com escreveu:
 Eu acho que se você fizer uma matriz do que pode ser, e for marcando
 0 / 1, sai. (Por exemplo, se na linha Erica e na coluna Inteligente
 tem um 0, isso quer dizer que Érica não é inteligente). Esse seu
 problema é razoavelmente fácil porque já tem uma afirmação sobre um
 dos 0/1 : Joyce. O resto sai daí direto. Talvez essa afirmação seja
 redundante... (eu não usei a penúltima, e nesse tipo de problema,
 haver uma informação redundante é quase sempre permutável).

 Mas não sei se isso você não chamaria de lógica. Eu sim... mesmo que
 a tabelinha faça parecer mais do que não é. Enfim, você tem um
 problema de lógica, porque você quer usar outra coisa? Ou fazendo uma
 pergunta mais simples: se fosse um problema de geometria, porque você
 acha que uma solução por geometria não é bom?

 Abraços,
 --
 Bernardo Freitas Paulo da Costa


 2011/8/18 Marcus Aurelio Gonçalves Rodrigues marcusaureli...@globo.com:
 Galera existe alguma forma matemática  sem usar a própria lógica para
 resolver esses tipos de problemas?

 Considere que são verdadeiras as seguintes afirma coes:
 Se Adriane não  e inteligente, então Joyce e linda.
 Se Joyce não  e comunicativa, então  Erica não  e linda.
 Se Luciana não  e inteligente, então  Erica e comunicativa.
 Joyce e imatura.
 Deduza o atributo (imatura, inteligente, comunicativa ou linda) de cada
 uma
 das garotas, sabendo-se que  cada mo ca tem exatamente um atributo e duas
 mo
 cas quaisquer nao tem o mesmo atributo.
 --
 Prof Marcus

 =
 Instruções para entrar na lista, sair da lista e usar a lista em
 http://www.mat.puc-rio.br/~obmlistas/obm-l.html
 =



-- 
/**/
神が祝福

Torres

=
Instru��es para entrar na lista, sair da lista e usar a lista em
http://www.mat.puc-rio.br/~obmlistas/obm-l.html
=


Re: [obm-l] Teorema sobre o resto

2011-08-17 Por tôpico Johann Dirichlet
Eureka! 2, Divisibilidade, Congruências e Aritmética módulo n.
Em 17/08/11, Pedro Chavesbrped...@hotmail.com escreveu: Caros amigos, 
Como podemos provar o teorema abaixo? Dados n números inteiros (x_1, x_2, 
..., x_n),  cujo produto é P, então o resto da divisão de P por d (d é inteiro 
diferente de zero) é o resto da divisão do produto (r_1).( r_2) .( r_n) 
por d, onde r_1, r_2, ..., r_n são os respectivos restos das divisões de x_1, 
x_2, ..., x_n por d. Muitíssimo grato! Pedro Chaves 

= 
Instruções para entrar na lista, sair da lista e usar a lista em 
http://www.mat.puc-rio.br/~obmlistas/obm-l.html 
=

-- /**/神が祝福
Torres
=
Instru��es para entrar na lista, sair da lista e usar a lista em
http://www.mat.puc-rio.br/~obmlistas/obm-l.html
=


Re: [obm-l] Determinante nulo (outra vez)

2011-08-16 Por tôpico Johann Dirichlet
Bem, a melhor ideia é tentar usar o Lema de Gauss, e ir diminuindo a
ordem do determinante.
Um caso qualquer:

a b c
d e f
g h i

Suponha a!=0 (trocando linhas e colunas)

Podemos, usando transforma,cões lineares, obter isto:
a b c
0 E F
0 H I

Fatorando o a, temos
E F
H I

Siga daí!

Em 16/08/11, Paulo  Argolopauloarg...@bol.com.br escreveu:
 Colegas da Lista,

 Solicito, assim como já fez o Ennius, há algum tempo, ajuda na demonstração
 do teorema abaixo.
 TROREMA:
 O determinante de uma matriz quadrada M, de ordem maior que 1, só é nulo
 quando M possui alguma fila que seja combinação linear das filas paralelas
 (a essa fila).

 Um grande abraço do Paulo!
 =
 Instru�ões para entrar na lista, sair da lista e usar a lista em
 http://www.mat.puc-rio.br/~obmlistas/obm-l.html
 =



-- 
/**/
神が祝福

Torres

=
Instru��es para entrar na lista, sair da lista e usar a lista em
http://www.mat.puc-rio.br/~obmlistas/obm-l.html
=


Re: [obm-l] 2^70 + 3^70 eh divisivel por 13

2011-08-10 Por tôpico Johann Dirichlet
Use o Teorema de Fermat: 2^(12a+b)=2^b e 3^(12a+b)=3^b módulo 13

Em 10/08/11, Luís Lopesqed_te...@hotmail.com escreveu:

 Sauda,c~oes,

 Alguém poderia resolver?

Solicitaria a voce uma solução para a questão :
demonstre que 270 + 370 é divisível por 13.




 []'s
 Luis

   


-- 
/**/
神が祝福

Torres

=
Instru��es para entrar na lista, sair da lista e usar a lista em
http://www.mat.puc-rio.br/~obmlistas/obm-l.html
=


[obm-l] Re: [obm-l] Re: [obm-l] Demonstração

2011-08-05 Por tôpico Johann Dirichlet
a^2=3^k*b, em que 3 não divide b.

Sabemos que k1, pois 3 é divisor de a^2.
Mas k deve ser necessariamente par, pois os expoentes da foatoração de
um quadrado perfeito são pares. Logo k=2l, com l1.
Então a^2=3^(2l)*b, o que acarreta (a/(3^l))^2 = b. Portanto, como b é
inteiro, b é quadrado perfeito:

(a/(3^l))^2 = c^2
(a/(3^l)) = c
a = 3^l*c
Como l1, está provado: 3 é divisor de a.

Em 05/08/11, Ricardo Lopesricardo.blackj...@gmail.com escreveu:
 Multiplo de 3?

 Abraços

 Em 5 de agosto de 2011 14:33, Marcus Aurelio Gonçalves Rodrigues 
 marcusaureli...@globo.com escreveu:

 Alguém da uma forcinha?

 se a^2 e divisível por 3, então a também é?

 --
 Prof Marcus




 --
 Ricardo Shydo
 (71)8126-2111
 ricardo.lopesmore...@gmail.com
 ricardo.blackj...@gmail.com
 sh...@bol.com.br
 moreira_lopes2...@ig.com.br
 sh...@linuxmail.org
 ntsh...@hotmail.com



-- 
/**/
神が祝福

Torres

=
Instru��es para entrar na lista, sair da lista e usar a lista em
http://www.mat.puc-rio.br/~obmlistas/obm-l.html
=


[obm-l] Re: [obm-l] Re: [obm-l] Números Primos

2011-08-04 Por tôpico Johann Dirichlet
Bem, eu conheço um assim:

Como estudo de caso, seja 7 o primo que estamos pesquisando.

1 - Encontre um divisor da forma M*10+1. No caso, 7*3=21, M=2.

2 - A cada passo, faça isto aqui:
2a - Arranque o último dígito, e duplique-o (M=2, e 7*3=2*10+1);
2b - Subtraia do restante do número.

Por exemplo, 1001 é múltiplo de 7?

1001 = 100-2=98 = 9-2*8=-7, OK, pois 7 é múltiplo!

Encontrar divisores da forma 10K+1 é fácil, basta olhar a tabuada.

Em 03/08/11, regis barrosregisgbar...@yahoo.com.br escreveu:
 Boa Tarde Pessoal
 Gostaria algum material sobre criterio de divisibilidade que nesta lista
 mandou algum tempo atrás sobre o assunto e do qual não estou encontrando o
 email com o link sobre o assunto.

 Regis Godoy BarrosGraduado em Licenciatura em Fisica - IFSPGraduando em
 Licenciatura em Matemática - UNICAMP




-- 
/**/
神が祝福

Torres

=
Instru��es para entrar na lista, sair da lista e usar a lista em
http://www.mat.puc-rio.br/~obmlistas/obm-l.html
=


[obm-l] Re: [obm-l] Re: [obm-l] Re: [obm-l] Re: [obm-l] Números Primos

2011-08-04 Por tôpico Johann Dirichlet
7^a*11^b têm 16 divisores no total.
(a+1)(b+1)=16

Liste as possibilidades e finalize!


Em 04/08/11, Marcus Aurelio Gonçalves
Rodriguesmarcusaureli...@globo.com escreveu:
 Determine os números inteiros positivos cujos únicos divisores primos são 7
 e 11 e que possuem exatamente 15 divisores positivos diferentes de 1



-- 
/**/
神が祝福

Torres

=
Instru��es para entrar na lista, sair da lista e usar a lista em
http://www.mat.puc-rio.br/~obmlistas/obm-l.html
=


[obm-l] Re: [obm-l] ANÁLISE COMBINATÓRIA

2011-07-27 Por tôpico Johann Dirichlet
Bem, para o 2, dou uma dica: divida o intervalo [0,1] em n partes, e
pense onde cairiam as partes fracionárias dos Kx.

Em 27/07/11, Marcelo Costamat.mo...@gmail.com escreveu:
 *1 - Prove que dado qualquer conjunto de dez inteiros positivos de dois
 dígitos cada, é possível obter dois subconjuntos disjuntos cujos elementos
 têm a mesma soma.

 2 - Sejam x um número real e n um inteiro positivo. Mostre que entre os
 números x, 2x, 3x, . . ., (n – 1)x, existe um cuja distância a algum inteiro
 é, no máximo, 1/n.

 AGRADEÇO DESDE JÁ VOSSA ATENÇÃO
 *



-- 
/**/
神が祝福

Torres

=
Instru��es para entrar na lista, sair da lista e usar a lista em
http://www.mat.puc-rio.br/~obmlistas/obm-l.html
=


[obm-l] Re: [obm-l] RE: [obm-l] Soma dos dígitos de um número

2011-07-26 Por tôpico Johann Dirichlet
Bem, você tem mesmo que torcer para que isto tenha um só dígito. Umas
contas e desigualdades do tipo 'o maior número de 100 dígitos é
9...9' dão conta, mais um pouquinho de logaritmos.

Depois, usa módulo 9!

Em 25/07/11, Frederico Matosfrederi...@hotmail.com escreveu:

 Bem, se adotarmos que F(F(F(2000^2000) só tem um digito tomamos a seguinte
 provisão:
 como jah bem citou: F(F(F(F(2000^2000) = F(F(F(F(2^2000) jah que a soma dos
 digitos de um numero n.10^x é n.
 Agora veja como no 2 a soma do digitos eh cíclica:

 2^0 = 1   = 1
 2^1 = 2   = 2
 2^2 = 4   = 4
 2^3 = 8   = 8
 2^4 = 16 = 7
 2^5 = 32 = 5

 2^6   = 64 = 10 = 1
 2^7   = 128   = 11 = 2
 2^8   = 256   = 13 = 4
 2^9   = 512   = 8
 2^10 = 1024 = 7
 2^11 = 2048 = 14 = 5

 Então F(2^n) = F(2^n_mod(6))
 Logo F(2^2000) = F(2^2000mod(6)) = F(2^2) = 4





 From: joao_maldona...@hotmail.com
 To: obm-l@mat.puc-rio.br
 Subject: [obm-l] Soma dos dígitos de um número
 Date: Sun, 24 Jul 2011 13:08:05 -0300





 Dado a função F(x) =  soma dos dígitos de x,


 calcule F(F(F(F(2000^2000

 Parece que se aplicarmos inúmeras vezes F,até que o número só tenha um
 dígito, o resultado é o resto da divisão do número por 9 (também não sei
 porque),  a não ser que o  número seja divisível por 9, daí o resto é  9.
 Como  F(2000^ 2000) = F(2^2000)  não sendo divísivel 9, pela regra daria
 2^1998. 2^2 mod(9) =  (-1)^1998.4 = 4 mod(9)

 Mas ainda  falta provar  tal regra e também que  F(F(F(F( 2^2000 só  tem
 1 dígito

 []'s
 João
   


-- 
/**/
神が祝福

Torres

=
Instru��es para entrar na lista, sair da lista e usar a lista em
http://www.mat.puc-rio.br/~obmlistas/obm-l.html
=


[obm-l] Re: [obm-l] Re: [obm-l] Re: [obm-l] Essa ainda não consegui!!!

2011-07-26 Por tôpico Johann Dirichlet
Mas esse é bem mais moleza!
Os pontos são da forma (x_i,y_i)
Os médios são da forma ((x_i+x_j)/2,(y_i+y_j)/2)

Se conseguirmos garantir que existem dois pontos (x_i,y_i) e (x_j,y_j)
tais que as coordenadas x tenham igual paridade, bem como as
coordenadas y, acabou.

Se isto não ocorresse, o que se daria?
Temos pontos do tipo (par,par), (par, impar), (impar, par) e (impar, impar).
Como são cinco pontos, um dos tipos se repete. E achamos os pontos!

Agor, seria interessante se pudéssemos ver este problema acima. Creio
que existe um numero tao grande de pontos quantos se queira, de modo
que as coordenadas de intersecção sejam sempre fracionárias.



Em 24/07/11, Pedro Júniorpedromatematic...@gmail.com escreveu:
 Exatamente caríssimo Ralph, tens razão, é que estava tentanto lembrar do
 problema e fui escrevendo, mas vc me fez lembrar direitinho, como sempre!!!
 Parabéns.

 Em 24 de julho de 2011 11:23, Ralph Teixeira ralp...@gmail.com escreveu:

 Ah... aposto que o problema original era para mostrar que um dos PONTOS
 MEDIOS desses 10 segmentos tem coordenadas inteiras, nao? Ai tudo faz
 sentido: basta olhar a paridade de ambas as coordenadas. Ha 4 classes de
 possibilidades: (Par,Par), (Par, Impar), (Impar, Par), (Impar, Impar).
 Como
 voce tem 5 pontos, pombas, tem que haver dois deles dentro da mesma
 classe, digamos, X e Y. Mas entao as coordenadas de X+Y serao ambas
 pares,
 isto eh, as coordenadas do ponto medio (X+Y)/2 serao inteiras.

 Aposto 10 pratas que era esse o problema! Em dolar! :)

 Abraco,
 Ralph

 2011/7/24 Pedro Júnior pedromatematic...@gmail.com

 Sejam A, B, C, D e E pontos do plano cartesiano de coordenadas inteiras.
 Três quaisquer desses pontos não estão alinhados, logo formam dez
 segmentos.
 Mostre que pelo menos um dos pontos de intersecção desses segmentos é um
 ponto, também, de coordenadas inteiras.
 Desde já agradeço.

 --

 Pedro Jerônimo S. de O. Júnior

 Professor de Matemática

 Geo João Pessoa – PB





 --

 Pedro Jerônimo S. de O. Júnior

 Professor de Matemática

 Geo João Pessoa – PB



-- 
/**/
神が祝福

Torres

=
Instru��es para entrar na lista, sair da lista e usar a lista em
http://www.mat.puc-rio.br/~obmlistas/obm-l.html
=


[obm-l] Re: [obm-l] Re: [obm-l] Brasil conquista medalhas de Prata e Bronze na Olimpíada Internacional de Matemática (IMO)

2011-07-23 Por tôpico Johann Dirichlet
Valeu o papinho ufanista, mas... Cadê a prova da IMO, meu povo? Nunca
mais esta lista se divertiu resolvendo os problemas dela não?

Em 22/07/11, Fernando A Candeiasfacande...@gmail.com escreveu:
 É um juso motivo de orgulho para esta sofrida nação. E sem nenhum apoio do
 papai governo. Temos ótimos matemáticos, o que certamente eleva nossa auto
 estima, e  que  deveria ser  adequadamente  valorizado.

 Fernando Candeias
 Em 22 de julho de 2011 10:11, Olimpiada Brasileira de Matematica 
 o...@impa.br escreveu:

 **

 *Brasil conquista medalhas de Prata e Bronze*

 *na Olimpíada Internacional de Matemática (IMO)*


 O Brasil obteve um excelente resultado este ano na 52a. Olimpíada
 Internacional de Matemática (IMO), que acontece até o dia 24 de julho na
 cidade de Amsterdã na Holanda,* *conquistando três medalhas de prata e
 três de bronze. Os estudantes: André Macieira Braga (Belo Horizonte – MG),
 João Lucas Camelo Sá (Fortaleza – CE) e Henrique Fiúza do Nascimento
 (Brasília – DF), conquistaram as medalhas de prata, enquanto Débora
 Barbosa
 Alves (São Paulo – SP), Maria Clara Mendes Silva (Pirajuba – MG) e Gustavo
 Lisbôa Empinotti (Florianópolis – SC) conquistaram medalhas de bronze. Com
 este resultado o Brasil classificou em vigésimo lugar entre os países
 participantes.

 Considerada pela Unesco como a competição mais importante da área, a IMO
 contou este ano com a participação de 101 países reunindo 564 estudantes,
 entre 14 e 19 anos, mais talentosos do mundo no assunto. O Brasil foi
 representado por uma equipe de seis estudantes liderados pelos professores
 Nicolau
 Corção Saldanha (Rio de Janeiro – RJ) e Eduardo Tengan (São Carlos – SP).


 Um comitê internacional elegeu os problemas que seriam resolvidos entre os
 propostos pelos países participantes. As provas foram realizadas em dois
 dias consecutivos abrangendo disciplinas como Álgebra, Teoria dos números,
 Geometria e Combinatória. Em cada dia, os participantes resolveram três
 problemas, com valor de sete pontos cada, aplicados em 4 horas e meia de
 prova. A resolução destes problemas requer mais criatividade, engenho e
 habilidade em matemática do que conhecimentos e fórmulas aplicadas.

 *Brasil e as medalhas na IMO*

 A Olimpíada Internacional de Matemática (IMO) é realizada desde 1959. O
 Brasil participa da competição desde 1979 conquistando desde então um
 total
 de 96 medalhas, sendo oito de ouro, 26 de prata e 62 de bronze.

 A participação brasileira na competição é organizada através da Olimpíada
 Brasileira de Matemática (OBM), iniciativa que tem desempenhado um
 importante papel em relação à melhoria do ensino e descoberta de talentos
 para a pesquisa em Matemática nas modalidades de ensino fundamental e
 médio
 nas escolas públicas e privadas de todo o Brasil.  A Olimpíada Brasileira
 de Matemática é um projeto conjunto do Instituto Nacional de Matemática
 Pura
 e Aplicada (IMPA) da Sociedade Brasileira de Matemática (SBM) e conta com
 o
 apoio do Conselho Nacional de Desenvolvimento Científico e Tecnológico
 (CNPq) e do Instituto Nacional de Ciência e Tecnologia de Matemática
 (INCT–
 Mat).



 *Informações:*

 Nelly Carvajal – Assessoria de Imprensa

 Tel: 21-25295077 – Fax: 21-25295023

 e-mail:o...@impa.br

 --
 Secretaria da Olimpíada Brasileira de Matemática
 Estrada Dona Castorina, 110 Jd. Botânico,
 Rio de Janeiro - RJ, 22460-320, Brasil
 Tel: 55-21-25295077 Fax:55-21-25295023
 e-mail: o...@impa.br
 web site: www.obm.org.br





-- 
/**/
神が祝福

Torres

=
Instru��es para entrar na lista, sair da lista e usar a lista em
http://www.mat.puc-rio.br/~obmlistas/obm-l.html
=


[obm-l] Re: [obm-l] Fatorial [último dígito não nulo]

2011-07-23 Por tôpico Johann Dirichlet
É bem mais divertido saber qual é o último dígito diferente de zero de
um fatorial.
Tente!

Em 23/07/11, Victor Seixas Souzasouza@gmail.com escreveu:
 Existe uma fórmula geral para isso:
 http://latex.codecogs.com/gif.latex?\dpi{150}space;Nspace;=space;\sum_{k=1}^{\inftyspace;}space;\leftspace;\lfloorspace;\frac{n}{5^{k}}space;\rightspace;\rfloorhttp://latex.codecogs.com/gif.latex?\dpi{150}space;Nspace;=space;\sum_{k}^{\inftyspace;}space;\leftspace;\lfloorspace;\frac{n}{5^{k}}space;\rightspace;\rfloor

 N = quantidade de zeros em n!
 N = somatório de k=1 até infinito de (aproxima para baixo (n/5^k))

 Ou seja, para 1500 fatorial seria:
 1500/5 = 300
 1500/25 = 60
 1500/125 = 12
 1500/625 = 2.4 = 2
 1500/3125 = 0.4 = 0
 
 N = 300 + 60 + 12 + 2 + 0 + 0 + 0 + ... = 374

 Agora vou tentar explicar porque essa forma funciona.
 A chave para entender a fórmula é perceber que os multiplos de 2 são mais
 comuns do que os de 5.
 Em um produto de inteiros, a única forma de aparecer 0 na terminação é
 multiplicar por 10 = 5x2, explicita ou implicitamente.
 Mas,
 2x5 = 10
 4x25 = 100
 8x125 = 1000
 16x625 = 1
 ...
 2^n x 5^n = 10^n
 Como em o fatorial é um produtório, você teria de contar quantos pares 2ˆn x
 5ˆn você acha.
 Os 2 são desnecessários no caso do fatorial, pois sempre existirão e
 sobrarão multiplos de 2 em relação aos de 5.
 O Fato de que você vai somando as divisões por 5^n é que os produtos de 4x25
 produz 2 zeros, 8x125 produz 3 zeros, logo você precisa contar estes mais de
 uma vez, no caso, n vezes.
 Isso contudo não é uma prova, apenas um feeling e uma explicação que espero
 que esteja clara.

 Victor Seixas Souza



-- 
/**/
神が祝福

Torres

=
Instru��es para entrar na lista, sair da lista e usar a lista em
http://www.mat.puc-rio.br/~obmlistas/obm-l.html
=


[obm-l] Re: [obm-l] Questão Eureka 33

2011-07-21 Por tôpico Johann Dirichlet
Ué, você acabou de demonstrar! É claro, se todas as contas estiverem
corretas, você não precisa fazer mais nada.
Se para os casos abaixo de 8 não deu certo, só daria de 8 para cima.
Mas deu certo para 8, logo 8 é o mínimo!

Em 20/07/11, João Maldonadojoao_maldona...@hotmail.com escreveu:

 Olá
 3) Encontre o menor k  2 para o qual existem k números inteiros
 consecutivos, tais que a soma dos seus quadrados é um quadrado.
 Minha resolução:
 para  k =3
 (r-1)²+r²+(r+1)² = x²3r²+2 = x², x = 3n+1 ou 3n-1, x² = 3p+1, impossível
 para k = 44r²+4r+6 = x² - x² é múltiplo de 2 mas não de 4, impossível
 para k=55r²+10 = x²5(r²+2)=x²r²+2 = 5kr=5p+2, 5p-2, 5 p+1, 5p-15n+6 ou 5n +
 3 = 5k, impossível
 para k=66r²+6r+19 = x²6(r²+r+3)+1 = x²x=6p+  3, 6p+2, 6p-2, 6p+1, 6p-1temos
 x =  6p+1 ou 6p-1
 6(r²+r+3)+1 =  36p² -+ 12p + 1X = r² + r + 3 = 2(3p² +-p)
 ser é par, X é ímpar, se r é ímpar, X é ímpar
 para k = 7   7r² + 28 = x²7 (r²+ 4)  = x²
 r²+4 múltiplo de 7,

 r = 7p+1, 7p-1, 7p+2, 7p-2, 7p+3, 7p-3r²+4 =  7n -1, 7n-2, 7n+1, absurdo
 para k = 8

 8r²+8r+44 = x²
 4(2r²  +2r+11) = x²

 Não vejo nenhum problema aqui,  será k = 8 a resposta? Se sim, como provar?
 []'s,  Joaao







   


-- 
/**/
神が祝福

Torres

=
Instru��es para entrar na lista, sair da lista e usar a lista em
http://www.mat.puc-rio.br/~obmlistas/obm-l.html
=


Re: [obm-l] UMA SENHORA AJUDA

2011-07-18 Por tôpico Johann Dirichlet
Mas isso é maldade! Ele quer apenas a prova de que e^2 é irracional,
não da transcedencia (que é mais treta, mas sai). É matar um cachorro
com espingarda de elefante.

Te recomendo o famigerado Proofs from THE BOOK. Lá tem uma
demonstração para este e os casos e^racional.

Em 17/07/11, Artur Costa Steinersteinerar...@gmail.com escreveu:
 e é transcendente e potências inteiras de transcendentes são transcendentes.
 Como todo real trancendente é irracional, temos a conclusão desejada.
 Abraços.

 Artur Costa Steiner
 Em 17/07/2011 18:35, Marcelo Costa mat.mo...@gmail.com escreveu:

 PROVE QUE e^2 É IRRACIONAL.

 JÁ DEMONSTREI QUE e É IRRACINAL, MAS ACREDITE O PROCESSO NÃO SE ENCAIXA
 PARA e^2, SÓ SEI QUE SERÁ POR ABSURDO, ABRAÇOS A TODOS!



-- 
/**/
神が祝福

Torres

=
Instru��es para entrar na lista, sair da lista e usar a lista em
http://www.mat.puc-rio.br/~obmlistas/obm-l.html
=


[obm-l] Re: [obm-l] Re: [obm-l] Qual o software usado para as questões de geometria?

2011-07-18 Por tôpico Johann Dirichlet
Eu recomendo o CaR (Compass and Ruler). Muito simples e bem prático.
http://zirkel.sourceforge.net/

Em 18/07/11, regis barrosregisgbar...@yahoo.com.br escreveu:
 Bom dia João
 Você conhece o geogebra, este é um software com código livre, acho que serve
 para vocês.

 Regis
 --- Em dom, 17/7/11, João Maldonado joao_maldona...@hotmail.com escreveu:

 De: João Maldonado joao_maldona...@hotmail.com
 Assunto: [obm-l] Qual o software usado para as questões de geometria?
 Para: obm-l@mat.puc-rio.br
 Data: Domingo, 17 de Julho de 2011, 23:15






 Boa Tarde,

 Para quem não sabe estou construindo um site online (faz uma semana ), com
 preparação para  vestibulares focados no contexto de exatas, como ITA/ IME.


 Quem quiser entrar resumaovestibular.com
 Mas  fica muito difícil fazer construções geométricas no fireworks ou
 photoshop.  Alguém sabe de algum programa bom de desenvolvimento geométrico?
 Queria saber qual o programa usado nas construções  na prova da OBM  (isto
 claro, se o programa for aberto e seja possível me dizer).


 Muito   Obrigado,
 João      



-- 
/**/
神が祝福

Torres

=
Instru��es para entrar na lista, sair da lista e usar a lista em
http://www.mat.puc-rio.br/~obmlistas/obm-l.html
=


[obm-l] Re: [obm-l] Média harmônica igual média geométrica

2011-07-12 Por tôpico Johann Dirichlet
Eureka! 5, artigo do Caminha sobre desigualdades.

Dá uma boa lida, é um bom material introdutório sobre desigualdades em
olimpíadas. Aliás, a Eureka! é um farto material pra discussões desta
natureza.

Me desculpe pela secura da resposta, mas é que Desigualdade das Médias
é algo tão comum para mim - e muitos da lista - quanto o teorema de
Pitágoras. Eu não faria nada além de um C-y aqui.

Pra não dizer que fui um cara malvado :-P, eis um site:
http://erdos.ime.usp.br/index.php/Desigualdades

Em 11/07/11, enniusenn...@bol.com.br escreveu:
 Caros Colegas,

 Peço-lhes uma prova, se possível for, da propriedade abaixo.

 Propriedade: A média harmônica e a média geométrica, de n números reais
 positivos dados, são iguais somente se os n números forem todos iguais.

 Agradeço-lhes muitíssimo desde já.

 Ennius Lima
 =
 Instru�ões para entrar na lista, sair da lista e usar a lista em
 http://www.mat.puc-rio.br/~obmlistas/obm-l.html
 =



-- 
/**/
神が祝福

Torres

=
Instru��es para entrar na lista, sair da lista e usar a lista em
http://www.mat.puc-rio.br/~obmlistas/obm-l.html
=


Re: [obm-l] P(x^2+1)=P(x)^2+1

2011-07-08 Por tôpico Johann Dirichlet
Que coisa... Onde tá o meu erro, again??

De todo modo, o problema original tá resolvido!
Acho que testei casos de graus que darão errado, e não os que dariam
certo... Mas é facto: o polinômio deve ser par, e só terá expoentes
pares.

Em 07/07/11, Carlos Yuzo Shinecysh...@yahoo.com escreveu:
 Ué, P(x) = x^2 + 1 não dá certo? Essencialmente, sendo f(x) = x^2 + 1,
 estamos
 querendo resolver P(f(x)) = f(P(x)), e se f = P isso dá certo.

 Na verdade, f(x) = P(P(...P(x)...)) dá certo para qualquer quantidade de
 vezes
 que aplicamos P. Não sei se são todas as soluções, porém.

 []'s
 Shine


 - Original Message 
 From: Johann Dirichlet peterdirich...@gmail.com
 To: obm-l@mat.puc-rio.br
 Sent: Thu, July 7, 2011 10:01:22 AM
 Subject: Re: [obm-l] P(x^2+1)=P(x)^2+1

 Bem, voltando ao novo problema:
 P(x^2+1)=(P(x))^2+1.

 O polinômio é mônico, basta aplicar uma ideia básica de limite para
 saber o valor do coeficiente líder de P.

 Primeiro, se P(c)=c para algum c real então P(x)=x. É só usar a
 formula acima para achar infinitos x tais que P(x)=x. E dois
 polinomios que se intersectam infinitamente são iguais.

 Logo, podemos supor P(x) != x para todo x.

 O polinomio acima ou é par ou é ímpar. Basta ver que
 (P(x))^2=(P(-x))^2, e ou P(x)=P(-x) pu P(x)=-P(-x). Como pelo menos
 uma destas alternativas ocorre infinitas vezes, o polinômio ou é par
 ou é ímpar.

 Mas se P(x) fosse ímpar, P(0)=0. E sabemos que P(x)!=x. Logo, o
 polinomio procurado é par.

 Meu problema está sendo o seguinte: quando eu testo um polinômio (isso
 mesmo, P(x)=x^4+bx^2+c, substituir e suar a caneta!), ele falha
 miseravelmente.
 Creio que este problema está sem solução, also...

 Em 01/07/11, Ralph Teixeiraralp...@gmail.com escreveu:
 Melhorando aos poucos, ainda usando as ideias do Dirichlet: p(x) não pode
 ser ímpar. Se fosse, 0 seria raiz. Mas então 0^2+1=1 seria raiz, e 1^2+1=2
 seria raiz, e 2^2+1=5 seria raiz... e p(x) não pode ter infinitas raízes.
 Então estamos à procura de um polinômio **par** p(x) tal que
 p(x^2+1)=[p(x)]^2.

 Aliás, esse raciocínio mostra que esse p(x) não pode ter nenhuma raiz real
 -- se tiver uma raiz real x, terá infinitas, já que x^2+1x para todo x
 real.

 (Por enquanto, fico com a terrível impressão de que tal polinômio não
 existe... Alguém achou o dito cujo?)
 2011/7/1 Ralph Teixeira ralp...@gmail.com

 O raciocínio do Dirichlet mostra que basta achar UM polinômio (não
 constante) que tenha esta propriedade. Afinal, como ele mostrou, se p(x)
 serve, então q(x)=(p(x))^2 também serve.

 Mas seja lá quem for o polinômio mágico, eu sei que ou ele é um polinômio
 par ou ele é ímpar. Afinal, escreva p(x)=P(x)+I(x) onde P(x) tem apenas
 os
 termos de grau par e I(x) tem apenas os de grau ímpar.

 Ora, p(x)^2=(P^2+I^2)+2PI. Note que P^2+I^2 é um polinômio par e 2PI é
 ímpar.

 Mas a condição manda que p^2=p(x^2+1), que é uma função par. Então o
 termo
 2PI não pode existir, isto é, P=0 ou I=0. Assim, p(x) é par ou ímpar. E
 x^2-x+1 não é um nem outro, então não funcionou...

 Então precisamos ainda mostrar que existe UM tal polinômio!

 Abraço,
   Ralph

 P.S.: Tem certeza que o enunciado é esse mesmo? Não seria, sei lá,
 p(x^2+1)=(p(x))^2+1 ao invés?
 2011/7/1 Johann Dirichlet peterdirich...@gmail.com

 Em 01/07/11, Johann Dirichletpeterdirich...@gmail.com escreveu:
  Em 30/06/11, marcone augusto araújo
  borgesmarconeborge...@hotmail.com escreveu:
 
  1) Se p é inteiro primo ímpar,mostre que o numerador da fração
  1+1/2+1/3+...1/(p-1) é um múltiplo de p.
 
  1) Teorema de Wolstenholme, se não me engano...
 
  Bora lá, usar o velho truque das pontas de Gauss:
 1/k+1/(p-k)=p/(k(p-k));
  assim sendo, temos um monte de frações p/(alguma coisa). Esta coisa
  não será múltipla de p em momento nenhum, logo nada aniquila este
  fator p.
 
 
  2) Mostre que existem infinitos polinômios p(x) com coeficientes
  reais
  tais
  que p(x^2+1) = [p(x)]^2.

 É mais mole do que eu pensei!

 1 - Se P e Q são soluções da equação acima, P*Q também será. Óbvio!
 2 - Um polinômio possível é x^2-x+1.
 Como sei? Simples:

 Se L é um zero de P, então L^2+1 também será.
 Se eu conseguir L=L^2+1, terei uma solução pronta!
 Basta abrir o polinomio sem medo.


 P.S.: saber todas as soluções me parece mais desgastante. Aplicando a
 transformação T(L)=L^2+1 um numero finito de vezes, todos os
 polinômios dos pontos fixos são soluções. A treta é saber se não
 escapa nenhum (até porque muitos desses polinomios são fatoráveis, I
 think so).

 
  3) Uma corda AB,de comprimento constante,desliza sobre uma
  semicircunferência determinada por um diâmetro d.
  Considere o triângulo cujos vértices são: o ponto médio da corda e as
  projeções ortogonais dos seus extremos A e B
  sobre o diâmetro d.Mostre que ,durante o deslizamento da corda,esse
  triângulo é sempre isósceles e nunca muda de formato(i.é.,os ângulos
  do
  triângulo são constantes)
 
  Faz um desenho!
  Diâmetro r;centro O, raio 1; corda AB, tamanho d, médio M; AB

Re: [obm-l] P(x^2+1)=P(x)^2+1

2011-07-08 Por tôpico Johann Dirichlet
Bem, no avanço da ciencia, estou crente de algumas coisas:

1 - Fixado o grau, se o polinomio existir será único.
Quando eu abro os expoentes, não há margem para substituições arbitrárias.
Eu começo a crer nisto por experiência.

2 - O grau do polinômio seria uma potência de 2: grau 6 falha.

Em 08/07/11, Johann Dirichletpeterdirich...@gmail.com escreveu:
 Que coisa... Onde tá o meu erro, again??

 De todo modo, o problema original tá resolvido!
 Acho que testei casos de graus que darão errado, e não os que dariam
 certo... Mas é facto: o polinômio deve ser par, e só terá expoentes
 pares.

 Em 07/07/11, Carlos Yuzo Shinecysh...@yahoo.com escreveu:
 Ué, P(x) = x^2 + 1 não dá certo? Essencialmente, sendo f(x) = x^2 + 1,
 estamos
 querendo resolver P(f(x)) = f(P(x)), e se f = P isso dá certo.

 Na verdade, f(x) = P(P(...P(x)...)) dá certo para qualquer quantidade de
 vezes
 que aplicamos P. Não sei se são todas as soluções, porém.

 []'s
 Shine


 - Original Message 
 From: Johann Dirichlet peterdirich...@gmail.com
 To: obm-l@mat.puc-rio.br
 Sent: Thu, July 7, 2011 10:01:22 AM
 Subject: Re: [obm-l] P(x^2+1)=P(x)^2+1

 Bem, voltando ao novo problema:
 P(x^2+1)=(P(x))^2+1.

 O polinômio é mônico, basta aplicar uma ideia básica de limite para
 saber o valor do coeficiente líder de P.

 Primeiro, se P(c)=c para algum c real então P(x)=x. É só usar a
 formula acima para achar infinitos x tais que P(x)=x. E dois
 polinomios que se intersectam infinitamente são iguais.

 Logo, podemos supor P(x) != x para todo x.

 O polinomio acima ou é par ou é ímpar. Basta ver que
 (P(x))^2=(P(-x))^2, e ou P(x)=P(-x) pu P(x)=-P(-x). Como pelo menos
 uma destas alternativas ocorre infinitas vezes, o polinômio ou é par
 ou é ímpar.

 Mas se P(x) fosse ímpar, P(0)=0. E sabemos que P(x)!=x. Logo, o
 polinomio procurado é par.

 Meu problema está sendo o seguinte: quando eu testo um polinômio (isso
 mesmo, P(x)=x^4+bx^2+c, substituir e suar a caneta!), ele falha
 miseravelmente.
 Creio que este problema está sem solução, also...

 Em 01/07/11, Ralph Teixeiraralp...@gmail.com escreveu:
 Melhorando aos poucos, ainda usando as ideias do Dirichlet: p(x) não
 pode
 ser ímpar. Se fosse, 0 seria raiz. Mas então 0^2+1=1 seria raiz, e
 1^2+1=2
 seria raiz, e 2^2+1=5 seria raiz... e p(x) não pode ter infinitas
 raízes.
 Então estamos à procura de um polinômio **par** p(x) tal que
 p(x^2+1)=[p(x)]^2.

 Aliás, esse raciocínio mostra que esse p(x) não pode ter nenhuma raiz
 real
 -- se tiver uma raiz real x, terá infinitas, já que x^2+1x para todo x
 real.

 (Por enquanto, fico com a terrível impressão de que tal polinômio não
 existe... Alguém achou o dito cujo?)
 2011/7/1 Ralph Teixeira ralp...@gmail.com

 O raciocínio do Dirichlet mostra que basta achar UM polinômio (não
 constante) que tenha esta propriedade. Afinal, como ele mostrou, se
 p(x)
 serve, então q(x)=(p(x))^2 também serve.

 Mas seja lá quem for o polinômio mágico, eu sei que ou ele é um
 polinômio
 par ou ele é ímpar. Afinal, escreva p(x)=P(x)+I(x) onde P(x) tem apenas
 os
 termos de grau par e I(x) tem apenas os de grau ímpar.

 Ora, p(x)^2=(P^2+I^2)+2PI. Note que P^2+I^2 é um polinômio par e 2PI é
 ímpar.

 Mas a condição manda que p^2=p(x^2+1), que é uma função par. Então o
 termo
 2PI não pode existir, isto é, P=0 ou I=0. Assim, p(x) é par ou ímpar. E
 x^2-x+1 não é um nem outro, então não funcionou...

 Então precisamos ainda mostrar que existe UM tal polinômio!

 Abraço,
   Ralph

 P.S.: Tem certeza que o enunciado é esse mesmo? Não seria, sei lá,
 p(x^2+1)=(p(x))^2+1 ao invés?
 2011/7/1 Johann Dirichlet peterdirich...@gmail.com

 Em 01/07/11, Johann Dirichletpeterdirich...@gmail.com escreveu:
  Em 30/06/11, marcone augusto araújo
  borgesmarconeborge...@hotmail.com escreveu:
 
  1) Se p é inteiro primo ímpar,mostre que o numerador da fração
  1+1/2+1/3+...1/(p-1) é um múltiplo de p.
 
  1) Teorema de Wolstenholme, se não me engano...
 
  Bora lá, usar o velho truque das pontas de Gauss:
 1/k+1/(p-k)=p/(k(p-k));
  assim sendo, temos um monte de frações p/(alguma coisa). Esta coisa
  não será múltipla de p em momento nenhum, logo nada aniquila este
  fator p.
 
 
  2) Mostre que existem infinitos polinômios p(x) com coeficientes
  reais
  tais
  que p(x^2+1) = [p(x)]^2.

 É mais mole do que eu pensei!

 1 - Se P e Q são soluções da equação acima, P*Q também será. Óbvio!
 2 - Um polinômio possível é x^2-x+1.
 Como sei? Simples:

 Se L é um zero de P, então L^2+1 também será.
 Se eu conseguir L=L^2+1, terei uma solução pronta!
 Basta abrir o polinomio sem medo.


 P.S.: saber todas as soluções me parece mais desgastante. Aplicando a
 transformação T(L)=L^2+1 um numero finito de vezes, todos os
 polinômios dos pontos fixos são soluções. A treta é saber se não
 escapa nenhum (até porque muitos desses polinomios são fatoráveis, I
 think so).

 
  3) Uma corda AB,de comprimento constante,desliza sobre uma
  semicircunferência determinada por um diâmetro d

Re: [obm-l] P(x^2+1)=P(x)^2+1

2011-07-07 Por tôpico Johann Dirichlet
Bem, voltando ao novo problema:
P(x^2+1)=(P(x))^2+1.

O polinômio é mônico, basta aplicar uma ideia básica de limite para
saber o valor do coeficiente líder de P.

Primeiro, se P(c)=c para algum c real então P(x)=x. É só usar a
formula acima para achar infinitos x tais que P(x)=x. E dois
polinomios que se intersectam infinitamente são iguais.

Logo, podemos supor P(x) != x para todo x.

O polinomio acima ou é par ou é ímpar. Basta ver que
(P(x))^2=(P(-x))^2, e ou P(x)=P(-x) pu P(x)=-P(-x). Como pelo menos
uma destas alternativas ocorre infinitas vezes, o polinômio ou é par
ou é ímpar.

Mas se P(x) fosse ímpar, P(0)=0. E sabemos que P(x)!=x. Logo, o
polinomio procurado é par.

Meu problema está sendo o seguinte: quando eu testo um polinômio (isso
mesmo, P(x)=x^4+bx^2+c, substituir e suar a caneta!), ele falha
miseravelmente.
Creio que este problema está sem solução, also...

Em 01/07/11, Ralph Teixeiraralp...@gmail.com escreveu:
 Melhorando aos poucos, ainda usando as ideias do Dirichlet: p(x) não pode
 ser ímpar. Se fosse, 0 seria raiz. Mas então 0^2+1=1 seria raiz, e 1^2+1=2
 seria raiz, e 2^2+1=5 seria raiz... e p(x) não pode ter infinitas raízes.
 Então estamos à procura de um polinômio **par** p(x) tal que
 p(x^2+1)=[p(x)]^2.

 Aliás, esse raciocínio mostra que esse p(x) não pode ter nenhuma raiz real
 -- se tiver uma raiz real x, terá infinitas, já que x^2+1x para todo x
 real.

 (Por enquanto, fico com a terrível impressão de que tal polinômio não
 existe... Alguém achou o dito cujo?)
 2011/7/1 Ralph Teixeira ralp...@gmail.com

 O raciocínio do Dirichlet mostra que basta achar UM polinômio (não
 constante) que tenha esta propriedade. Afinal, como ele mostrou, se p(x)
 serve, então q(x)=(p(x))^2 também serve.

 Mas seja lá quem for o polinômio mágico, eu sei que ou ele é um polinômio
 par ou ele é ímpar. Afinal, escreva p(x)=P(x)+I(x) onde P(x) tem apenas os
 termos de grau par e I(x) tem apenas os de grau ímpar.

 Ora, p(x)^2=(P^2+I^2)+2PI. Note que P^2+I^2 é um polinômio par e 2PI é
 ímpar.

 Mas a condição manda que p^2=p(x^2+1), que é uma função par. Então o termo
 2PI não pode existir, isto é, P=0 ou I=0. Assim, p(x) é par ou ímpar. E
 x^2-x+1 não é um nem outro, então não funcionou...

 Então precisamos ainda mostrar que existe UM tal polinômio!

 Abraço,
   Ralph

 P.S.: Tem certeza que o enunciado é esse mesmo? Não seria, sei lá,
 p(x^2+1)=(p(x))^2+1 ao invés?
 2011/7/1 Johann Dirichlet peterdirich...@gmail.com

 Em 01/07/11, Johann Dirichletpeterdirich...@gmail.com escreveu:
  Em 30/06/11, marcone augusto araújo
  borgesmarconeborge...@hotmail.com escreveu:
 
  1) Se p é inteiro primo ímpar,mostre que o numerador da fração
  1+1/2+1/3+...1/(p-1) é um múltiplo de p.
 
  1) Teorema de Wolstenholme, se não me engano...
 
  Bora lá, usar o velho truque das pontas de Gauss:
 1/k+1/(p-k)=p/(k(p-k));
  assim sendo, temos um monte de frações p/(alguma coisa). Esta coisa
  não será múltipla de p em momento nenhum, logo nada aniquila este
  fator p.
 
 
  2) Mostre que existem infinitos polinômios p(x) com coeficientes reais
  tais
  que p(x^2+1) = [p(x)]^2.

 É mais mole do que eu pensei!

 1 - Se P e Q são soluções da equação acima, P*Q também será. Óbvio!
 2 - Um polinômio possível é x^2-x+1.
 Como sei? Simples:

 Se L é um zero de P, então L^2+1 também será.
 Se eu conseguir L=L^2+1, terei uma solução pronta!
 Basta abrir o polinomio sem medo.


 P.S.: saber todas as soluções me parece mais desgastante. Aplicando a
 transformação T(L)=L^2+1 um numero finito de vezes, todos os
 polinômios dos pontos fixos são soluções. A treta é saber se não
 escapa nenhum (até porque muitos desses polinomios são fatoráveis, I
 think so).

 
  3) Uma corda AB,de comprimento constante,desliza sobre uma
  semicircunferência determinada por um diâmetro d.
  Considere o triângulo cujos vértices são: o ponto médio da corda e as
  projeções ortogonais dos seus extremos A e B
  sobre o diâmetro d.Mostre que ,durante o deslizamento da corda,esse
  triângulo é sempre isósceles e nunca muda de formato(i.é.,os ângulos
  do
  triângulo são constantes)
 
  Faz um desenho!
  Diâmetro r;centro O, raio 1; corda AB, tamanho d, médio M; AB
  projetado em r dá XY.
 
  O triangulo AOB é obviamente isósceles.
  Os quadrilateros XOMA e YOMB são inscritíveis de diâmetros OA e OB
  respectivamente (angulos de 90 graus).
 
  Temos OXM=OAM=OBM=OYM, logo XMY é isosceles. E o angulo OBA depende
  unicamente de d.
 
  P.S.: duvido que os triangulos sejam todos congruentes. O angulo XOM
  define o tamanho de XM.
 
 
  Meus agradecimentos por qualquer esclarecimento.
 
 
  --
  /**/
  神が祝福
 
  Torres
 


 --
 /**/
 神が祝福

 Torres

 =
 Instru�ões para entrar na lista, sair da lista e usar a lista em
 http://www.mat.puc-rio.br/~obmlistas/obm-l.html

Re: [obm-l] Problemas(polinomoi- ideias!)

2011-07-06 Por tôpico Johann Dirichlet
Continuando: acho que, quando se faz alguma manipulação algébrica, a
conta falha miseravelmente para graus grandes.

Usando a ideia do Ralph, o polinomio em questão é par ou ímpar. Mas
quando eu abro as contas, usando um exemplo finito (uma tentativa do
genero f(x)=ax^2+bx+c), dá muito desencontro de expoentes. Vou mostrar
as contas outro dia (elas sao grandinhas!).

Em 05/07/11, Johann Dirichletpeterdirich...@gmail.com escreveu:
 Eu, na verdade, tentei achar um polinomio que desse certo. E cantei
 vitória antes do tempo...
 E a sua ideia de par-ou-impar matou de vez as esperanças: L^2+1
 aumenta o módulo.

 O Marcone tambem me enviou este e-mail corrigido. Eu estou matutando
 nele, e achei alguns exemplos. Ao que me parece, para cada grau de
 polinomio, existe um único polinômio que resolve a equação funcional.

 Eu descobri que estes poucos polinomios tem coeficiente líder 1 e
 sub-líder 0. Mas não avencei muito para dar um parecer final.

 No mais, acho que x^2+1 me lembra de usar i^2=-1...

 Em 04/07/11, Ralph Teixeiraralp...@gmail.com escreveu:
 Como voce disse, se a eh uma raiz de P(x), entao a^2+1 tem que ser raiz
 de
 P(x) tambem. Entao se voce pegar as raizes de P(x) e aplicar x^2+1
 nelas,
 voce ainda tem que cair em raizes. Portanto, dada uma raiz qualquer a,
 temos
 que a^2+1, (a^2+1)^2+1, etc. gera varias raizes de P(x). Como P(x) tem
 que
 ter um numero finito de raizes distintas, essa sequencia tem que gerar um
 ciclo, tem que repetir em algum momento.

 Que ciclo? O caso mais simples seria fazer um ciclo de tamanho 1, ou
 seja,
 fazer logo a^2+1=a, para o ciclo soh ter um termo. Foi a ideia que voce
 colocou... mas nao dah certo -- a condicao deste ciclo ser finita eh
 NECESSARIA para ter a igualdade pedida, mas ter um ciclo de raizes nao eh
 SUFICENTE para garantir a igualdade pedida.

 O Shine botou a bola embaixo do braco e levou para casa: nao ha polinomio
 com a condicao pedida... :)

 Mas, olha soh:* o Marcone, que propos o problema original, me mandou um
 E-mail dizendo que realmente o enunciado original era mesmo
 p(x^2+1)=(p(x))^2+1, mas por algum motivo ele nao conseguiu colocar a
 correcao na lista.* Entao ainda ha um problema interessante (mas bem
 diferente) para fazer.

 (Eu jah vi isso em algum lugar, mas nao lembro onde...)

 Abraco,
 Ralph
 2011/7/4 Johann Dirichlet peterdirich...@gmail.com

 Puxa! Mas onde esta o erro da minha solução?

 Anyway, inicialmente pensei em fatorar o dito polinomio.
 Creio que ele seja mônico, abrindo a expressão geral o fator máximo é
 a^2=a.
 Aí, escreve ele na forma deprodutos (x-a_i).. Basicamente, um lado
 fica na forma
 x^2+1-a_ i, e o outro como (x-a_ i)^2. Supóndo que as raízes são, em
 alguma ordem, iguais, dá pra chegar em algum lugar.




 Em 01/07/11, Ralph Teixeiraralp...@gmail.com escreveu:
  O raciocínio do Dirichlet mostra que basta achar UM polinômio (não
  constante) que tenha esta propriedade. Afinal, como ele mostrou, se
  p(x)
  serve, então q(x)=(p(x))^2 também serve.
 
  Mas seja lá quem for o polinômio mágico, eu sei que ou ele é um
  polinômio
  par ou ele é ímpar. Afinal, escreva p(x)=P(x)+I(x) onde P(x) tem
  apenas
 os
  termos de grau par e I(x) tem apenas os de grau ímpar.
 
  Ora, p(x)^2=(P^2+I^2)+2PI. Note que P^2+I^2 é um polinômio par e 2PI é
  ímpar.
 
  Mas a condição manda que p^2=p(x^2+1), que é uma função par. Então o
 termo
  2PI não pode existir, isto é, P=0 ou I=0. Assim, p(x) é par ou ímpar.
  E
  x^2-x+1 não é um nem outro, então não funcionou...
 
  Então precisamos ainda mostrar que existe UM tal polinômio!
 
  Abraço,
Ralph
 
  P.S.: Tem certeza que o enunciado é esse mesmo? Não seria, sei lá,
  p(x^2+1)=(p(x))^2+1 ao invés?
  2011/7/1 Johann Dirichlet peterdirich...@gmail.com
 
  Em 01/07/11, Johann Dirichletpeterdirich...@gmail.com escreveu:
   Em 30/06/11, marcone augusto araújo
   borgesmarconeborge...@hotmail.com escreveu:
  
   1) Se p é inteiro primo ímpar,mostre que o numerador da fração
   1+1/2+1/3+...1/(p-1) é um múltiplo de p.
  
   1) Teorema de Wolstenholme, se não me engano...
  
   Bora lá, usar o velho truque das pontas de Gauss:
   1/k+1/(p-k)=p/(k(p-k));
   assim sendo, temos um monte de frações p/(alguma coisa). Esta coisa
   não será múltipla de p em momento nenhum, logo nada aniquila este
   fator p.
  
  
   2) Mostre que existem infinitos polinômios p(x) com coeficientes
 reais
   tais
   que p(x^2+1) = [p(x)]^2.
 
  É mais mole do que eu pensei!
 
  1 - Se P e Q são soluções da equação acima, P*Q também será. Óbvio!
  2 - Um polinômio possível é x^2-x+1.
  Como sei? Simples:
 
  Se L é um zero de P, então L^2+1 também será.
  Se eu conseguir L=L^2+1, terei uma solução pronta!
  Basta abrir o polinomio sem medo.
 
 
  P.S.: saber todas as soluções me parece mais desgastante. Aplicando a
  transformação T(L)=L^2+1 um numero finito de vezes, todos os
  polinômios dos pontos fixos são soluções. A treta é saber se não
  escapa nenhum (até porque muitos desses

Re: [obm-l] Problemas(polinomoi- ideias!)

2011-07-05 Por tôpico Johann Dirichlet
Eu, na verdade, tentei achar um polinomio que desse certo. E cantei
vitória antes do tempo...
E a sua ideia de par-ou-impar matou de vez as esperanças: L^2+1
aumenta o módulo.

O Marcone tambem me enviou este e-mail corrigido. Eu estou matutando
nele, e achei alguns exemplos. Ao que me parece, para cada grau de
polinomio, existe um único polinômio que resolve a equação funcional.

Eu descobri que estes poucos polinomios tem coeficiente líder 1 e
sub-líder 0. Mas não avencei muito para dar um parecer final.

No mais, acho que x^2+1 me lembra de usar i^2=-1...

Em 04/07/11, Ralph Teixeiraralp...@gmail.com escreveu:
 Como voce disse, se a eh uma raiz de P(x), entao a^2+1 tem que ser raiz de
 P(x) tambem. Entao se voce pegar as raizes de P(x) e aplicar x^2+1 nelas,
 voce ainda tem que cair em raizes. Portanto, dada uma raiz qualquer a, temos
 que a^2+1, (a^2+1)^2+1, etc. gera varias raizes de P(x). Como P(x) tem que
 ter um numero finito de raizes distintas, essa sequencia tem que gerar um
 ciclo, tem que repetir em algum momento.

 Que ciclo? O caso mais simples seria fazer um ciclo de tamanho 1, ou seja,
 fazer logo a^2+1=a, para o ciclo soh ter um termo. Foi a ideia que voce
 colocou... mas nao dah certo -- a condicao deste ciclo ser finita eh
 NECESSARIA para ter a igualdade pedida, mas ter um ciclo de raizes nao eh
 SUFICENTE para garantir a igualdade pedida.

 O Shine botou a bola embaixo do braco e levou para casa: nao ha polinomio
 com a condicao pedida... :)

 Mas, olha soh:* o Marcone, que propos o problema original, me mandou um
 E-mail dizendo que realmente o enunciado original era mesmo
 p(x^2+1)=(p(x))^2+1, mas por algum motivo ele nao conseguiu colocar a
 correcao na lista.* Entao ainda ha um problema interessante (mas bem
 diferente) para fazer.

 (Eu jah vi isso em algum lugar, mas nao lembro onde...)

 Abraco,
 Ralph
 2011/7/4 Johann Dirichlet peterdirich...@gmail.com

 Puxa! Mas onde esta o erro da minha solução?

 Anyway, inicialmente pensei em fatorar o dito polinomio.
 Creio que ele seja mônico, abrindo a expressão geral o fator máximo é
 a^2=a.
 Aí, escreve ele na forma deprodutos (x-a_i).. Basicamente, um lado
 fica na forma
 x^2+1-a_ i, e o outro como (x-a_ i)^2. Supóndo que as raízes são, em
 alguma ordem, iguais, dá pra chegar em algum lugar.




 Em 01/07/11, Ralph Teixeiraralp...@gmail.com escreveu:
  O raciocínio do Dirichlet mostra que basta achar UM polinômio (não
  constante) que tenha esta propriedade. Afinal, como ele mostrou, se p(x)
  serve, então q(x)=(p(x))^2 também serve.
 
  Mas seja lá quem for o polinômio mágico, eu sei que ou ele é um
  polinômio
  par ou ele é ímpar. Afinal, escreva p(x)=P(x)+I(x) onde P(x) tem apenas
 os
  termos de grau par e I(x) tem apenas os de grau ímpar.
 
  Ora, p(x)^2=(P^2+I^2)+2PI. Note que P^2+I^2 é um polinômio par e 2PI é
  ímpar.
 
  Mas a condição manda que p^2=p(x^2+1), que é uma função par. Então o
 termo
  2PI não pode existir, isto é, P=0 ou I=0. Assim, p(x) é par ou ímpar. E
  x^2-x+1 não é um nem outro, então não funcionou...
 
  Então precisamos ainda mostrar que existe UM tal polinômio!
 
  Abraço,
Ralph
 
  P.S.: Tem certeza que o enunciado é esse mesmo? Não seria, sei lá,
  p(x^2+1)=(p(x))^2+1 ao invés?
  2011/7/1 Johann Dirichlet peterdirich...@gmail.com
 
  Em 01/07/11, Johann Dirichletpeterdirich...@gmail.com escreveu:
   Em 30/06/11, marcone augusto araújo
   borgesmarconeborge...@hotmail.com escreveu:
  
   1) Se p é inteiro primo ímpar,mostre que o numerador da fração
   1+1/2+1/3+...1/(p-1) é um múltiplo de p.
  
   1) Teorema de Wolstenholme, se não me engano...
  
   Bora lá, usar o velho truque das pontas de Gauss:
   1/k+1/(p-k)=p/(k(p-k));
   assim sendo, temos um monte de frações p/(alguma coisa). Esta coisa
   não será múltipla de p em momento nenhum, logo nada aniquila este
   fator p.
  
  
   2) Mostre que existem infinitos polinômios p(x) com coeficientes
 reais
   tais
   que p(x^2+1) = [p(x)]^2.
 
  É mais mole do que eu pensei!
 
  1 - Se P e Q são soluções da equação acima, P*Q também será. Óbvio!
  2 - Um polinômio possível é x^2-x+1.
  Como sei? Simples:
 
  Se L é um zero de P, então L^2+1 também será.
  Se eu conseguir L=L^2+1, terei uma solução pronta!
  Basta abrir o polinomio sem medo.
 
 
  P.S.: saber todas as soluções me parece mais desgastante. Aplicando a
  transformação T(L)=L^2+1 um numero finito de vezes, todos os
  polinômios dos pontos fixos são soluções. A treta é saber se não
  escapa nenhum (até porque muitos desses polinomios são fatoráveis, I
  think so).
 
  
   3) Uma corda AB,de comprimento constante,desliza sobre uma
   semicircunferência determinada por um diâmetro d.
   Considere o triângulo cujos vértices são: o ponto médio da corda e
   as
   projeções ortogonais dos seus extremos A e B
   sobre o diâmetro d.Mostre que ,durante o deslizamento da corda,esse
   triângulo é sempre isósceles e nunca muda de formato(i.é.,os ângulos
 do
   triângulo são

[obm-l] Correção do Enunciado [era: Enunciado errado(lista da obm)]

2011-07-04 Por tôpico Johann Dirichlet
Reescrevendo: p(x^2+1)=(p(x))^2+1
É isto?
Vou dar uma pensada, mas acho que a ideia das raízes ainda rola... Ou
um dose de complexos?

Em 02/07/11, marcone augusto araújo
borgesmarconeborge...@hotmail.com escreveu:

 Desculpe enviar diretamente,tentei várias vezes pela lista,não consegui.
 Na questão 2,onde tá escrito [p(x)]^2,o correto é [p(x)]^2 + 1.
 Perdão pelo erro.
 Abraço,Marcone.   


-- 
/**/
神が祝福

Torres

=
Instru��es para entrar na lista, sair da lista e usar a lista em
http://www.mat.puc-rio.br/~obmlistas/obm-l.html
=


Re: [obm-l] Problemas(polinomoi- ideias!)

2011-07-04 Por tôpico Johann Dirichlet
Puxa! Mas onde esta o erro da minha solução?

Anyway, inicialmente pensei em fatorar o dito polinomio.
Creio que ele seja mônico, abrindo a expressão geral o fator máximo é a^2=a.
Aí, escreve ele na forma deprodutos (x-a_i).. Basicamente, um lado
fica na forma
x^2+1-a_ i, e o outro como (x-a_ i)^2. Supóndo que as raízes são, em
alguma ordem, iguais, dá pra chegar em algum lugar.




Em 01/07/11, Ralph Teixeiraralp...@gmail.com escreveu:
 O raciocínio do Dirichlet mostra que basta achar UM polinômio (não
 constante) que tenha esta propriedade. Afinal, como ele mostrou, se p(x)
 serve, então q(x)=(p(x))^2 também serve.

 Mas seja lá quem for o polinômio mágico, eu sei que ou ele é um polinômio
 par ou ele é ímpar. Afinal, escreva p(x)=P(x)+I(x) onde P(x) tem apenas os
 termos de grau par e I(x) tem apenas os de grau ímpar.

 Ora, p(x)^2=(P^2+I^2)+2PI. Note que P^2+I^2 é um polinômio par e 2PI é
 ímpar.

 Mas a condição manda que p^2=p(x^2+1), que é uma função par. Então o termo
 2PI não pode existir, isto é, P=0 ou I=0. Assim, p(x) é par ou ímpar. E
 x^2-x+1 não é um nem outro, então não funcionou...

 Então precisamos ainda mostrar que existe UM tal polinômio!

 Abraço,
   Ralph

 P.S.: Tem certeza que o enunciado é esse mesmo? Não seria, sei lá,
 p(x^2+1)=(p(x))^2+1 ao invés?
 2011/7/1 Johann Dirichlet peterdirich...@gmail.com

 Em 01/07/11, Johann Dirichletpeterdirich...@gmail.com escreveu:
  Em 30/06/11, marcone augusto araújo
  borgesmarconeborge...@hotmail.com escreveu:
 
  1) Se p é inteiro primo ímpar,mostre que o numerador da fração
  1+1/2+1/3+...1/(p-1) é um múltiplo de p.
 
  1) Teorema de Wolstenholme, se não me engano...
 
  Bora lá, usar o velho truque das pontas de Gauss:
  1/k+1/(p-k)=p/(k(p-k));
  assim sendo, temos um monte de frações p/(alguma coisa). Esta coisa
  não será múltipla de p em momento nenhum, logo nada aniquila este
  fator p.
 
 
  2) Mostre que existem infinitos polinômios p(x) com coeficientes reais
  tais
  que p(x^2+1) = [p(x)]^2.

 É mais mole do que eu pensei!

 1 - Se P e Q são soluções da equação acima, P*Q também será. Óbvio!
 2 - Um polinômio possível é x^2-x+1.
 Como sei? Simples:

 Se L é um zero de P, então L^2+1 também será.
 Se eu conseguir L=L^2+1, terei uma solução pronta!
 Basta abrir o polinomio sem medo.


 P.S.: saber todas as soluções me parece mais desgastante. Aplicando a
 transformação T(L)=L^2+1 um numero finito de vezes, todos os
 polinômios dos pontos fixos são soluções. A treta é saber se não
 escapa nenhum (até porque muitos desses polinomios são fatoráveis, I
 think so).

 
  3) Uma corda AB,de comprimento constante,desliza sobre uma
  semicircunferência determinada por um diâmetro d.
  Considere o triângulo cujos vértices são: o ponto médio da corda e as
  projeções ortogonais dos seus extremos A e B
  sobre o diâmetro d.Mostre que ,durante o deslizamento da corda,esse
  triângulo é sempre isósceles e nunca muda de formato(i.é.,os ângulos do
  triângulo são constantes)
 
  Faz um desenho!
  Diâmetro r;centro O, raio 1; corda AB, tamanho d, médio M; AB
  projetado em r dá XY.
 
  O triangulo AOB é obviamente isósceles.
  Os quadrilateros XOMA e YOMB são inscritíveis de diâmetros OA e OB
  respectivamente (angulos de 90 graus).
 
  Temos OXM=OAM=OBM=OYM, logo XMY é isosceles. E o angulo OBA depende
  unicamente de d.
 
  P.S.: duvido que os triangulos sejam todos congruentes. O angulo XOM
  define o tamanho de XM.
 
 
  Meus agradecimentos por qualquer esclarecimento.
 
 
  --
  /**/
  神が祝福
 
  Torres
 


 --
 /**/
 神が祝福

 Torres

 =
 Instru�ões para entrar na lista, sair da lista e usar a lista em
 http://www.mat.puc-rio.br/~obmlistas/obm-l.html
 =




-- 
/**/
神が祝福

Torres

=
Instru��es para entrar na lista, sair da lista e usar a lista em
http://www.mat.puc-rio.br/~obmlistas/obm-l.html
=


Re: [obm-l] Problemas(polinomoi- ideias!)

2011-07-04 Por tôpico Johann Dirichlet
Sobre a sua mesg: por indução infinita na derivação, nunca acharemos
um polinômio?

Comments:
Bem, não tô com a mensagem full, mas basicamente o Ralph provou que o
polinômio é par ou ímpar. Não dá pra ser coluna do meio.
Realmente, raiz 0 falha miseravelmente, isto é imediato. Mas, o lance
é: não existe nenhum polinômio que funcione??

Realmente,pensando na minha ideia das raizes, pareece que L^2+1 tem
modulo maior que L (uma desigualdade das medias mataria isso). Ou
seja, não tem como, depois de aplicar L^2+1 um numero finito de vezes,
voltar a L.

Quanto a outra, comecei a atacar hoje.


Em 04/07/11, Carlos Yuzo Shinecysh...@yahoo.com escreveu:
 Eu acho que não existem infinitos polinômios (a única solução é o polinômio
 nulo). Isso, é claro, falando do problema P(x^2 + 1) = [P(x)]^2.

 Antes, lembramos que zero não pode ser raiz de P: como o Ralph já provou,
 isso
 gera infinitas raízes e aí P é o polinômio nulo.

 Olha só: derivando P(x^2 + 1) = P(x)^2 dos dois lados encontramos
 2xP'(x^2+1) =
 2P(x)P'(x). Aí vou tentar usar uma ideia parecida com a do Ralph. x = 0 só
 pode
 ser raiz de P' (o que atesta o fato de P ser par, que também foi provado).
 Agora
 note que se x diferente de zero é raiz de P' então x^2 + 1 também é. Pena
 que só
 temos o zero. Mas aí fazemos de trás para frente (isto é, aplicamos uma
 inversa
 f de x^2 + 1): troca x = f(0) = i. Então i é raiz de P ou P'. Mas se for
 raiz de
 P, então i^2 + 1 = 0 é raiz e dá problema do mesmo jeito. Então é raiz de
 P'.
 Mas aí é só continuar.

 Definindo f melhor: f(r) é a raiz quadrada de r - 1 que tem argumento arg(r
 -
 1)/2. A minha pergunta é: partindo do r = 0, ele entra em loop?

 Eu acho que não. De fato, f é injetiva: f(r) = f(s) implica (f(r))^2 + 1 =
 (f(s))^2 + 1 que é o mesmo que r = s. Então se a sequência n_0 = 0 e n_k =
 f(n_{k-1}) é periódica, ela é puramente periódica. Mas 0 não é periódica
 para
 f^{-1}(x) = x^2 + 1, então acabou (eu acho).


 Isso faz sentido?

 []'s
 Shine


 - Original Message 
 From: Johann Dirichlet peterdirich...@gmail.com
 To: obm-l@mat.puc-rio.br
 Sent: Mon, July 4, 2011 12:46:11 PM
 Subject: Re: [obm-l] Problemas(polinomoi- ideias!)

 Puxa! Mas onde esta o erro da minha solução?

 Anyway, inicialmente pensei em fatorar o dito polinomio.
 Creio que ele seja mônico, abrindo a expressão geral o fator máximo é a^2=a.
 Aí, escreve ele na forma deprodutos (x-a_i).. Basicamente, um lado
 fica na forma
 x^2+1-a_ i, e o outro como (x-a_ i)^2. Supóndo que as raízes são, em
 alguma ordem, iguais, dá pra chegar em algum lugar.




 Em 01/07/11, Ralph Teixeiraralp...@gmail.com escreveu:
 O raciocínio do Dirichlet mostra que basta achar UM polinômio (não
 constante) que tenha esta propriedade. Afinal, como ele mostrou, se p(x)
 serve, então q(x)=(p(x))^2 também serve.

 Mas seja lá quem for o polinômio mágico, eu sei que ou ele é um polinômio
 par ou ele é ímpar. Afinal, escreva p(x)=P(x)+I(x) onde P(x) tem apenas os
 termos de grau par e I(x) tem apenas os de grau ímpar.

 Ora, p(x)^2=(P^2+I^2)+2PI. Note que P^2+I^2 é um polinômio par e 2PI é
 ímpar.

 Mas a condição manda que p^2=p(x^2+1), que é uma função par. Então o termo
 2PI não pode existir, isto é, P=0 ou I=0. Assim, p(x) é par ou ímpar. E
 x^2-x+1 não é um nem outro, então não funcionou...

 Então precisamos ainda mostrar que existe UM tal polinômio!

 Abraço,
   Ralph

 P.S.: Tem certeza que o enunciado é esse mesmo? Não seria, sei lá,
 p(x^2+1)=(p(x))^2+1 ao invés?
 2011/7/1 Johann Dirichlet peterdirich...@gmail.com

 Em 01/07/11, Johann Dirichletpeterdirich...@gmail.com escreveu:
  Em 30/06/11, marcone augusto araújo
  borgesmarconeborge...@hotmail.com escreveu:
 
  1) Se p é inteiro primo ímpar,mostre que o numerador da fração
  1+1/2+1/3+...1/(p-1) é um múltiplo de p.
 
  1) Teorema de Wolstenholme, se não me engano...
 
  Bora lá, usar o velho truque das pontas de Gauss:
  1/k+1/(p-k)=p/(k(p-k));
  assim sendo, temos um monte de frações p/(alguma coisa). Esta coisa
  não será múltipla de p em momento nenhum, logo nada aniquila este
  fator p.
 
 
  2) Mostre que existem infinitos polinômios p(x) com coeficientes reais
  tais
  que p(x^2+1) = [p(x)]^2.

 É mais mole do que eu pensei!

 1 - Se P e Q são soluções da equação acima, P*Q também será. Óbvio!
 2 - Um polinômio possível é x^2-x+1.
 Como sei? Simples:

 Se L é um zero de P, então L^2+1 também será.
 Se eu conseguir L=L^2+1, terei uma solução pronta!
 Basta abrir o polinomio sem medo.


 P.S.: saber todas as soluções me parece mais desgastante. Aplicando a
 transformação T(L)=L^2+1 um numero finito de vezes, todos os
 polinômios dos pontos fixos são soluções. A treta é saber se não
 escapa nenhum (até porque muitos desses polinomios são fatoráveis, I
 think so).

 
  3) Uma corda AB,de comprimento constante,desliza sobre uma
  semicircunferência determinada por um diâmetro d.
  Considere o triângulo cujos vértices são: o ponto médio da corda e as
  projeções ortogonais dos

Re: [obm-l] Problemas(ajuda)

2011-07-01 Por tôpico Johann Dirichlet
Em 30/06/11, marcone augusto araújo
borgesmarconeborge...@hotmail.com escreveu:

 1) Se p é inteiro primo ímpar,mostre que o numerador da fração
 1+1/2+1/3+...1/(p-1) é um múltiplo de p.

1) Teorema de Wolstenholme, se não me engano...

Bora lá, usar o velho truque das pontas de Gauss: 1/k+1/(p-k)=p/(k(p-k));
assim sendo, temos um monte de frações p/(alguma coisa). Esta coisa
não será múltipla de p em momento nenhum, logo nada aniquila este
fator p.


 2) Mostre que existem infinitos polinômios p(x) com coeficientes reais tais
 que p(x^2+1) = [p(x)]^2.

 3) Uma corda AB,de comprimento constante,desliza sobre uma
 semicircunferência determinada por um diâmetro d.
 Considere o triângulo cujos vértices são: o ponto médio da corda e as
 projeções ortogonais dos seus extremos A e B
 sobre o diâmetro d.Mostre que ,durante o deslizamento da corda,esse
 triângulo é sempre isósceles e nunca muda de formato(i.é.,os ângulos do
 triângulo são constantes)

Faz um desenho!
Diâmetro r;centro O, raio 1; corda AB, tamanho d, médio M; AB
projetado em r dá XY.

O triangulo AOB é obviamente isósceles.
Os quadrilateros XOMA e YOMB são inscritíveis de diâmetros OA e OB
respectivamente (angulos de 90 graus).

Temos OXM=OAM=OBM=OYM, logo XMY é isosceles. E o angulo OBA depende
unicamente de d.

P.S.: duvido que os triangulos sejam todos congruentes. O angulo XOM
define o tamanho de XM.


 Meus agradecimentos por qualquer esclarecimento.  
 


-- 
/**/
神が祝福

Torres

=
Instru��es para entrar na lista, sair da lista e usar a lista em
http://www.mat.puc-rio.br/~obmlistas/obm-l.html
=


Re: [obm-l] Problemas(ajuda)

2011-07-01 Por tôpico Johann Dirichlet
Em 01/07/11, Johann Dirichletpeterdirich...@gmail.com escreveu:
 Em 30/06/11, marcone augusto araújo
 borgesmarconeborge...@hotmail.com escreveu:

 1) Se p é inteiro primo ímpar,mostre que o numerador da fração
 1+1/2+1/3+...1/(p-1) é um múltiplo de p.

 1) Teorema de Wolstenholme, se não me engano...

 Bora lá, usar o velho truque das pontas de Gauss: 1/k+1/(p-k)=p/(k(p-k));
 assim sendo, temos um monte de frações p/(alguma coisa). Esta coisa
 não será múltipla de p em momento nenhum, logo nada aniquila este
 fator p.


 2) Mostre que existem infinitos polinômios p(x) com coeficientes reais
 tais
 que p(x^2+1) = [p(x)]^2.

É mais mole do que eu pensei!

1 - Se P e Q são soluções da equação acima, P*Q também será. Óbvio!
2 - Um polinômio possível é x^2-x+1.
Como sei? Simples:

Se L é um zero de P, então L^2+1 também será.
Se eu conseguir L=L^2+1, terei uma solução pronta!
Basta abrir o polinomio sem medo.


P.S.: saber todas as soluções me parece mais desgastante. Aplicando a
transformação T(L)=L^2+1 um numero finito de vezes, todos os
polinômios dos pontos fixos são soluções. A treta é saber se não
escapa nenhum (até porque muitos desses polinomios são fatoráveis, I
think so).


 3) Uma corda AB,de comprimento constante,desliza sobre uma
 semicircunferência determinada por um diâmetro d.
 Considere o triângulo cujos vértices são: o ponto médio da corda e as
 projeções ortogonais dos seus extremos A e B
 sobre o diâmetro d.Mostre que ,durante o deslizamento da corda,esse
 triângulo é sempre isósceles e nunca muda de formato(i.é.,os ângulos do
 triângulo são constantes)

 Faz um desenho!
 Diâmetro r;centro O, raio 1; corda AB, tamanho d, médio M; AB
 projetado em r dá XY.

 O triangulo AOB é obviamente isósceles.
 Os quadrilateros XOMA e YOMB são inscritíveis de diâmetros OA e OB
 respectivamente (angulos de 90 graus).

 Temos OXM=OAM=OBM=OYM, logo XMY é isosceles. E o angulo OBA depende
 unicamente de d.

 P.S.: duvido que os triangulos sejam todos congruentes. O angulo XOM
 define o tamanho de XM.


 Meus agradecimentos por qualquer esclarecimento. 
 


 --
 /**/
 神が祝福

 Torres



-- 
/**/
神が祝福

Torres

=
Instru��es para entrar na lista, sair da lista e usar a lista em
http://www.mat.puc-rio.br/~obmlistas/obm-l.html
=


[obm-l] Re: [obm-l] Re: [obm-l] questão trigonometria complicada

2011-06-28 Por tôpico Johann Dirichlet
Este foi um problema da revista Kvant, na verdade um artigo.

Eis o site (pra quem encarar um russinho básico...)
http://kvant.mccme.ru/


Em 27/06/11, Ralph Teixeiraralp...@gmail.com escreveu:
 Ah, já vi errinho de sinal no meio do caminho, no sinal de ab+ac+bc e no de
 abc. Corrigi abaixo, mas deve haver outros. De qualquer forma, a ideia ainda
 vale.
 2011/6/27 Ralph Teixeira ralp...@gmail.com

 Hmmm, vejamos. Será que a gente arruma algum polinômio cujas raízes sejam
 as 3 parcelas da sua soma?

 Considere a famosa identidade
 trigonométrica sin7t=(8(cos2t)^3+4(cos2t)^2-4(cos2t)-1).sint

 (Desculpa, não pude resistir.)

 Note que t=kpi/7 (k=1,2,4) dá três raízes de sin7t, mas nenhum deles dá
 raiz de sint. Então estes valores de t devem anular o termo entre
 parênteses... Em outras palavras, se você considerar o polinômio
 P(x)=8x^3+4x^2-4x-1, você verá que suas raízes são exatamente cos(2pi/7),
 cos(4pi/7) e cos(8pi/7) -- exatamente porque é um polinômio do 3o grau,
 então se eu achei 3 raízes distintas, achei todas.

 (O argumento também vale para k=3,5,6, mas então obtemos
 cos(6pi/7)=cos(8pi/7), cos(10pi/7)=cos(4pi/7) e cos(12pi/7)=cos(2pi/7),
 que
 são aquelas raízes de novo)

 Em suma, o problema agora é: sejam a,b e c as raízes de
 P(x)=8x^3+4x^2-4x-1. Encontre a^(1/3)+b^(1/3)+c^(1/3).

 Vou escrever a^(1/3)=A, b^(1/3)=B e c^(1/3)=C. Mas, do polinômio sabemos
 que
 a+b+c=-1/2, isto é, A^3+B^3+C^3=-1/2
 ab+ac+bc=-1/2, isto é, A^3B^3+A^3C^3+B^3C^3=-1/2
 abc=1/8, isto é, ABC=1/2.

 Poxa, eu até consigo fazer o resto, mas é HORRENDO. Vamos lá.

 Agora, talvez você já tenha visto a identidade
 x^3+y^3+z^3-3xyz=(x+y+z)((x+y+z)^2-3(xy+xz+yz))

 Aplicando esta identidade com (x,y,z)=(A,B,C) temos:
 -1/2-3/2=-2=S(S^2-3D) (onde S=A+B+C e D=AB+AC+BC)

 Aplicando esta identidade com (x,y,z)=(AB,AC,BC), temos:
 -1/2-3(1/4)=-5/4=D(D^2-3SP)=D(D^2-3S/2) (onde P=ABC=1/2)

 Enfim, duas equações e duas incógnitas! Tire D da primeira e jogue na
 segunda -- fica horrendo, mas dá uma equação polinomial de grau 9 em S,
 com
 termos apenas em S^3, S^6 e S^9. Faça S^3=Z, resolva a equação cúbica em
 Z,
 S é a raiz cúbica de Z.

 Argh! Tá, fiquei sem vontade de terminar as contas, e devo ter errado algo
 no meio do caminho, mas saiu!

 Abraço,
Ralph


 2011/6/26 Jefferson Franca jeffma...@yahoo.com.br

 Boa tarde senhores. Será que alguém poderia me iluminar nesta questão:
 Calcule o valor da soma (cos(2*pi/7)^1/3 + (cos(4*pi/7))^1/3 +
 (cos(8*pi/7))^1/3 ?
 abs






-- 
/**/
神が祝福

Torres

=
Instru��es para entrar na lista, sair da lista e usar a lista em
http://www.mat.puc-rio.br/~obmlistas/obm-l.html
=


Re: [obm-l] Desigualdade (Como provar?)

2011-06-21 Por tôpico Johann Dirichlet
Você,Você,Você,Você,Você,Você,Você quer uma demo por PIF?
Bem, vou te dar a dica: prove de n para 2n, e depois de n para n-1.
Em 21/06/11, Carlos Nehabne...@infolink.com.br escreveu: Oi, Paulo. É 
simples e clássico. Basta usar média aritmética = média geométrica em S e 
S'. Abraços Nehab Em 21/6/2011 08:34, Paulo Argolo escreveu: Caros 
Colegas, Não consegui ainda uma demonstração. Seria possível fazê-la por 
indução finita? Abraços do Paulo. 
-
 Date: Mon, 13 Jun 2011 22:49:41 +0200 Subject: Re: [obm-l] Desigualdade 
(Como provar?) From: bernardo...@gmail.com To: obm-l@mat.puc-rio.br 
2011/6/13 Paulo Argoloargolopa...@hotmail.com: Caros Colegas, Como 
podemos provar que, dados n numeros reais positivos (n1), nem todos 
iguais, vale a desigualdade abaixo? S . S'  n^2 (S é a soma dos n 
números, S' é a soma dos inversos desses n números.) Tente mostrar 
isso para n = 2, n = 3, expandindo tudo. Dá poucos termos, e daí acho qu!
e você vai ver como prova para n qualquer. Abraços, -- Bernardo 
Freitas Paulo da Costa 
= 
Instruções para entrar na lista, sair da lista e usar a lista em 
http://www.mat.puc-rio.br/~obmlistas/obm-l.html 
=

= 
Instruções para entrar na lista, sair da lista e usar a lista em 
http://www.mat.puc-rio.br/~obmlistas/obm-l.html 
= 
= 
Instruções para entrar na lista, sair da lista e usar a lista em 
http://www.mat.puc-rio.br/~obmlistas/obm-l.html 
=

-- /**/神が祝福
Torres
=
Instru��es para entrar na lista, sair da lista e usar a lista em
http://www.mat.puc-rio.br/~obmlistas/obm-l.html
=


[obm-l] Re: [obm-l] Re: [obm-l] Re: [obm-l] Re: [obm-l] FW: Teoria dos números

2011-05-30 Por tôpico Johann Dirichlet
O que eu posso fazer se eu sou lento e preguiçoso no computador?

Ah, deixa eu ficar ninja no dvorak... huahuahuahua!

Em 27/05/11, Rogerio Ponceabrlw...@gmail.com escreveu:
 Pois e', Dirichlet, o Ralph tem este pessimo habito...
 :)

 []'s
 Rogerio Ponce

 Em 27 de maio de 2011 17:39, Ralph Teixeira ralp...@gmail.com escreveu:

 Yeah! Ninjei de novo! :) :) :) ;)
 2011/5/27 Johann Dirichlet peterdirich...@gmail.com

 Poxa! O Ralph destruiu minha mensagem! Mas acabei respondendo do mesmo
 jeito (ou nao!:))

 Em 27/05/11, Johann Dirichletpeterdirich...@gmail.com escreveu:
  Ce já estudou congruencias? Um bom começo é pegar a Eureka! 2 na
  página da OBM, www.obm.org.br (ou comprar da OBM! É baratinho, uma
  anuidade de uns 30 reais e uns 4 contos por cada atrasado que quiser).
  Anyway, vou tentar deixar fácil...
 
  1)
  2^n=(x-1)(x^2+x+1)
 
  Vamos tentar calcular o MDC:
  d|x-1
  d|x^2+x+1
 
  x =1 (mod d)
  x^2+x+1=0 (mod d)
 
  primeira na segunda, d|3. Como d=3 é impossível (potencias de 2 nao
  tem fatores 3 :) ), d=1.
 
  Em especial, x-1=1 ou x^2+x+1=1 (ambos sao potencias de 2, e o MDC é
  1, logo um deles é 1).
  Ou seja, x=2 ou 0. Substitui e chora!
 
  2)
  7|4n^2-3
  Multiplica por 2
  7|8n^2-6=n^2+1+(7n^2-7)
 
  7|n^2+1
  Por congruências, é possível provar que basta testar n de 0 a 6.Mas
  vou usar descenso infinito.
 
  Teste de 0 a 6 (larga a mão de ser preguiçoso!). Vai falhar (eu acho :)
 ).
 
  Se funcionar para algum cara maior que 6, seja F o menor dos caras
  para os quais funciona (se existe, existe o menor, este é o lema da
  boa ordem).
 
  Seja J=F-7. Então J é maior ou igual a 0.
  7|(J+7)^2+1=J^2+2*7*J+7^2+1=7*(um termo chato que não interessa)+J^2+1
  7|J^2+1
 
  Mas epa! Achei um cara (J) menor que o menor(F)!
  E este é um absurdo, que surgiu quando eu disse que funcionava para
  algum cara maior que 6!
  Então, só faltaria testar para caras menores que 7. Você já testou,
  então sabe que não funciona!
 
  É isso.
 
  P.S.: otruque de multiplicar por 2 facilita a vida pacas, mas não
  precisava aplica-lo: a ideia do descenso infinito ainda daria conta.
 
 
  Em 27/05/11, marcone augusto araújo
  borgesmarconeborge...@hotmail.com escreveu:
 
 
 
 
 
  From: marconeborge...@hotmail.com
  To: obm-l@mat.puc-rio.br
  Subject: Teoria dos números
  Date: Fri, 27 May 2011 12:28:34 +
 
 
 
 
   1) Mostrar que para nenhum número natural n ,( 2^n)+1 nunca é
 um
  cubo.
 
   Pensei:2^n=x^3-1=(x-1)(x^2+x+1).Se eu conseguisse mostrar q
  mdc((x-1,x^2+x+1)=1 e que x-1 e
 x^2+x+1 não podem ser cubos ao mesmo tempo,acredito q
  resolveria
 a
  questão.
   Tentei outras formas também ,mas não consegui.
 
   2) Provar q não exiiste número natural n tal q 7 divide
 4n^2-3.
 
Considerei n= 7k+ 1 ou 7k-1 ou 7k+2 ou 7k-2 ou 7k+3 ou 7k-3
  e
  verifiquei q 4n^2-3 não é múltiplo de 7.
Sei q há outras formas(e talvez mais interessantes).
 
 
 
  --
  /**/
  神が祝福
 
  Torres
 


 --
 /**/
 神が祝福

 Torres

 =
 Instru�ões para entrar na lista, sair da lista e usar a lista em
 http://www.mat.puc-rio.br/~obmlistas/obm-l.html
 =






-- 
/**/
神が祝福

Torres

=
Instru��es para entrar na lista, sair da lista e usar a lista em
http://www.mat.puc-rio.br/~obmlistas/obm-l.html
=


[obm-l] Re: [obm-l] FW: Teoria dos números

2011-05-27 Por tôpico Johann Dirichlet
Ce já estudou congruencias? Um bom começo é pegar a Eureka! 2 na
página da OBM, www.obm.org.br (ou comprar da OBM! É baratinho, uma
anuidade de uns 30 reais e uns 4 contos por cada atrasado que quiser).
Anyway, vou tentar deixar fácil...

1)
2^n=(x-1)(x^2+x+1)

Vamos tentar calcular o MDC:
d|x-1
d|x^2+x+1

x =1 (mod d)
x^2+x+1=0 (mod d)

primeira na segunda, d|3. Como d=3 é impossível (potencias de 2 nao
tem fatores 3 :) ), d=1.

Em especial, x-1=1 ou x^2+x+1=1 (ambos sao potencias de 2, e o MDC é
1, logo um deles é 1).
Ou seja, x=2 ou 0. Substitui e chora!

2)
7|4n^2-3
Multiplica por 2
7|8n^2-6=n^2+1+(7n^2-7)

7|n^2+1
Por congruências, é possível provar que basta testar n de 0 a 6.Mas
vou usar descenso infinito.

Teste de 0 a 6 (larga a mão de ser preguiçoso!). Vai falhar (eu acho :) ).

Se funcionar para algum cara maior que 6, seja F o menor dos caras
para os quais funciona (se existe, existe o menor, este é o lema da
boa ordem).

Seja J=F-7. Então J é maior ou igual a 0.
7|(J+7)^2+1=J^2+2*7*J+7^2+1=7*(um termo chato que não interessa)+J^2+1
7|J^2+1

Mas epa! Achei um cara (J) menor que o menor(F)!
E este é um absurdo, que surgiu quando eu disse que funcionava para
algum cara maior que 6!
Então, só faltaria testar para caras menores que 7. Você já testou,
então sabe que não funciona!

É isso.

P.S.: otruque de multiplicar por 2 facilita a vida pacas, mas não
precisava aplica-lo: a ideia do descenso infinito ainda daria conta.


Em 27/05/11, marcone augusto araújo
borgesmarconeborge...@hotmail.com escreveu:





 From: marconeborge...@hotmail.com
 To: obm-l@mat.puc-rio.br
 Subject: Teoria dos números
 Date: Fri, 27 May 2011 12:28:34 +




  1) Mostrar que para nenhum número natural n ,( 2^n)+1 nunca é um
 cubo.

  Pensei:2^n=x^3-1=(x-1)(x^2+x+1).Se eu conseguisse mostrar q
 mdc((x-1,x^2+x+1)=1 e que x-1 e
x^2+x+1 não podem ser cubos ao mesmo tempo,acredito q resolveria a
 questão.
  Tentei outras formas também ,mas não consegui.

  2) Provar q não exiiste número natural n tal q 7 divide 4n^2-3.

   Considerei n= 7k+ 1 ou 7k-1 ou 7k+2 ou 7k-2 ou 7k+3 ou 7k-3 e
 verifiquei q 4n^2-3 não é múltiplo de 7.
   Sei q há outras formas(e talvez mais interessantes).
   


-- 
/**/
神が祝福

Torres

=
Instru��es para entrar na lista, sair da lista e usar a lista em
http://www.mat.puc-rio.br/~obmlistas/obm-l.html
=


[obm-l] Re: [obm-l] FW: Teoria dos números

2011-05-27 Por tôpico Johann Dirichlet
Poxa! O Ralph destruiu minha mensagem! Mas acabei respondendo do mesmo
jeito (ou nao!:))

Em 27/05/11, Johann Dirichletpeterdirich...@gmail.com escreveu:
 Ce já estudou congruencias? Um bom começo é pegar a Eureka! 2 na
 página da OBM, www.obm.org.br (ou comprar da OBM! É baratinho, uma
 anuidade de uns 30 reais e uns 4 contos por cada atrasado que quiser).
 Anyway, vou tentar deixar fácil...

 1)
 2^n=(x-1)(x^2+x+1)

 Vamos tentar calcular o MDC:
 d|x-1
 d|x^2+x+1

 x =1 (mod d)
 x^2+x+1=0 (mod d)

 primeira na segunda, d|3. Como d=3 é impossível (potencias de 2 nao
 tem fatores 3 :) ), d=1.

 Em especial, x-1=1 ou x^2+x+1=1 (ambos sao potencias de 2, e o MDC é
 1, logo um deles é 1).
 Ou seja, x=2 ou 0. Substitui e chora!

 2)
 7|4n^2-3
 Multiplica por 2
 7|8n^2-6=n^2+1+(7n^2-7)

 7|n^2+1
 Por congruências, é possível provar que basta testar n de 0 a 6.Mas
 vou usar descenso infinito.

 Teste de 0 a 6 (larga a mão de ser preguiçoso!). Vai falhar (eu acho :) ).

 Se funcionar para algum cara maior que 6, seja F o menor dos caras
 para os quais funciona (se existe, existe o menor, este é o lema da
 boa ordem).

 Seja J=F-7. Então J é maior ou igual a 0.
 7|(J+7)^2+1=J^2+2*7*J+7^2+1=7*(um termo chato que não interessa)+J^2+1
 7|J^2+1

 Mas epa! Achei um cara (J) menor que o menor(F)!
 E este é um absurdo, que surgiu quando eu disse que funcionava para
 algum cara maior que 6!
 Então, só faltaria testar para caras menores que 7. Você já testou,
 então sabe que não funciona!

 É isso.

 P.S.: otruque de multiplicar por 2 facilita a vida pacas, mas não
 precisava aplica-lo: a ideia do descenso infinito ainda daria conta.


 Em 27/05/11, marcone augusto araújo
 borgesmarconeborge...@hotmail.com escreveu:





 From: marconeborge...@hotmail.com
 To: obm-l@mat.puc-rio.br
 Subject: Teoria dos números
 Date: Fri, 27 May 2011 12:28:34 +




  1) Mostrar que para nenhum número natural n ,( 2^n)+1 nunca é um
 cubo.

  Pensei:2^n=x^3-1=(x-1)(x^2+x+1).Se eu conseguisse mostrar q
 mdc((x-1,x^2+x+1)=1 e que x-1 e
x^2+x+1 não podem ser cubos ao mesmo tempo,acredito q resolveria a
 questão.
  Tentei outras formas também ,mas não consegui.

  2) Provar q não exiiste número natural n tal q 7 divide 4n^2-3.

   Considerei n= 7k+ 1 ou 7k-1 ou 7k+2 ou 7k-2 ou 7k+3 ou 7k-3 e
 verifiquei q 4n^2-3 não é múltiplo de 7.
   Sei q há outras formas(e talvez mais interessantes).
  


 --
 /**/
 神が祝福

 Torres



-- 
/**/
神が祝福

Torres

=
Instru��es para entrar na lista, sair da lista e usar a lista em
http://www.mat.puc-rio.br/~obmlistas/obm-l.html
=


[obm-l] Re: [obm-l] Re: [obm-l] número primo e soma de quadrados

2011-05-17 Por tôpico Johann Dirichlet
Você encontrará umas três demonstrações bem legais no livro Proofs
from THE BOOK, Martin Aigner e Günter M. Ziegler.



Em 16/05/11, Tiagohit0...@gmail.com escreveu:
 Existem diversas maneiras de demonstrar isso. Algumas delas usando ideias e
 áreas da matemática bem diferentes.

 http://en.wikipedia.org/wiki/Proofs_of_Fermat%27s_theorem_on_sums_of_two_squares

 2011/5/16 marcone augusto araújo borges marconeborge...@hotmail.com

  Todo número primo da forma 4k+1pode ser escrito de uma única maneira como
 soma de dois quadrados.Como demonstrar?




 --
 Tiago J. Fonseca
 http://legauss.blogspot.com



-- 
/**/
神が祝福

Torres

=
Instru��es para entrar na lista, sair da lista e usar a lista em
http://www.mat.puc-rio.br/~obmlistas/obm-l.html
=


Re: [obm-l] problem numero 15 eureka numero 5

2011-05-17 Por tôpico Johann Dirichlet
1 - Enunciado completo,please!

Vou tentar reescrever para deixar mais claro:

Em um conjunto de MN+1 inteiros positivos, postos em ordem crescente,
uma das duas situações abaixo ocorrerá:

-- haverá uma subsequencia de M+1 inteiros, tais nenhum deles é
divisor de algum outro;
--haverá uma subsequencia de N+1 inteiros, tal que cada termo da
subsequencia será divisor do seguinte;


Um exemplo:
Em um conjunto de 16=3*5+1 elementos,
ou há 3+1=4 inteiros tais que nenhum divide outros,
ou há 5+1=6 tais que cada um divide o seguinte.


Acho que você pensou algo como 5 divide 5+1, o que é obviamente falso ;-P

Em 16/05/11, maurikleber araujomaurikle...@hotmail.com escreveu:

 pessoal alguem ai poderia explicar o problema numero 15 (principio das
 gavetas) da eureka numero 5
 onde ele diz n +1 numeros divide o seguinte ele quer dizer que divide o
 sucessor ou qualquer numero depois
 devo estar entendendo errado porque se for divide o sucessor nao tem como
 provar ou ao menos penso ter achado um contra exemplo
 alguem pode explicar sem resolver a questao   


-- 
/**/
神が祝福

Torres

=
Instru��es para entrar na lista, sair da lista e usar a lista em
http://www.mat.puc-rio.br/~obmlistas/obm-l.html
=


Re: [obm-l] Teorema sobre mediana (OFFTOPIC)

2011-05-13 Por tôpico Johann Dirichlet
Em 13/05/11, Ralph Teixeiraralp...@gmail.com escreveu:
 Isso é legal, né?

 -- A média minimiza a soma dos quadrados dos desvios
 -- A mediana minimiza a soma dos módulos dos desvios.

 Olhando deste jeito, a mediana parece mais natural do que média para
 resumir os dados de uma sequência alíás, vocês já pararam para
 pensar PORQUE a gente usa a média o tempo todo? No fundo no fundo, é
 só por costume, não há uma razão matemática muito forte não... vou
 exagerar um pouco: se eu pudesse, usava a mediana para calcular as
 notas finais dos meus alunos.


A ideia é que a média é um balanço entre perdas e ganhos.
Por exemplo, a média entre 5 e 7 é 6, e 6 perde 1 de 7 mas ganha 1 de 5.
O problema é que a média é muito sensível a variações dos dados.

Antes de eu fugir da facul, já tive professores que usavam média
harmônica para as provas!
Zerou uma prova, já era! Ou tão pior quanto, ao se tirar vários 10, um
5 te jogava pra baixo.

Mas enfim, acho que a média é mais usada porque embute a ideia que eu
exibi acima.

-- 
/**/
神が祝福

Torres

=
Instru��es para entrar na lista, sair da lista e usar a lista em
http://www.mat.puc-rio.br/~obmlistas/obm-l.html
=


Re: [obm-l] Geometria Cone Sul

2011-05-13 Por tôpico Johann Dirichlet
Em 12/05/11, Luís Lopesqed_te...@hotmail.com escreveu:

 Sauda,c~oes,

 Fonte: Treinamento Cone Sul Volume 2.

 Problema 26 p. 135

 H_b , H_c pés das alturas de B e C.
 H ortocentro
 M_a médio de BC
 Gamma Circuncírculo de ABC
 phi Circuncírculo de AH_bH_c
 S segunda interseção de phi com Gamma

 Mostre que S, H, M_a são colineares

Opa! É Geometria Projetiva na cabeça!

Pegue em mãos sua Eureka! 8, dê uma estudada no artigo do Luciano, e
mãos à obra!
Para melhor acompanhar a demonstração, pegue um papel e faça um
desenho caprichado!

Vamos antes tomar um leminha:

Triângulo ABC;
Alturas AH_a, BH_b, CH_c; ortocentro H
Médios M_a,M_b,M_c;
X_a é o ponto comum a BC e H_bH_c

Demonstrar que M_aH e AX_a são perpendiculares!

Vamlá:
Tome o circulo de centro M_a e raio BM_a, chame-o Epsilon. Como BH_bC
e CH_cB são angulos retos, esta circunferencia passa pelos pontos H_b
e H_c.
Vamos dualizar em Epsilon agora.

A reta polar de A é HX_a
A reta polar de X_a é AH.

Portanto, o ponto polar (polo) de AX_a é H.
Logo, M_aH é perpendicular a AX_a.

Seja Y o ponto comum a AX_a e M_aH.
Pelo lema, AYH=90 graus.

Agora, potência de ponto!

AH_bHH_c é cíclico (angulos retos);
CH_bH_cB é cíclico (angulos retos);
ABC é cíclico (triangulo).

X_a é centro radical dos tres circulos acima descritos (é meio óbvio,
mas basta calcular as potencias de ponto em relação a cada par de
circulos).

Seja Z o ponto em que AX_a corta o circulo AH_bHH_c. Por potencia de
ponto, A,Z,B,C são concíclicos, bem como A, Z, H_c, H.
Logo angulo AZH = angulo AH_cH=90 graus = angulo AYH. Portanto, Y=Z, e
tá demonstrado!


 Como fazer? Com geometria sintética de preferência.

Pra variar um pouco, não vou fazer com trigonometria. Mas a ideia é simples:
Demonstrar que M_aH e AX_a são perpendiculares!


 []'s
 Luís

   


-- 
/**/
神が祝福

Torres

=
Instru��es para entrar na lista, sair da lista e usar a lista em
http://www.mat.puc-rio.br/~obmlistas/obm-l.html
=


Re: [obm-l] produto interno

2011-05-11 Por tôpico Johann Dirichlet
Em 06/05/11, Samuel Wainersswai...@hotmail.com escreveu:

 qual a diferença entre produto hermetiano e produto interno?sempre ouvi
 falar em operador hermetiano, não em produto hermetiano. Eles são a mesma
 coisa?

 Para toda matriz simétrica A, existe uma matriz invertível P tq: A = (P^-1)
 D (P) onde D é diagonal.?Usa isso num teorema que estou lendo, mas é fato?

Bem, é um fato demonstrável; é fácil achar em bons livros de álgebra linear.

Normalmente ele vem depois de uma série de teoremas. Mas o que você
precisa saber para pesquisar é o seguinte:

0 - Duas matrizes A,B são ditas semelhantes se A=P*B*(P^-1) em que P é
uma matriz conveniente.

1 - Uma matriz quadrada A é dita diagonalizável se é semelhante a uma
matriz diagonal.

Teorema: toda matriz simétrica é diagonalizável!

Entre o 1 e o teorema, se costuma apelar para umas teorias fáceis de
aprender e um pouco difíceis de testar ;-P

Topa uma leitura online?
http://joshua.smcvt.edu/linearalgebra/

100% full, disponível para download.

E um review
http://www.randomhacks.net/articles/2007/03/07/hefferon-linear-algebra-review

-- 
/**/
神が祝福

Torres

=
Instru��es para entrar na lista, sair da lista e usar a lista em
http://www.mat.puc-rio.br/~obmlistas/obm-l.html
=


Re: [obm-l] Descobrir formula geral e provar f(n+1)=2f(n) +3

2011-05-09 Por tôpico Johann Dirichlet
Outra maneira:
f(0)=0
f(n+1)=2f(n) +3

Vendo que f(n+1) é quase o dobro de f(n), uma ideia seria obter uma PG.

f(n+1)+C=2f(n) +3+C= 2(f(n)+(C+3)/2)
Se C=(C+3)/2, ou C=3, obtemos uma relacao interessante:

f(n+1)+3=2(f(n)+3).
E isto é uma PG!
O resto segue acima.

Em 06/05/11, Julio Teixeirajcesarp...@gmail.com escreveu:
 Pessoal, a um tempo acho que vi essa questao aki  e por acaso, ontem me
 deparei com ela em alguns foruns, e o pessoal estava com dificuldades..entao
 vou por aki a minha resolucao..

 questao 157 do Vol. 1 da colecao do G. Iezzi - Fundamentos de matematica
 elemtentar

 157 - Seja f uma funcao, definida no conjunto dos numeros naturais, tal que,
 f(n+1)=2f(n) +3
 com f(0) = 0. Achar a formula geral de f(n) e prova-la por inducao..

 equacao: f(n+1)=2f(n) + 3  e f(0)=0

 para..
 n=0 = f(0+1)=2f(0)+3 = f(1)=3
 n=1 = f(1+1)=2f(1)+3 = f(2)=9
 n=2 = f(2+1)=2f(2)+3 = f(3)=21
 n=3 = f(3+1)=2f(3)+3 = f(4)=45
 n=4 = f(4+1)=2f(4)+3 = f(5)=93

 observando os valores retornado pelas imagens e pondo em produto de um fator
 por 3..
 f(1)=3  = f(1)=3*1
 f(2)=9  = f(2)=3*3
 f(3)=21 = f(3)=3*7
 f(4)=45 = f(4)=3*15
 f(5)=93 = f(5)=3*31

 agora observando os segundos fatores dos produtos acima nas imagens...
 comecamos com 1, depois 3, depois 7, e

 assim temos:
 a diferenca entre  3 e 1 = 2
 a diferenca entre  7 e 3 = 4
 a diferenca entre 15 e 7 = 8
 a diferenca entre 31 e 15 = 16

 obrservando essas diferencas, nota-se que temos uma PG, de razao 2, e com o
 primeiro termo sendo igual a 1

 assim a formula ja comeca a ficar evidente.. sendo 3 vezes essas
 diferencas...

 agora se montarmos essa PG, teremos..

 a1 = 1
 a2 = 2
 a3 = 4
 a4 = 8
 a5 = 16

 opa.. entao a proxima observacao a ser feita eh que, com os resultados
 obtidos temos que,por exemplo,
 f(1)=3*( a1 de nossa PG)
 f(2)=3*( a soma de a1 com o a2 de nossa PG)
 f(3)=3*( a soma de a1 com o a2  e a3 de nossa PG)
 f(4)=3*( a soma de a1 com o a2  e a3 e a4 de nossa PG)

 agora a formula do somatorio de nossa PG seria:
 Sn = a1 * (q^n - 1)/ (q - 1)

 onde substituindo, obteriamos:
 2^n -1

 agora deduzimos entao que a formula geral seria: f(n)= 3 * ( 2^n - 1)

 para provarmos por inducao, vamos provar que eh valido para n=1
 f(1) = 3 * ( 2^1 -1)
 f(1) = 3 * ( 1 ) = f(1) = 3   ( OK, provamos para n=1 )

 agora substituimos por n, por um k, qualquer e obtemos:
 f(k)= 3 * (2^k -1)

 agora substituimos por k+1
 f(k+1)= 3 * (2^(k+1) -1)

 ok, agora note que se pegarmos a formula inicial e aplicarmos n=k, obteremos
 o seguinte..
 f(k+1)=2 * f(k) + 3

 ja que obtemos f(k+1) de nossa formula e f(k+1) da formula original, para
 provarmos que descobrimos a formula geral
 entao o resultado de f(k+1), tem que ser igual, assim tb testamos se eh
 valida para qualquer elemento, provando isso para qualquer sucessor de k, ou
 seja (k+1)
 entao temos o seguinte..
 f(k)= 3 * (2^k -1)
 f(k+1)= 3 * (2^(k+1) -1)
 f(k+1)=2 * f(k) + 3

 agora igualando os f(k+1), obtemos..
 2 * f(k) + 3 = 3 * (2^(k+1) -1)
 substituindo f(k), pelo valor conhecido tb.. ( da nossa formula geral )
 2 * (3 * (2^k -1)) + 3 = 3 * (2^(k+1) -1)
 6 * (2^k -1) + 3 = 3 * (2^(k+1)) -3
 agora, dividimos amobs os lados por 3
 2 * (2^k -1) + 1 = 2^(k+1) - 1
 2^(k+1) -2 + 1 = 2^(k+1) - 1
 2^(k+1) - 1 = 2^(k+1) - 1(OK)
 obtemos assim, a nossa prova...



-- 
/**/
神が祝福

Torres

=
Instru��es para entrar na lista, sair da lista e usar a lista em
http://www.mat.puc-rio.br/~obmlistas/obm-l.html
=


[obm-l] Re: [obm-l] Re: [obm-l] Olimpíada Universitária.

2011-05-06 Por tôpico Johann Dirichlet
Bem, a dica de estudo é a mesma. Eu sugiro que você pegue as
olimpiadas internacionais também (a IMC é muito legal! e serve bem pra
estudar a OBM universitária).

Na verdade, acho que nem mesmo restrição de graduação deve ter. Lembro
de um aluno que tinha um ano a mais de graduação (por ter quebrado a
grade em porções mais digeríveis) e pôde participar. De todo modo, a
Nelly deve saber :-).

Em 25/04/11, Tiagohit0...@gmail.com escreveu:
 Olá, acho que não tem limite de idade, só de anos de graduação.

 Nas olimpíadas universitárias, a teoria que você tem que saber do ensino
 superior são basicamente Cálculo I e Álgebra linear. Mas o melhor jeito de
 estudar, imagino eu, é pegando as provas anteriores (quem sabe as dos níveis
 mais baixos também) e tentar entender as soluções (obviamente, tentando
 trabalhar nos problemas sozinho antes).

 Então o meu conselho é este, não tente seguir uma bibliografia, tente
 trabalhar nos problemas e, assim que certas assuntos surgirem e você
 perceber que não sabe muito bem a teoria, estude esse assunto.

 2011/4/25 Luís Junior jrcarped...@gmail.com

 Olá a todos,

 Gostaria de ouvir a opnião de vcs com relação a esse meu sonho/projeto.
 Sempre gostei de matemática mas frequentemente, na minha vida, um grande
 esforço se fez necessário para que eu alcançasse a média escolar. De fato,
 posso afirmar que sou um aluno abaixo da média e que 'rala' bastante para
 ser mediano. Ontem, tomei conhecimento das Olimpíadas Universitárias.
 Sempre
 tive esse sonho, de me preparar e participar de uma dessas Olimpíadas.
 Pois
 bem, tenho 30 anos e estou no primeiro semestre de um curso universitário
 regular. Procurei pelo regulamento para saber se há um limite de idade
 para
 os participantes mas não encontrei, então esta se torna a minha primeira
 dúvida. Sendo possível a minha participação, então se iniciaria um projeto
 de 5 anos (tempo médio da graduação) que contemplaria a minha preparação e
 participação no evento. Neste ponto, gostaria de saber a opnião de vcs
 sobre
 a possibilidade/dificuldade de empreender um projeto desses e como começar
 (Revisando o conteúdo de 2° grau?, seguindo uma bibliografia específica?,
 contratando um mestre?) visto que não tenho a mínima idéia.
 Agradeço pela atenção e peço desculpas pelo incômodo. Por favor participem
 com sua opnião!


 ~Carpe Diem~

 L.




 --
 Tiago J. Fonseca
 http://legauss.blogspot.com



-- 
/**/
神が祝福

Torres

=
Instru��es para entrar na lista, sair da lista e usar a lista em
http://www.mat.puc-rio.br/~obmlistas/obm-l.html
=


Re: [obm-l] Geometria

2011-05-05 Por tôpico Johann Dirichlet
[cuidado! resposta longa e chata detected!!]

Cara, esse tipo de problema eu sempre fiz do mesmo jeito:
trigonometria até enjoar!
Eu sempre preferi desta maneira, pois pra mim usar álgebra é mais
rápido que usar magia. Nem sempre estes truques são reaplicáveis, e
minha mente computeira se acostuma melhor a coisas que podem ser
guardadas para o futuro.

Mas, antes que falem mal, em hipótese nenhuma estou dizendo que
trigonometria é melhor, mas sim que eu tenho esta preferência por
resolver assim. E também concordo que geometria sintética é muito boa,
e que trigonometria não é superpoderosa (que o diga Erdos-Mordell!)

Como exemplo do que estou falando:
Tem um problema da Inglaterra, 1970, do mesmo estilo destes. Eu
resolvi bem rápido com trigonometria, mas tive que pedir ajuda ao
Naoki Sato, do MathLinks (na verdade ele foi o primeiro a responder),
para obter uma solução sintética (usando somente geometria e
construções auxiliares). Para todos os efeitos, ele reconstrói o
desenho usando uma sequência de lemas, e depois prova que este desenho
construído é igual ao do enunciado.

Vejam em
http://www.artofproblemsolving.com/Forum/viewtopic.php?f=47t=329268
e depois tirem suas conclusões...

P.S.: depois eu vou resolver este com trigonometria, haha!

Em 03/05/11, Marcelo Gomeselementos@gmail.com escreveu:
 Olá professor Albert,

 por favor, se for possível, poderia enviar sua solução também para mim ?

 Abraços, Marcelo.

 Em 28 de abril de 2011 18:04, Albert Bouskela bousk...@msn.com escreveu:

 Olá, Nehab e João,

 O trabalho da Silvana é mesmo bem legal, mas...

 Para resolver o problema proposto - o Nehab tem razão: é um dos mais
 clássicos - prefiro fazer um truque mais palatável: construir triângulos
 auxiliares. Estou enviando - através de um arquivo PDF - a solução para o
 e-mail de vocês.

 Sds.,
 Albert Bouskela
 bousk...@msn.com

  -Mensagem original-
  De: owner-ob...@mat.puc-rio.br [mailto:owner-ob...@mat.puc-rio.br] Em
  nome de Carlos Nehab
  Enviada em: 28 de abril de 2011 17:30
  Para: obm-l@mat.puc-rio.br
  Assunto: Re: [obm-l] Geometria
 
  Oi, João,
 
  O seu exercício é um clássico.
  Ai vai a dica. Um trabalho legal da Silvana: você vai gostar.
 
  http://www.mat.puc-rio.br/~hjbortol/complexidade/complexidade-em-
  geometria.pdf
 
  Capítulo 2 a partir da página 28
  Olhe também a página 36.
 
  Abraços,
  Nehab
 
  Em 26/4/2011 20:22, João Maldonado escreveu:
   O seguinte problema está no livro  Geometria I de Morgado, e não sei
  porque  não estou conseguindo resolvê-lo. Sei que a resposta é 30º, se
  alguém  puder ajudar fico grato.
  
   Em um triângulo isósceles ABC, se base BC, o ângulo  vale 20º. P é um
  ponto sobre AB tal que o ângulo PCB = 60º. Q é um ponto em AC tal que
  QBC
  = 50º. Qual a medida do ângulo CPQ?
   []'sJoão
 
  =
  
  Instruções para entrar na lista, sair da lista e usar a lista em
  http://www.mat.puc-rio.br/~obmlistas/obm-l.html
  =
  


 =
 Instruções para entrar na lista, sair da lista e usar a lista em
 http://www.mat.puc-rio.br/~obmlistas/obm-l.html
 =




-- 
/**/
神が祝福

Torres

=
Instru��es para entrar na lista, sair da lista e usar a lista em
http://www.mat.puc-rio.br/~obmlistas/obm-l.html
=


[obm-l] Re: [obm-l] Geometrias não Euclidianas - Como realizar as construções geométricas ?

2011-04-11 Por tôpico Johann Dirichlet
Bem, creio que não tenha muita lógica em falar de construçoes
euclidianas em espaços não-euclidianos. Mas, levando em conta os
postulados, dá pra brincar um pouco (só não espere algo com muito
sentido :) )

De todo modo, achei este site via Google:
http://cs.unm.edu/~joel/NonEuclid/NonEuclid.html

Em 09/04/11, Marcelo Gomeselementos@gmail.com escreveu:
 Caros amigos da lista,

 Estou impressionado coma  falta de material sobre as geometrias elíptica e
 hiperbólica. Estou tentando reproduzir construções geométricas que envolvam
 estas geometrias em softwares como o Geogebra e ReC.

 Gostaria de perguntar aos senhores se conhecem materiais que demonstrem como
 operar as construções (com régua e compasso) destas geometrias. Se alguém
 souber e quiser compartilhar agradeço muito.

 Abraços, Marcelo.



-- 
/**/
神が祝福

Torres

=
Instru��es para entrar na lista, sair da lista e usar a lista em
http://www.mat.puc-rio.br/~obmlistas/obm-l.html
=


[obm-l] Re: [obm-l] Re: [obm-l] Nº de funções sobrejetoras

2011-03-29 Por tôpico Johann Dirichlet
Cara, algo me diz que isso tá errado. Eu lembro de um artigo na antiga
RPM que contava, de um modo meio complicado, quantas funções existem.
Por exemplo, nesta fórmula (um somatório esquisito usando números de
Stirling), se |A|  |B|, tinha que dar 0.

Se pensarmos de B para A, cada elemento de B terá uma flechinha OU
MAIS chegando de elementos de A.

Posso estar errado, mas não dou certeza...

Em 28/03/11, Marcelo Salhab Brogliatomsbro...@gmail.com escreveu:
 Olá, Pedro,
 para cada elemento de B, temos que ter pelo menos um elemento de A que leve
 a ele.
 Logo, para o primeiro elemento de B, temos n opções.
 Para o segundo elemento de B, temos n-1 opções.
 E assim por diante.

 Assim, ficamos com:
 n*(n-1)*(n-2)*...*(n-m+1) = n! / (n-m)! = Arranjo(n, m) = A(n, m)

 Mas, ainda sobram n-m elementos em A.
 Para esses n-m elementos, tanto faz em qual elemento de B eles levam.
 Logo, para cada um desses n-m elementos, temos m opções.
 Ficando com: m^(n-m)

 Portanto, a resposta fica: A(n, m) * m^(n-m) = m^(n-m) * n! / (n-m)!

 Abraços,
 Salhab


 2011/3/26 Pedro Júnior pedromatematic...@gmail.com

 Um colega me propôs o seguinte problema, e não consegui modelar:
 Seja A um conjunto com n elementos e seja B um conjunto com m elementos,
 com n = m. Quantas funções sobrejetoras, f : A -- B, podemos formar?

 --

 Pedro Jerônimo S. de O. Júnior

 Professor de Matemática

 Geo João Pessoa – PB





-- 
/**/
神が祝福

Torres

=
Instru��es para entrar na lista, sair da lista e usar a lista em
http://www.mat.puc-rio.br/~obmlistas/obm-l.html
=


Re: [obm-l] Prova da OBM-nivel U (problema 6)

2011-03-26 Por tôpico Johann Dirichlet
Concordo plenamente, apesar de eu não ter conseguido fazer sozinho.

Depois de uma grata ajuda do Lopes, foi fácil:
1 - Se n é ímpar, a expressão do numerador é uma soma de quadrados,
logo p teria que ser da forma 4k+1
2 - Se n é par, o seu rtaciocínio prova que k é par.

A próxima ideia que tive era saber se, para todo n, existe um k, e vice-versa.

Em 26/03/11, charles9char...@gmail.com escreveu:
 Agora que papirei reciprocidade quadrática sei fazer :

 10^2n + 8*10^n + 1 = (10^n + 1)^2 + 6*10^n = 0 (60*k + 7)-
 (-6*10^n / 60*k + 7) = 1  (Símbolo de Legendre) .

 Agora é só aplicar as propriedades fundamentais ( tentem fazer! ) :

 (a*b / p) = (a/p) * (b/p)   (p primo ímpar)
 (-1 / p) = (-1)^((p-1)/2)
 (2 / p) = (-1)^((p^2 - 1) / 8)
 e o teorema da reciprocidade quadrática.

 Depois vc chega em algo do tipo : n+nk+k = 0 (mod 2) - (n+1)(k+1) = 0 (mod
 2) -
 n e k são pares!

 É incrivelmente fácil pra um problema 6, vcs não acham?



-- 
/**/
神が祝福

Torres

=
Instru��es para entrar na lista, sair da lista e usar a lista em
http://www.mat.puc-rio.br/~obmlistas/obm-l.html
=


[obm-l] Re: [obm-l] Questão de teoria dos números

2011-03-26 Por tôpico Johann Dirichlet
Isto parece óbvio: a parte inteira de uma fração é justamente oquociente na 
divisão euclidiana clássica. Logo, se aumentamos odivisor, o quociente 
naturalmente diminui.
Talvez a parte difícil seja usar álgebra nisso aí...
Em 26/03/11, enniusenn...@bol.com.br escreveu: Caros Colegas, Como podemos 
provar que a parte inteira de n/(2^k) é maior que a parte inteira de 
n/(5^k), para todo inteiro n1 e n maior ou igual a 2^k?  (k é inteiro 
positivo.) Abraços do Ennius. 
= 
Instruções para entrar na lista, sair da lista e usar a lista em 
http://www.mat.puc-rio.br/~obmlistas/obm-l.html 
=

-- /**/神が祝福
Torres
=
Instru��es para entrar na lista, sair da lista e usar a lista em
http://www.mat.puc-rio.br/~obmlistas/obm-l.html
=


Re: [obm-l] raizes

2011-03-24 Por tôpico Johann Dirichlet
Em 21/03/11, marcone augusto araújo
borgesmarconeborge...@hotmail.com escreveu:

 Onde encontro a fórmula para achar as raízes de uma equação do quarto grau?
A princípio, conheço dois métodos, e ambos só servem pra dizer que
existe uma fórmula usando radicais para as raízes de uma equação de
quarto grau.

O mais legal, EMMO, é o método Gugu-Euler:

Reduza a quártica a uma forma do tipo x^4=p*x^2+q*x+r, e escreva
x=sqrt(y1)+sqrt(y2)+sqrt(y3).

Abrindo as expressões feito um doido, você pode agrupar uns termos e
fazer umas suposições acerca dos y's, e descobrir que os y's são
raízes de uma equação de grau 3.
Aí é só resolver esta coisinha fofa que se origina, e pronto!

Estou sendo bem conciso, mas prometo que alguém poderá explicar em
detalhes depois...

   


-- 
/**/
神が祝福

Torres

=
Instru��es para entrar na lista, sair da lista e usar a lista em
http://www.mat.puc-rio.br/~obmlistas/obm-l.html
=


[obm-l] Re: [obm-l] Re: [obm-l] RES: [obm-l] nome de Matemático

2011-03-02 Por tôpico Johann Dirichlet
Quem me contou algo semelhante foi o Tengan ou o Humberto Naves: um
problema que foi resolvido por um aluno, porque ele se atrasou. Depois
ele entregou o trabalho de casa pro professor, que ficou apavorado!
com a notícia. Resolveu ate publica-los em umas revistas.

O nome e esse mesmo, George Dantzig. O cara virou a maior lenda urbana
da Matematica com isso, e a historia e mais lembrada do que a pessoa.

Outro caso curioso foi o de Erdos:

Ele tinha chegado numa universidade, e perguntou O que e isso na lousa?
Era um problema de uma area da qual Erdos nunca ouviu falar. Ademais,
a solução obtida pelos caras daquela faculdade tinha um monte de
folhas escritas.

Erdos fez umas perguntas sobre alguns detalhes que ele não entendia do
problema (algo de terminologia, creio eu), e depois deu uma solução em
poucas linhas.

Tem uma do Eduardo Tengan que eu conto outra hora...


Em 28/02/11, Welma Pereirawelma.pere...@gmail.com escreveu:
 Obrigada Albert. Eu não tenho certeza se história é real pois ouvi de um
 colega há uns anos atrás na universidade.

 2011/2/27 Albert Bouskela bousk...@msn.com

 Olá, Welma,



 Estudo, com razoável afinco, a história da Matemática e a biografia dos
 seus expoentes. Contudo, desconheço completamente o fato pitoresco ao qual
 você se refere.



 O mais próximo dele, que tenho na memória, é atribuído a Gauss: –
 Acredita-se que Gauss, quando estava no ensino básico, por castigo imposto
 pelo seu professor de Aritmética, tenha sido obrigado a somar todos os
 números compreendidos entre 1 e 100 (incluindo-os).  Gauss, então, deduziu
 a
 fórmula da soma dos termos de uma P.A., ao observar que 1+100 = 2+99 =
 3+98
 etc. = soma dos extremos [ a(1)+a(n) ].



 Atenção:

 i.Não há comprovação de que esta história (ou fábula)
 seja verdadeira;

   ii.A fórmula da soma dos termos de uma P.A. já era, há
 muito, conhecida.



 Albert Bouskela

 bousk...@msn.com



 *De:* owner-ob...@mat.puc-rio.br [mailto:owner-ob...@mat.puc-rio.br] *Em
 nome de *Welma Pereira
 *Enviada em:* 27 de fevereiro de 2011 13:07
 *Para:* obm-l@mat.puc-rio.br
 *Assunto:* [obm-l] nome de Matemático



 Olá Pessoal,



 Será que podiam me ajudar? Estou a procura do nome do matemático que
 resolveu um grande problema porque pensou que era lição de casa?



 Agradeço

 Welma




-- 
/**/
神が祝福

Torres

=
Instru��es para entrar na lista, sair da lista e usar a lista em
http://www.mat.puc-rio.br/~obmlistas/obm-l.html
=


[obm-l] Re: [obm-l] Re: [obm-l] RE: [obm-l] Infinitas soluç ões(números inteiros)

2011-01-10 Por tôpico Johann Dirichlet
Eu propus este problema (na verdade uma versao) na Eureka!
Bem, ele ja foi resolvido, no numero 30 se nao me engano.

Em 09/01/11, Ralph Teixeiraralp...@gmail.com escreveu:
 Aprendi esta ideia num problema de uma IMO:

 -- (1,1,1) eh solucao.
 -- Pense na equacao como uma quadratica em x: x^2-(3yz)x+(y^2+z^2)=0.
 A soma das raizes eh 3yz. Entao, se x=a eh uma solucao, a outra eh
 x=3yz-a.
 -- Em outras palavras, o que mostramos eh que se (x,y,z) eh solucao,
 entao (3yz-x,y,z) tambem eh (o que poderia ser verificado
 diretamente).
 -- Por simetria, (x,3xz-y,z) e (x,y,3xy-z) tambem servem.
 -- Isso gera uma famila de solucoes:
 (1,1,1) - (3.1.1-1,1,1)=(2,1,1) - (2,3.1.1-1,1)=(2,5,1) -
 (2,5,3.2.5-1)=(2,5,29) - (3.5.29-2,5,29) - ...-
 -- Note que na construcao eu escolhi sempre trocar o MENOR dos numeros
 (x,y,z). Entao a cada passo a soma da terna passou de x+y+z para
 (3yz-x)+(y+z), onde x eh o menor dos tres numeros. Como claramente
 3yz-xx (pois 3yz=3.x.12x), entao a soma eh estritamente crescente,
 e as ternas sao todas diferentes.

 Abraco, Ralph.

 2011/1/9 marcone augusto araújo borges marconeborge...@hotmail.com:
 corrigindo: x^2 + y^2 + z^2 = 3xyz

 
 From: marconeborge...@hotmail.com
 To: obm-l@mat.puc-rio.br
 Subject: [obm-l] Infinitas soluções(números inteiros)
 Date: Sun, 9 Jan 2011 02:10:07 +

 mostre q a equação x^2 + y^2 +z^2 = xyz tem infinitas soluções onde x,y,z
 são números inteiros.
 Agradeço a todos q ajudarem.


 =
 Instruções para entrar na lista, sair da lista e usar a lista em
 http://www.mat.puc-rio.br/~obmlistas/obm-l.html
 =



-- 
/**/
Quadrinista e Taverneiro!

http://tavernadofimdomundo.blogspot.com  Quadrinhos, histórioas e afins
http://baratoeletrico.blogspot.com / Um pouco sobre elétrons em movimento
http://bridget-torres.blogspot.com/  Personal! Do not edit!

=
Instru��es para entrar na lista, sair da lista e usar a lista em
http://www.mat.puc-rio.br/~obmlistas/obm-l.html
=


Re: [obm-l] Prova da OBM-nivel U (problema 6)

2011-01-10 Por tôpico Johann Dirichlet
Aonde eu acho esse cara??


Em 01/01/11, charles9char...@gmail.com escreveu:
 O Leandro Farias fez!



-- 
/**/
Quadrinista e Taverneiro!

http://tavernadofimdomundo.blogspot.com  Quadrinhos, histórioas e afins
http://baratoeletrico.blogspot.com / Um pouco sobre elétrons em movimento
http://bridget-torres.blogspot.com/  Personal! Do not edit!

=
Instru��es para entrar na lista, sair da lista e usar a lista em
http://www.mat.puc-rio.br/~obmlistas/obm-l.html
=


Re: [obm-l] Teoria dos Conjuntos

2011-01-01 Por tôpico Johann Dirichlet
Bem, isto me parece um pouco com um daqueles paradoxos (acho que o de
Russel). Acho que nao e muito facil construir uma coisa dessas.
Antes de mais nada, isto vai incorrer em perguntas do tipo mas isto é
um axioma da teoria dos conjuntos?. Como eu conheço bem pouco, eu
prefiro dar uma referência: Paul Halmos, Naive Set Theory (Teoria
Ingenua dos Conjuntos),. aliás um livro nada ingênuo :P

P.S.:  Um errinho de portugues: este consertando e com s. Mas acho que
se eu repetir isto provocarei uma flame war...

Em 22/12/10, Vinícius Harlockcortes...@gmail.com escreveu:
 Concertando:

 Agora eu só quero um exemplo.
 Seja R o conjunto dos números reais, diga-me um elemento que pertença ao
 conjunto A, tal que A={x∈R|x∉x} e outro elemento que pertença a B, tal  B={x
 ∈R|x∈x}. Se por acaso R não servir como conjunto universo, troque-o por um
 que sirva.


 Agradeço muito a atenção de todos =8^B
 a[b]'s



-- 
/**/
Quadrinista e Taverneiro!

http://tavernadofimdomundo.blogspot.com  Quadrinhos, histórioas e afins
http://baratoeletrico.blogspot.com / Um pouco sobre elétrons em movimento
http://bridget-torres.blogspot.com/  Personal! Do not edit!

=
Instru��es para entrar na lista, sair da lista e usar a lista em
http://www.mat.puc-rio.br/~obmlistas/obm-l.html
=


[obm-l] Re: [obm-l] SEMANA OLÍMPICA

2011-01-01 Por tôpico Johann Dirichlet
Bem, eu direi em carater nao-oficial: acho que ate os medalhistas de
prata sao bancados pela OBM. Ja o bronze, e uma especie de
50-50(passagem, mas nao os dias de hospedagem), e os menção honrosa e
100-0.
De todo modo, se voce foi agraciado com um premio, voce sera melhor informado.

Em 28/12/10, charles9char...@gmail.com escreveu:
 Os premiados com medalha do nível universitário recebem medalha como nos
 outros níveis?
 E quanto a semana olímpica, devemos pagar para participar?

 Obrigado.



-- 
/**/
Quadrinista e Taverneiro!

http://tavernadofimdomundo.blogspot.com  Quadrinhos, histórioas e afins
http://baratoeletrico.blogspot.com / Um pouco sobre elétrons em movimento
http://bridget-torres.blogspot.com/  Personal! Do not edit!

=
Instru��es para entrar na lista, sair da lista e usar a lista em
http://www.mat.puc-rio.br/~obmlistas/obm-l.html
=


[obm-l] Prova da OBM-nivel U (problema 6)

2011-01-01 Por tôpico Johann Dirichlet
Ola povo!
Alguem resolveu o problema 6 da OBMU?
Se p e um primo da forma 60k+7, e p divide 10^(2n)+8*10^n+1, então n
e k são ambos pares.

Por ora, não estou conseguindo ter nenhuma ideia... Por ora, pensei em
hensel, mas nao testei ainda.

-- 
/**/
Quadrinista e Taverneiro!

http://tavernadofimdomundo.blogspot.com  Quadrinhos, histórioas e afins
http://baratoeletrico.blogspot.com / Um pouco sobre elétrons em movimento
http://bridget-torres.blogspot.com/  Personal! Do not edit!

=
Instru��es para entrar na lista, sair da lista e usar a lista em
http://www.mat.puc-rio.br/~obmlistas/obm-l.html
=


[obm-l] Re: [obm-l] Campeões da Matemática - Resultados da 32a. Olimpíada Brasileira de Matemática - OBM

2010-12-19 Por tôpico Johann Dirichlet
faltou o resultado do Torneio das Cidades :)
Apesar de eu nao saber se ele e realizado pela SBM ou OBM, seria bom
ter alguma info.


Em 16/12/10, Olimpiada Brasileira de Matematicao...@impa.br escreveu:

 Campeões da Matemática - Resultados da 32a. Olimpíada Brasileira de
 Matemática - OBM

 Por mais um ano consecutivo estamos finalizando a realização da
 Olimpíada Brasileira de Matemática. Conheça a listagem de premiados da
 OBM -- 2010 no endereço: www.obm.org.br http://www.obm.org.br/

 A Olimpíada Brasileira de Matemática tem crescido nos últimos anos,
 contando, este ano, com a adesão ao Programa de mais de 4.651 escolas,
 sendo 2.770 da rede pública e 1.881 da rede privada de ensino, o que
 implica em uma participação na Olimpíada Brasileira de Matemática de
 cerca de 350.000 jovens estudantes e seus professores.Além disso, o
 Programa Nacional de Olimpíadas de Matemática conta com a colaboração de
 professores universitários em 155 instituições de ensino superior: eles
 participam de todas as atividades da Olimpíada Brasileira de Matemática,
 inclusive aquelas referentes à OBM Nível Universitário em atividades de
 coordenação, divulgação, treinamento de alunos, aperfeiçoamento de
 professores e aplicação das distintas fases da Olimpíada Brasileira de
 Matemática.

 No que se refere à participação em competições internacionais, os
 resultados são excelentes:

 * Olimpíada de Matemática do Cone Sul (Águas de São Pedro -- São
   Paulo): uma medalha de Ouro, duas de Prata e uma de Bronze.
 * Olimpíada Internacional de Matemática -- IMO (Astana --
   Cazaquistão): duas medalhas de Prata, uma medalha de Bronze.
 * Olimpíada Internacional de Matemática para Estudantes
   Universitários -- IMC (Blagoevgrad -- Bulgária): uma medalha de
   Ouro, duas medalhas de Prata, oito medalhas de Bronze e oito
   Menções Honrosas.
 * Olimpíada Iberoamericana de Matemática (Assunção -- Paraguai.):
   três medalhas de Ouro e uma medalha de Prata.
 * Competição Interuniversitária Iberoamericana de Matemática -- CIIM
   (Rio de Janeiro -- RJ): seis medalhas de Ouro (sendo duas de ouro
   especial), cinco medalhas de Prata e oito medalhas de Bronze.
 * Olimpíada de Maio (Organizada pela Argentina): duas medalhas de
   Ouro, quatro medalhas de Prata e oito medalhas de Bronze
   distribuídas entre os dois níveis da competição.
 * Asian Pacific Mathematical Olympiad (APMO): três medalhas de
   Prata, quatro medalhas de Bronze distribuídas entre os dois níveis
   da competição.
 * Romanian Máster in Mathematics (RMM) (Bucarest -- Romênia): três
   medalhas de Bronze e uma menção honrosa.
 * Este ano também realizamos o Concurso Canguru Matemático Sem
   Fronteiras, concurso que reúne participantes de 42 países.

 Durante 2009 a CAPES e o CNPq lançaram o Programa de Iniciação
 Científica -- Mestrado (PICME) para medalhistas da OBMEP e OBM, com o
 objetivo de aumentar o número de matemáticos no país, e oferecer uma
 formação matemática mais sólida a jovens profissionais de outras áreas
 científicas e tecnológicas.

 Todos estes resultados nacionais e internacionais demonstram que, além
 de influenciar positivamente o ensino da Matemática nas instituições de
 ensino fundamental, médio e superior, conseguimos detectar jovens muito
 talentosos que são estimulados a seguir uma carreira científica, o que é
 fundamental para o crescimento da Ciência e Tecnologia no país.

 A Olimpíada Brasileira de Matemática é um projeto conjunto da Sociedade
 Brasileira de Matemática (SBM), do Instituto Nacional de Matemática Pura
 e Aplicada (IMPA) e conta com o apoio do Conselho Nacional de
 Desenvolvimento Científico e Tecnológico (CNPq) e do Instituto Nacional
 de Ciência e Tecnologia de Matemática (INCT -- Mat).

 *Informações:*

 --
 Secretaria da Olimpíada Brasileira de Matemática
 Estrada Dona Castorina, 110 Jd. Botânico,
 Rio de Janeiro - RJ, 22460-320, Brasil
 Tel: 55-21-25295077 Fax:55-21-25295023
 e-mail:o...@impa.br
 web site:www.obm.org.br




-- 
/**/
Quadrinista e Taverneiro!

http://tavernadofimdomundo.blogspot.com  Quadrinhos, histórioas e afins
http://baratoeletrico.blogspot.com / Um pouco sobre elétrons em movimento
http://bridget-torres.blogspot.com/  Personal! Do not edit!

=
Instru��es para entrar na lista, sair da lista e usar a lista em
http://www.mat.puc-rio.br/~obmlistas/obm-l.html
=


[obm-l] Demonstrar Frações Parciais com Álgebra Linear

2010-12-19 Por tôpico Johann Dirichlet
Olá pessoas!

Faz algum tempo atrás, eu tinha um livro de Cálculo 1 + Álgebra
Linear. Entre outras coisas, ele ensinava a calcular integrais de
funcoes racionais (aquelas que estao ficando famosas na lista:
integral de (P(x)/Q(x)), em que P e Q são polinômios).

Nisto, ele tinha um apêndice em que demonstrava, usando Álgebra
Linear, que e possivel quebrar P/Q em fracoes parciais.

Mais precisamente, todos devem conhecer o resultado: se o grau de P e
menor que o grau de Q, e Q se fatora como (x-r)^m, as fracoes parciais
tem a forma
C/(x-r)^(1)+C/(x-r)^(2)+...+C/(x-r)^(m)

Pois bem, eu nao tenho mais o livro :(
Portanto, eu queria uma demonstração usando Álgebra Linear do fato acima.
Eu lembro que era algo corriqueiro: demonstrar que as fracoes acima
formavam um espaco vetorial de dimensao K, e depois achar uma K-base.
Mas os detalhes me fogem...

Desde já, agradeço!

-- 
/**/
Quadrinista e Taverneiro!

http://tavernadofimdomundo.blogspot.com  Quadrinhos, histórioas e afins
http://baratoeletrico.blogspot.com / Um pouco sobre elétrons em movimento
http://bridget-torres.blogspot.com/  Personal! Do not edit!

=
Instru��es para entrar na lista, sair da lista e usar a lista em
http://www.mat.puc-rio.br/~obmlistas/obm-l.html
=


[obm-l] Re: [obm-l] Re: [obm-l] Demonstrar Frações Parciai s com Álgebra Linear

2010-12-19 Por tôpico Johann Dirichlet
O titulo era simplesmente O Calculo com Algebra Linear. Nao sei nem
os autores direito... Ele versava sobre Calculo e bem pouco sobre
AlgeLin, A mais marcante aplicação foi justamente esta.

Em 19/12/10, João Luís Gomes Guimarãesjoaolui...@uol.com.br escreveu:

 Olá Johann,

 Não se lembra qual era o livro?

 JL

 -Mensagem Original-
 From: Johann Dirichlet
 Sent: Sunday, December 19, 2010 3:05 PM
 To: obm-l
 Subject: [obm-l] Demonstrar Frações Parciais com Álgebra Linear

 Olá pessoas!

 Faz algum tempo atrás, eu tinha um livro de Cálculo 1 + Álgebra
 Linear. Entre outras coisas, ele ensinava a calcular integrais de
 funcoes racionais (aquelas que estao ficando famosas na lista:
 integral de (P(x)/Q(x)), em que P e Q são polinômios).

 Nisto, ele tinha um apêndice em que demonstrava, usando Álgebra
 Linear, que e possivel quebrar P/Q em fracoes parciais.

 Mais precisamente, todos devem conhecer o resultado: se o grau de P e
 menor que o grau de Q, e Q se fatora como (x-r)^m, as fracoes parciais
 tem a forma
 C/(x-r)^(1)+C/(x-r)^(2)+...+C/(x-r)^(m)

 Pois bem, eu nao tenho mais o livro :(
 Portanto, eu queria uma demonstração usando Álgebra Linear do fato acima.
 Eu lembro que era algo corriqueiro: demonstrar que as fracoes acima
 formavam um espaco vetorial de dimensao K, e depois achar uma K-base.
 Mas os detalhes me fogem...

 Desde já, agradeço!

 --
 /**/
 Quadrinista e Taverneiro!

 http://tavernadofimdomundo.blogspot.com  Quadrinhos, histórioas e afins
 http://baratoeletrico.blogspot.com / Um pouco sobre elétrons em movimento
 http://bridget-torres.blogspot.com/  Personal! Do not edit!

 =
 Instru��es para entrar na lista, sair da lista e usar a lista em
 http://www.mat.puc-rio.br/~obmlistas/obm-l.html
 =


 =
 Instru�ões para entrar na lista, sair da lista e usar a lista em
 http://www.mat.puc-rio.br/~obmlistas/obm-l.html
 =



-- 
/**/
Quadrinista e Taverneiro!

http://tavernadofimdomundo.blogspot.com  Quadrinhos, histórioas e afins
http://baratoeletrico.blogspot.com / Um pouco sobre elétrons em movimento
http://bridget-torres.blogspot.com/  Personal! Do not edit!

=
Instru��es para entrar na lista, sair da lista e usar a lista em
http://www.mat.puc-rio.br/~obmlistas/obm-l.html
=


[obm-l] Re: [obm-l] Re: [obm-l] Re: [obm-l] Re: [obm-l] Re: [obm-l] O produto de n inteiros consecutivos é múltiplo do fatorial de n

2010-12-09 Por tôpico Johann Dirichlet
Bem, respondendo:
1 - Errei: para k=0 o valor é 1
2 - Tem uma especie de dispositivo pratico, que funciona na mesma
ideia do triangulo de Pascal:

0 0 0 0 0 ... 0 1
 0 0 0 0 ... 0 1
  0 0 0 ... 0 1
   0 0 ... 0 1
 0 ... 1

  1

Este e o triangulo das diferenças de f(n,k).
Depois de um numero finito de passos (n+1, se nao me engano) a ultima
linha fica constante (neste caso igual a 1).
Ai e so reverter...

Existe uma formula pronta, mas eu quase nao decoro...

Em 09/12/10, Henrique Rennóhenrique.re...@gmail.com escreveu:
 Em 28/11/10, Johann Dirichletpeterdirich...@gmail.com escreveu:
 Por que este povo tem tanto pavor de uma prova que não use outros
 conceitos alem do enunciado?
 Eu mesmo conheço vários problemas que são resolvidos usando outras
 técnicas. Na IMO de Glasgow teve um problema de Teoria dos Números com
 uma solução que usava polinômios. E tem um monte de problemas de
 teoria dos números que se resolvem usando técnicas de combinatória (o
 teorema de Euler-Fermat, por exemplo).

 De todo modo, só pra não perder o propósito da mensagem:

 Uma maneira seria observar que f(n,k)=(k+1)(k+2)...(k+n)/n! é um
 polinômio de grau n em k.
 Ele é completamnte determinado se eu utilizar (n+1) valores de k.

 Para k de -1 até -n, este polinômio é igual a zero, e para k=n+1 ele vale
 1.
 A partir daí, usando a fórmula de interpolação de Newton (ou uma
 modificação do triângulo de Pascal), este polinômio é inteiro para
 todo n inteiro.

 Como isso pode ser verificado?



 Em 27/11/10, Carlos Alberto da Silva Victorvictorcar...@globo.com
 escreveu:
 Olá Paulo,
 Verifique se esta ideia satisfaz o que desejas .

  Por indução :

 1) para n=1,2 e 3 é fácil de observar tal fato .
 2) hipótese : válida para  n fatores consecutivos.

 3) Tomemos (n+1) fatores consecutivos :P =  k(k+1)(k+n-1).(k+n) .Por
 hipótese k(k+1)(k+n-1) é divisível por n! . Não é difícil mostrar que
 o
 produto de n fatores consecutivos é divisível por n .Como P possui (n+1)
 fatores, temos que o valor (n+1) está em um dos fatores(ou divisor de um
 dos
 fatores) de P e, já que n e (n+1) são primos entre si , P será divisível
 por
 n! e (n+1) , ou seja, divisível por (n+1)! , ok ?

 Abraços

 Carlos  Victor





 Em 27 de novembro de 2010 18:29, Paulo Argolo
 argolopa...@hotmail.comescreveu:

  Obrigado, Tiago.

 O que desejo, na verdade, é obter uma demonstração que não use
 propriedades
 dos coeficientes binomiais, nem recorra à Análise Combinatória. Em suma:
 gostaria de ver uma prova puramente aritmética.

 Abraços do Paulo!






 --
 /**/
 Quadrinista e Taverneiro!

 http://tavernadofimdomundo.blogspot.com  Quadrinhos, histórioas e afins
 http://baratoeletrico.blogspot.com / Um pouco sobre elétrons em
 movimento
 http://bridget-torres.blogspot.com/  Personal! Do not edit!

 =
 Instru�ões para entrar na lista, sair da lista e usar a lista em
 http://www.mat.puc-rio.br/~obmlistas/obm-l.html
 =



 --
 Henrique

 =
 Instru�ões para entrar na lista, sair da lista e usar a lista em
 http://www.mat.puc-rio.br/~obmlistas/obm-l.html
 =



-- 
/**/
Quadrinista e Taverneiro!

http://tavernadofimdomundo.blogspot.com  Quadrinhos, histórioas e afins
http://baratoeletrico.blogspot.com / Um pouco sobre elétrons em movimento
http://bridget-torres.blogspot.com/  Personal! Do not edit!

=
Instru��es para entrar na lista, sair da lista e usar a lista em
http://www.mat.puc-rio.br/~obmlistas/obm-l.html
=


[obm-l] Re: [obm-l] Re: [obm-l] Re: [obm-l] O produto de n i nteiros consecutivos é múltiplo do fatorial de n

2010-11-28 Por tôpico Johann Dirichlet
Por que este povo tem tanto pavor de uma prova que não use outros
conceitos alem do enunciado?
Eu mesmo conheço vários problemas que são resolvidos usando outras
técnicas. Na IMO de Glasgow teve um problema de Teoria dos Números com
uma solução que usava polinômios. E tem um monte de problemas de
teoria dos números que se resolvem usando técnicas de combinatória (o
teorema de Euler-Fermat, por exemplo).

De todo modo, só pra não perder o propósito da mensagem:

Uma maneira seria observar que f(n,k)=(k+1)(k+2)...(k+n)/n! é um
polinômio de grau n em k.
Ele é completamnte determinado se eu utilizar (n+1) valores de k.

Para k de -1 até -n, este polinômio é igual a zero, e para k=n+1 ele vale 1.
A partir daí, usando a fórmula de interpolação de Newton (ou uma
modificação do triângulo de Pascal), este polinômio é inteiro para
todo n inteiro.


Em 27/11/10, Carlos Alberto da Silva Victorvictorcar...@globo.com escreveu:
 Olá Paulo,
 Verifique se esta ideia satisfaz o que desejas .

  Por indução :

 1) para n=1,2 e 3 é fácil de observar tal fato .
 2) hipótese : válida para  n fatores consecutivos.

 3) Tomemos (n+1) fatores consecutivos :P =  k(k+1)(k+n-1).(k+n) .Por
 hipótese k(k+1)(k+n-1) é divisível por n! . Não é difícil mostrar que o
 produto de n fatores consecutivos é divisível por n .Como P possui (n+1)
 fatores, temos que o valor (n+1) está em um dos fatores(ou divisor de um dos
 fatores) de P e, já que n e (n+1) são primos entre si , P será divisível por
 n! e (n+1) , ou seja, divisível por (n+1)! , ok ?

 Abraços

 Carlos  Victor





 Em 27 de novembro de 2010 18:29, Paulo Argolo
 argolopa...@hotmail.comescreveu:

  Obrigado, Tiago.

 O que desejo, na verdade, é obter uma demonstração que não use
 propriedades
 dos coeficientes binomiais, nem recorra à Análise Combinatória. Em suma:
 gostaria de ver uma prova puramente aritmética.

 Abraços do Paulo!






-- 
/**/
Quadrinista e Taverneiro!

http://tavernadofimdomundo.blogspot.com  Quadrinhos, histórioas e afins
http://baratoeletrico.blogspot.com / Um pouco sobre elétrons em movimento
http://bridget-torres.blogspot.com/  Personal! Do not edit!

=
Instru��es para entrar na lista, sair da lista e usar a lista em
http://www.mat.puc-rio.br/~obmlistas/obm-l.html
=


[obm-l] Re: [obm-l] Como provar que C(n,p) é número natura l?

2010-11-24 Por tôpico Johann Dirichlet
A melhor que eu posso imaginar e simplesmente pensar assim:

1 - Determine, para cada primo p, a maior potencia de p que divide n!
(ou seja, descubra na raça a fatoração de n!).
E facil: basta contar quanto cada p, 2p, 3p, ... (p-1)p, p^2, etc vai
contribuir (voce vai obter um somatorio).
Isso tem mais a ver com teoria dos conjuntos que com teoria dos números.

2 - A partir dai, fica facil provar que a potencia de p que aparece no
numerador não é menor que no denominador.



Em 23/11/10, Pedro Chavesbrped...@hotmail.com escreveu:

 Amigos da Lista,


 Como posso provar que C(n,p) é um número natural, usando apenas a definição
 C(n,p) = n! / [p! (n-p)!]?
 (p e n são números naturais, com p menor ou igual a n)
 Meu objetivo é obter uma prova direta, isto é, que não recorra à Análise
 Combinatória, nem às  propriedades dos números binomiais.

 Muito grato!

 Pedro Chaves
   


-- 
/**/
Quadrinista e Taverneiro!

http://tavernadofimdomundo.blogspot.com  Quadrinhos, histórioas e afins
http://baratoeletrico.blogspot.com / Um pouco sobre elétrons em movimento
http://bridget-torres.blogspot.com/  Personal! Do not edit!

=
Instru��es para entrar na lista, sair da lista e usar a lista em
http://www.mat.puc-rio.br/~obmlistas/obm-l.html
=


Re: [obm-l] geometria com 20 graus

2010-11-24 Por tôpico Johann Dirichlet
Se não me engano este problema foi proposto numa Eureka! Assim que der
eu vejo qual o número, mas é recente (entre as últimas 8 ou 10).

Em 16/11/10, Luís Lopesqed_te...@hotmail.com escreveu:

 Sauda,c~oes,

 Pediram-me a solução do problema abaixo. Como muito provavelmente
 tal problema já apareceu por aqui, pergunto se alguém teria a solução
 dele à mão.

 Obrigado.

 []'s
 Luís




  Prezado Luís mais uma vez venho pedir a sua ajuda na solução do exercicio
 abaixo.
 Dado o triângulo ABC ,isósceles, com BÂC medindo 20 graus ,onde AC = AB.
 Pelo vértice B traça-se até AC, BD tal que AD = BC. Calcule o valor do
 ângulo B^DC.


   


-- 
/**/
Quadrinista e Taverneiro!

http://tavernadofimdomundo.blogspot.com  Quadrinhos, histórioas e afins
http://baratoeletrico.blogspot.com / Um pouco sobre elétrons em movimento
http://bridget-torres.blogspot.com/  Personal! Do not edit!

=
Instru��es para entrar na lista, sair da lista e usar a lista em
http://www.mat.puc-rio.br/~obmlistas/obm-l.html
=


[obm-l] Re: [obm-l] mdc (a^x – 1, a^y – 1, a^z – 1, .. .......) = [a^mdc(x, y, z,...)] – 1

2010-11-23 Por tôpico Johann Dirichlet
Para dois caras, é fácil demonstrar na raça, usando Euclides:
MDC(a^x-1,a^y-1)= MDC(a^x-1,a^(x-y)-1). Daí se faz por indução no
número de variáveis.

Em 23/11/10, Paulo  Argolopauloarg...@bol.com.br escreveu:
 Caros Colegas,
 Estou refazendo o enunciado da questão.

 Como provar o teorema seguinte sobre máximo divisor comum?

 TEOREMA:

 O máximo divisor comum (mdc) dos números do tipo
 a^x -1 , onde a e x são números inteiros maiores do que 1, é dado pela
 expressão abaixo:
 mdc(a^x - 1, a^y - 1, a^z - 1, ... ) = [a^mdc(x, y, z,...)] -1

 Grato,

 Paulo Argolo
 =
 Instru�ões para entrar na lista, sair da lista e usar a lista em
 http://www.mat.puc-rio.br/~obmlistas/obm-l.html
 =



-- 
/**/
Quadrinista e Taverneiro!

http://tavernadofimdomundo.blogspot.com  Quadrinhos, histórioas e afins
http://baratoeletrico.blogspot.com / Um pouco sobre elétrons em movimento
http://bridget-torres.blogspot.com/  Personal! Do not edit!

=
Instru��es para entrar na lista, sair da lista e usar a lista em
http://www.mat.puc-rio.br/~obmlistas/obm-l.html
=


[obm-l] Re: [obm-l] Alguém conseguirá provar?

2010-11-21 Por tôpico Johann Dirichlet
O que significa limitada primorialmente?


Em 05/11/10, Marco Bivarmarco.bi...@gmail.com escreveu:
 Prove isto:

 Em toda sucessão (c_1, c_2, ..., c_w) de números compostos limitada
 primorialmente, se c_i = z_i . x_i, i=1,2,..., w, onde z_i é um primo ou
 produto de primos tal que z_i=x_i e z_1z_2...z_w, então x_1x_2...x_w,
 onde x_i é um primo ou produto de primos.

 Obs.: Tomar apenas números primos positivos.


 --
 Marco A. B. C. Jr.



-- 
/**/
Quadrinista e Taverneiro!

http://tavernadofimdomundo.blogspot.com  Quadrinhos, histórioas e afins
http://baratoeletrico.blogspot.com / Um pouco sobre elétrons em movimento
http://bridget-torres.blogspot.com/  Personal! Do not edit!

=
Instru��es para entrar na lista, sair da lista e usar a lista em
http://www.mat.puc-rio.br/~obmlistas/obm-l.html
=


Re: [obm-l] Teorema sobre logaritmos irracionais

2010-11-09 Por tôpico Johann Dirichlet
log_b a= x é o mesmo que a^x=b.
Usando o lema da fatoração única, vemos que se x fosse racional então
a e b teriam os mesmos fatores primos e com os expoentes múltiplos.

Em 06/11/10, Pedro Chavesbrped...@hotmail.com escreveu:

 Estou reapresentando o teorema sobre logaritmos, pois não consegui ainda uma
 demonstração completa. Peço, mais uma vez, a colaboração dos colegas.

 Teorema:
 Sendo a e b números inteiros maiores do que 1, que não podem ser
 representados como potências de mesma base (inteira), com expoente inteiro,
 então o logaritmo de a, na base b, é um número irracional.


 Um abraço do Pedro!   


-- 
/**/
Quadrinista e Taverneiro!

http://tavernadofimdomundo.blogspot.com  Quadrinhos, histórioas e afins
http://baratoeletrico.blogspot.com / Um pouco sobre elétrons em movimento
http://bridget-torres.blogspot.com/  Personal! Do not edit!

=
Instru��es para entrar na lista, sair da lista e usar a lista em
http://www.mat.puc-rio.br/~obmlistas/obm-l.html
=


[obm-l] Re: [obm-l] Conjuntos Enumeráveis

2010-10-30 Por tôpico Johann Dirichlet
A ideia não é difícil, e o mais importante é o caso 2: X x Yé
enumerável se X,Y são.
Faz assim: os elementos de X são x1,x2,... e os de Y são y1,y2,y3...
(ambos são enumeráveis, então eu posso colocar índices)

Então podemos fazer assim:
Para cada natural N = 1,2,3,4,5...
liste os pares (xi,yj) tal que i+j=N

Teremos algo assim:
(x1,y1)
(x1,y2),(x2,y1)
(x1,y3),(x2,y2),(x3,y1)
E por aí vai...

Aí, basta aplicar o caso n=2 fazendo X=A1 x A2 x ... x An e Y=A(n+1)

Sem indução é mais fácil ainda: basta utilizar o algoritmo acima.

Em 30/10/10, Luiz Neto Netouizn...@yahoo.com.br escreveu:
 Sejam A1,An conjuntos enumeráveis, então A1xxAn é enumerável(Use
 Indução)






-- 
/**/
Quadrinista e Taverneiro!

http://tavernadofimdomundo.blogspot.com  Quadrinhos, histórioas e afins
http://baratoeletrico.blogspot.com / Um pouco sobre elétrons em movimento
http://bridget-torres.blogspot.com/  Personal! Do not edit!

=
Instru��es para entrar na lista, sair da lista e usar a lista em
http://www.mat.puc-rio.br/~obmlistas/obm-l.html
=


[obm-l] Olimpíadas ao Redor do Mundo

2010-10-30 Por tôpico Johann Dirichlet
Eis um problema legal:

Temos três caixas, cada uma com pelo menos uma bolinha dentro.
Podemos dobrar o total de bolinhas de uma das caixas, tirando as
bolinhas de uma das outras caixas para tal.
É possível esvaziar uma das caixas, fazendo uma escolha acertada de
operações permitidas?

-- 
/**/
Quadrinista e Taverneiro!

http://tavernadofimdomundo.blogspot.com  Quadrinhos, histórioas e afins
http://baratoeletrico.blogspot.com / Um pouco sobre elétrons em movimento
http://bridget-torres.blogspot.com/  Personal! Do not edit!

=
Instru��es para entrar na lista, sair da lista e usar a lista em
http://www.mat.puc-rio.br/~obmlistas/obm-l.html
=


[obm-l] Re: [obm-l] Re: [obm-l] Fibonacci e Razão Áurea

2010-10-28 Por tôpico Johann Dirichlet
Poxa, alguém tem um exemplo de uma sequencia x_n que sempre é positiva
mas o limite não é?
Eu acho que 1/n tende a zero sempre sendo maior que zero, mas tem que
tomar cuidado com o estritamente positivo.

P.S.: um treco legal sobre racionais tendendo a irracionais é o artigo
do Gugu na Eureka! 3, sobre frações contínuas. Se eu não me engano os
F/F são reduzidas da fração contínua da razão áurea.


Em 27/10/10, Ralph Teixeiraralp...@gmail.com escreveu:
 Como pode uma razão de números inteiros convergir para um número irracional
 ?

 Bom, como ilustração, pi é irracional, e é o limite da sequencia:

 3
 3,1=31/10
 3,14=314/100
 3,141=3141/1000
 3,1415=31415/1
 ...

 Acho que este exemplo deve te convencer que qualquer número irracional é
 limite de uma sequencia de racionais (razões entre inteiros).

 ---///---

 Para ponderar: raciocínios do tipo: se cada x_n tem a propriedade P, então
 lim(x_n) tem a propriedade P são muito naturais. Infelizmente, este tipo de
 raciocínio está frequentemente errado! Por exemplo, seu espanto acima seria
 representado pela frase:

 se cada x_n é racional (quociente de inteiros), então lim(x_n), se existir,
 também será.
 (FALSO!)

 Outras frases FALSAS do mesmo tipo (todos os limites são quando n-+Inf):
 se cada x_n é positivo, então lim(x_n) é positivo.
 se cada x_n é menor que 1, então lim(x_n) é menor que 1 (que, no fundo no
 fundo, é o problema que o pessoal tem com 0,9...=1)
 se cada uma das funções f_n(x) é contínua, então f(x)=lim f_n(x) é
 contínua
 se cada uma das funções f_n(x) é derivável, então f(x)=lim f_n(x) é
 derivável

 Bom, e assim por diante. O que eu quero dizer é que passar um raciocínio
 ao limite é perigoso (mas, quando funciona, é bem legal)

 Abraço,

 Ralph


 2010/10/27 luiz silva luizfelipec...@yahoo.com.br

   Pessoal,

 Pelo que lembro, a razão entre dois números consecutivos(an+1/an), da
 sequência Fibonacci converge para 1,61834 quando n tende a infinito.
 Porém, pelo que lembro, tb, este número é um número irracional.

 Como pode uma razão de números inteiros convergir para um número
 irracional
 ?

 Abs
 Felipe





-- 
/**/
Quadrinista e Taverneiro!

http://tavernadofimdomundo.blogspot.com  Quadrinhos, histórioas e afins
http://baratoeletrico.blogspot.com / Um pouco sobre elétrons em movimento
http://bridget-torres.blogspot.com/  Personal! Do not edit!

=
Instru��es para entrar na lista, sair da lista e usar a lista em
http://www.mat.puc-rio.br/~obmlistas/obm-l.html
=


[obm-l] Re: [obm-l] Re: [obm-l] Determinar parte inteira e p arte fracionária

2010-10-28 Por tôpico Johann Dirichlet
Eu creio que a resposta é o famigerado -9.

Melhor ser mais preciso nas definições: parte inteira de x ou maior
inteiro que não supera x?

Em 27/10/10, Adalberto Dornellesaadornell...@gmail.com escreveu:
 Olá Pedro,

 A resposta depende de como você define parte fracionária. Parece que não
 há uma definição universalmente aceita. Veja por exemplo:
 http://mathworld.wolfram.com/FractionalPart.html

 http://mathworld.wolfram.com/FractionalPart.htmlTalvez a definição mais
 usual seja

 Parte fracionaria de x = x - arredondar para baixo x

 ou, em notação (também não muito universal),

 {x} = x - [x]

 Assim,
 {8,25} = 8,25 - [8,25] = 8,25 - 8 = 0,25
 e
 {-8,25} = -8,25 - [-8,25] = -8,25 - (-9) = 0,75

 Agora, observe que arredondar para baixo significa encontrar o inteiro n
 à esquerda de x ou arredondar na direção de - infinito. No entanto, outra
 definição de arredondar para baixo pode ser arredondar na direção de 0,
 assim

 {8,25} = 8,25 - [8,25] = 8,25 - 8 = 0,25
 e
 {-8,25} = -8,25 - [-8,25] = -8,25 - (-8) = -0,25

 Em computação, geralmente as linguagens [Matlab, Mathematica, etc.] tem
 comandos relacionados a isso (floor, ceil, fix, round, mod, ...). cuja
 definição não é muito homogênea. Sempre recomendo leia o manual para saber
 como a linguagem define cada coisa.

 Veja também http://en.wikipedia.org/wiki/Floor_and_ceiling_functions

 Abraço,
 Adalberto

 Em 27 de outubro de 2010 08:09, Pedro Chaves brped...@hotmail.comescreveu:

  Caros Colegas,

 Qual é a parte inteira e a parte fracionária do número real negativo
 -8,25?

 Obrigado!
 Pedro Chaves




-- 
/**/
Quadrinista e Taverneiro!

http://tavernadofimdomundo.blogspot.com  Quadrinhos, histórioas e afins
http://baratoeletrico.blogspot.com / Um pouco sobre elétrons em movimento
http://bridget-torres.blogspot.com/  Personal! Do not edit!

=
Instru��es para entrar na lista, sair da lista e usar a lista em
http://www.mat.puc-rio.br/~obmlistas/obm-l.html
=


[obm-l] Re: [obm-l] Quadrados mágicos: problema da Eureka 0 1:

2010-10-24 Por tôpico Johann Dirichlet
O unico pre-requisito para se ler uma Eureka! e ler as anteriores.
Desculpe falar algo tao obvio, mas e que nao tem bem um pre-Eureka! no
Brasil, ate onde eu sei. Se voce encara uma leitura em ingles, a
melhor referencia que conheco e o site mathlinks.ro. La tem tutoriais
e artigos de todos os niveis. Tambem tem um arquivo de problemas de
olimpiadas de todo o mundo.
Por ora, esta e minha recomendacao: o site da OBM e as Eureka!s, todas elas.

Bem, eu pretendo lancar um arquivo pessoal contendo solucoes da
Eureka! e de alguns problemas que eu fiz ha milenios em papel, mas
ainda não posso garantir nada...

Quanto ao problema, tente resolver o sistema de equacoes gerado pelas
somas, e voce descobrira que o numero central e igual a 5. Depois eu
posto algo competo.

Em 23/10/10, Rafaelapolo_hiperbo...@terra.com.br escreveu:
 Olá, pessoal.

 Antes de comentar sobre um problema da Eureka 01, uma pergunta:

 Alguém aqui costuma resolver todos os problemas sem solução da revista
 Eureka e deixar em um arquivo no word, por exemplo ? Se sim, gostaria muito
 de um arquivo com esses problemas que contém apenas o gabarito e não a
 solução.

 Uma vez enviaram aqui um arquivo com questões resolvidas do IME, inclusive
 bem antigas. Alguém aqui tem arquivos de questões resolvidas assim também,
 mas olímpicas ? Seja da Eureka ou não. No site Excalibur, há muitos
 problemas assim, mas o nível é bem alto. Gostaria de um arquivo com
 problemas resolvidos de forma preparatória à leitura das Eurekas.

 Comecei a ler as Eurekas. Veja este problema da Eureka 01:

 Você já conhece o quadrado mágico de ordem 3: a soma dos números das linhas,
 das colunas e das diagonais é 15. A figura a seguir mostra uma das oito
 possibilidades de escrever os números no quadrado:

 a11 = 8; a12 = 1; a13 = 6
 a21 = 3; a22 = 5; a23 = 7
 a31 = 4; a32 = 9; a33 = 2

 O único número que não pode mudar de posição em todos esses quadrados
 mágicos é:



 a) 1  b) 3  c) 5  d) 7  e) 9


 Eu percebi que a correta é a C (gabarito), pois se girarmos o quadrado no
 sentido horário ou anti-horário, teremos 4 quadrados (incluindo o original)
 e em todos eles não houve mudança do número 5 em relação ao quadrado do
 enunciado. Eu gostaria de uma solução mais formal e por que são 8
 possibilidades e não 4.

 Obs1: Saber todo o conteúdo do ensino médio já é o suficiente para ler e
 entender as Revistas Eurekas OU deve haver uma outra condição prévia, como
 ler algum livro específico ou estudar por problemas de outros sites ?
 Pergunto isso, pois abri aleatoriamente algumas revistas e li alguns termos
 matemáticos não abordados em livros normais do ensino médio, daí pensei: -
 OU os elaboradores das Eureka estão partindo do pressuposto que os leitores
 já saibam determinadas coisas (mesmo que não estejam em livros regulares do
 ensino médio); OU há uma gradação de conhecimentos nas revistas, ou seja, se
 não entendeu algum termo ou conceito OLÍMPICOS em alguma revista, então é
 provável que haja um explicação em alguma das revistas anteriores.


 Regards,
 Rafael


-- 
/**/
Quadrinista e Taverneiro!

http://tavernadofimdomundo.blogspot.com  Quadrinhos, histórioas e afins
http://baratoeletrico.blogspot.com / Um pouco sobre elétrons em movimento
http://bridget-torres.blogspot.com/  Personal! Do not edit!

=
Instru��es para entrar na lista, sair da lista e usar a lista em
http://www.mat.puc-rio.br/~obmlistas/obm-l.html
=


[obm-l] Re: [obm-l] Parte fracionária = parte decimal?

2010-10-24 Por tôpico Johann Dirichlet
Com certeza! (eu acho...)

Por definicao, a parte inteira de um real e o maior inteiro que fica
abaixo deste real. Por exemplo, 7 e a parte inteira de 7,1234.

A parte fracionaria e esta diferenca entre o numero e sua parte
inteira. No caso, 0,1234.

O lance e que as vezes voce tem um numero feio. Por exemplo,
pi=3,141592..., e e mais pratico escrever pi-3 do que 0,141592...

Um exemplo: qual a parte inteira de  -3,1415?

Em 22/10/10, Guilherme Vieirarjguilhermevie...@hotmail.com escreveu:

 Caros Colegas,

 Dado o número decimal 7,1234, pode-se dizer que sua parte fracionária é
 0,1234?  Isto é, a parte fracionária é a parte decimal?

 Um abração!
 Guilherme

   


-- 
/**/
Quadrinista e Taverneiro!

http://tavernadofimdomundo.blogspot.com  Quadrinhos, histórioas e afins
http://baratoeletrico.blogspot.com / Um pouco sobre elétrons em movimento
http://bridget-torres.blogspot.com/  Personal! Do not edit!

=
Instru��es para entrar na lista, sair da lista e usar a lista em
http://www.mat.puc-rio.br/~obmlistas/obm-l.html
=


Re: [obm-l] Algoritmo de Euclides estendido

2010-10-20 Por tôpico Johann Dirichlet
Suponha que p é divisor de ab, mas não seja de a.
Então a e p serão primos entre si, e assim podemos achar x e y tais que
xa+yp=1
Multiplicando por b, temos
xab+ybp=b
Como xab e ybp são múltiplos de p, a soma também será. É isso!


Em 15/10/10, luizluizvalve...@globo.com escreveu:

 Alguem pode me ajudar.?





 O algoritmo de Euclides estendido é o seguinte:

 Dados a e b inteiros, seja d = mdc(a,b) então existem r e s inteiros tais
 que sa+rb=d.

 Usando o algoritmo de Euclides estendido mostre que se p é primo e a e b são
 inteiros tais que p é divisor de ab, então p é divisor de a ou p é divisor
 de b.



-- 
/**/
Quadrinista e Taverneiro!

http://tavernadofimdomundo.blogspot.com  Quadrinhos, histórioas e afins
http://baratoeletrico.blogspot.com / Um pouco sobre elétrons em movimento
http://bridget-torres.blogspot.com/  Personal! Do not edit!

=
Instru��es para entrar na lista, sair da lista e usar a lista em
http://www.mat.puc-rio.br/~obmlistas/obm-l.html
=


Re: [obm-l] Semelhantes ou iguais?

2010-10-07 Por tôpico Johann Dirichlet
Pense no mapa de uma cidade. Os  Ãngulos são iguais, mas as distâncias não.

Em 07/10/10, Nathália Santosnathalia...@hotmail.com escreveu:
 Serão sempre semelhantes, mas não necessariamente iguais, já que ângulos
 iguais não determinam sempre lados iguais.

 From: rhilbert1...@hotmail.com
 To: obm-l@mat.puc-rio.br
 Subject: [obm-l] Semelhantes ou iguais?
 Date: Wed, 6 Oct 2010 22:25:19 +





 Colegas, uma discussão sem solução, acontenceu por conta da seguinte dúvida.

 Dois triângulos com os seus ângulos, respectivamente, de mesma medida, são
 iguais (lados  respectivos de mesma medida) ou semelhantes (lados
 respectivos proporcionais)?

 Exemplo: Triângulos ABC e A'B'C'  com   ângulos A=A, B=B e C=C  = AB=A'B',
 AC=A'C' e BC=B'C' ?










   
   


-- 
/**/
Quadrinista e Taverneiro!

http://tavernadofimdomundo.blogspot.com  Quadrinhos, histórioas e afins
http://baratoeletrico.blogspot.com / Um pouco sobre elétrons em movimento
http://bridget-torres.blogspot.com/  Personal! Do not edit!

=
Instru��es para entrar na lista, sair da lista e usar a lista em
http://www.mat.puc-rio.br/~obmlistas/obm-l.html
=


Re: [obm-l] REPASSE COM URGENCIA, URGENCIA

2010-10-05 Por tôpico Johann Dirichlet
Legal, agora temos spam político na OBM-L...

Em 05/10/10, Luís Juniorjrcarped...@gmail.com escreveu:
 Concordo com todos os projetos.

 2010/10/5 Aline Rosane aline.ace...@hotmail.com




  *PENSE BEM ANTES DE VOTAR NA DONA DILMA E SUA GANG !!!
 *



 Cordialmente,
 Camargo Júnior
 69-3421-3061



 --
 *Resumo de projetos que podem virar lei após as eleição: ENTÃO VOTE NA
 DILMA*

 **

 *Fica proibido fazer:*
 ·   Cultos ou evangelismo na rua (Reforma Constitucional)
 ·   Programas evangélicos na televisão por mais de uma hora por dia.
 ·   Programa de rádio ou televisão, quem não possuir faculdade
 de'jornalismo'.
 ·   Pregar sobre dízimos e ofertas, havendo reclamações, obreiros serão
 presos.

 *Quanto aos cultos:*
 - Cultos somente com portas fechadas (Reforma Constitucional)
 - As igrejas serão obrigadas a pagarem impostos sobre dízimos,
 ofertas e contribuições,as católicas também.
 - Será considerado crime pregar sobre espiritismo, feitiçaria e idolatria,
 e também veicular mensagem no rádio, televisão, jornais e internet,
 sobre essas práticas contrárias a Palavra de Deus.
 - Pastores e Padres que forem presos por pregar sobre práticas condenadas
 pela
 Bíblia Sagrada (homossexualismo, idolatria e espiritismo), não terão
 direito a se defender por meio de ação judicial.

 *Se estabeleça:*
 -O dia do “Orgulho Gay” e que seja oficializado em todas as cidades
 brasileiras e comemorado nas Instituições de Ensino Fundamental
 (primeira a 8.a série), público e particular.
 -Que as Igrejas que se negarem a realização das solenidades
 dos casamentos de homem com homem e de mulher com mulher,
 estarão fazendo “discriminação”, seja multadas e seus pastores e padres
 processados criminalmente por discriminação e desobediência civil.


 *Projeto nº 4.720/03 - Altera a legislação constitucional** **

 -Projeto nº 3.331/04 –* Altera o artigo 12 da Lei nº 9.250/95, que trata
 da legislação do imposto de renda das 'pessoas físicas' Se convertidos em
 Lei, os dois projetos obrigariam as igrejas a recolherem impostos sobre
 dízimos, ofertas e contribuições.


 *-**Projeto nº 299/99 –* Altera o código brasileiro de telecomunicações
 (Lei 4.117/62).
 Se aprovado, reduziria programas evangélicos
 no rádio e televisão a apenas uma hora.
 **

 *-Projeto nº 6.398/05 –* Regulamenta a profissão de Jornalista
 Contém artigos que estabelecem que só poderão fazer programas
 de rádio e televisão, pessoas com formação em JORNALISMO, Significa que
 pastores,padres e ouros sem a formação em jornalismo não
 poderão fazer programas através desses meios.


 *-Projeto nº 1.154/03 –* Proíbe veiculação de programas  em que o teor
 seja considerado preconceito religioso. Se aprovado, será considerado
 crime
 pregar sobre idolatria, feitiçaria e rituais satânicos. Será proibido que
 mensagens sobre essas práticas sejam veiculadas no rádio, televisão,
 jornais
 e internet. A verdade sobre esses atos contrários
 a Palavra de Deus, não poderá mais ser mostrada.


 *-Projeto nº 952/03 – *Estabelece que é crime atos religiosos que possam
 ser considerados abusivos a boa-fé das pessoas. Convertido em Lei, pelo
 número de reclamações, pastores serão considerados 'criminosos' por
 pregarem
 sobre dízimos e ofertas.


 *-Projeto nº 4.270/04[/b] –** *Determina que comentários feitos contra
 ações praticadas por grupos religiosos possam ser passíveis de ação civil.
 Se convertido em Lei, as Igrejas Evangélicas ficariam proibidas de pregar
 sobre práticas condenadas pela Bíblia Sagrada, como espiritismo,
 feitiçaria,
 idolatria e outras. Se o fizerem, não terão direito a se defender por meio
 de ação judicial.


 *-**Projeto de nº 216/04[/b] –* Torna inelegível a função religiosa com a
 governamental. Significa que todo pastor ou líder religioso lançado a
 candidaturas para qualquer cargo político, não poderá de forma alguma
 exercer trabalhos na igreja.





-- 
/**/
Quadrinista e Taverneiro!

http://tavernadofimdomundo.blogspot.com  Quadrinhos, histórioas e afins
http://baratoeletrico.blogspot.com / Um pouco sobre elétrons em movimento
http://bridget-torres.blogspot.com/  Personal! Do not edit!

=
Instru��es para entrar na lista, sair da lista e usar a lista em
http://www.mat.puc-rio.br/~obmlistas/obm-l.html
=


[obm-l] Spam político e terrorismo, aqui não!! (era: REPAS SE COM URGENCIA, URGENCIA)

2010-10-05 Por tôpico Johann Dirichlet
Olá pessoas!
Bem, não sei se estou sendo redundandte ou mesmo se o que eu direi irá
ofender a inteligência dos presentes desta lista:
Pesquise as fontes sempre que surgir um e-mail ou mensagem destas por
esta lista (e de qualquer outro meio de comunicação). Como diziam na
Internet, O Google é seu amigo.

Enfim, é isso. Tem algum problema bom de Matemática aí?

Em 05/10/10, Saon Crispim Vieirasao...@gmail.com escreveu:
 Olá pessoal, boa tarde.
 Isto é pura fantasia! É inaceitável ouvir tais acusações sem indícios e/ou
 provas!
 Sou espírita e não eleitor da Dilma, mas este não é o nível de debate que
 espero desta lista.
 Segue anexa uma matéria interessante, de uma fonte confiável e com tema mais
 relacionado à proposta desta lista.

 http://www.nature.com/news/2010/100929/full/467511b.html?s=news_rss

 2010/10/5 Aline Rosane aline.ace...@hotmail.com




  *PENSE BEM ANTES DE VOTAR NA DONA DILMA E SUA GANG !!!
 *



 Cordialmente,
 Camargo Júnior
 69-3421-3061



 --
 *Resumo de projetos que podem virar lei após as eleição: ENTÃO VOTE NA
 DILMA*

 **

 *Fica proibido fazer:*
 ·   Cultos ou evangelismo na rua (Reforma Constitucional)
 ·   Programas evangélicos na televisão por mais de uma hora por dia.
 ·   Programa de rádio ou televisão, quem não possuir faculdade
 de'jornalismo'.
 ·   Pregar sobre dízimos e ofertas, havendo reclamações, obreiros serão
 presos.

 *Quanto aos cultos:*
 - Cultos somente com portas fechadas (Reforma Constitucional)
 - As igrejas serão obrigadas a pagarem impostos sobre dízimos,
 ofertas e contribuições,as católicas também.
 - Será considerado crime pregar sobre espiritismo, feitiçaria e idolatria,
 e também veicular mensagem no rádio, televisão, jornais e internet,
 sobre essas práticas contrárias a Palavra de Deus.
 - Pastores e Padres que forem presos por pregar sobre práticas condenadas
 pela
 Bíblia Sagrada (homossexualismo, idolatria e espiritismo), não terão
 direito a se defender por meio de ação judicial.

 *Se estabeleça:*
 -O dia do “Orgulho Gay” e que seja oficializado em todas as cidades
 brasileiras e comemorado nas Instituições de Ensino Fundamental
 (primeira a 8.a série), público e particular.
 -Que as Igrejas que se negarem a realização das solenidades
 dos casamentos de homem com homem e de mulher com mulher,
 estarão fazendo “discriminação”, seja multadas e seus pastores e padres
 processados criminalmente por discriminação e desobediência civil.


 *Projeto nº 4.720/03 - Altera a legislação constitucional** **

 -Projeto nº 3.331/04 –* Altera o artigo 12 da Lei nº 9.250/95, que trata
 da legislação do imposto de renda das 'pessoas físicas' Se convertidos em
 Lei, os dois projetos obrigariam as igrejas a recolherem impostos sobre
 dízimos, ofertas e contribuições.


 *-**Projeto nº 299/99 –* Altera o código brasileiro de telecomunicações
 (Lei 4.117/62).
 Se aprovado, reduziria programas evangélicos
 no rádio e televisão a apenas uma hora.
 **

 *-Projeto nº 6.398/05 –* Regulamenta a profissão de Jornalista
 Contém artigos que estabelecem que só poderão fazer programas
 de rádio e televisão, pessoas com formação em JORNALISMO, Significa que
 pastores,padres e ouros sem a formação em jornalismo não
 poderão fazer programas através desses meios.


 *-Projeto nº 1.154/03 –* Proíbe veiculação de programas  em que o teor
 seja considerado preconceito religioso. Se aprovado, será considerado
 crime
 pregar sobre idolatria, feitiçaria e rituais satânicos. Será proibido que
 mensagens sobre essas práticas sejam veiculadas no rádio, televisão,
 jornais
 e internet. A verdade sobre esses atos contrários
 a Palavra de Deus, não poderá mais ser mostrada.


 *-Projeto nº 952/03 – *Estabelece que é crime atos religiosos que possam
 ser considerados abusivos a boa-fé das pessoas. Convertido em Lei, pelo
 número de reclamações, pastores serão considerados 'criminosos' por
 pregarem
 sobre dízimos e ofertas.


 *-Projeto nº 4.270/04[/b] –** *Determina que comentários feitos contra
 ações praticadas por grupos religiosos possam ser passíveis de ação civil.
 Se convertido em Lei, as Igrejas Evangélicas ficariam proibidas de pregar
 sobre práticas condenadas pela Bíblia Sagrada, como espiritismo,
 feitiçaria,
 idolatria e outras. Se o fizerem, não terão direito a se defender por meio
 de ação judicial.


 *-**Projeto de nº 216/04[/b] –* Torna inelegível a função religiosa com a
 governamental. Significa que todo pastor ou líder religioso lançado a
 candidaturas para qualquer cargo político, não poderá de forma alguma
 exercer trabalhos na igreja.




 --
 Saon Crispim Vieira



-- 
/**/
Quadrinista e Taverneiro!

http://tavernadofimdomundo.blogspot.com  Quadrinhos, histórioas e afins
http://baratoeletrico.blogspot.com / Um pouco sobre elétrons em movimento
http://bridget-torres.blogspot.com/  Personal! Do not edit!

=
Instru��es para entrar 

[obm-l] Re: [obm-l] Teoria dos números

2010-09-24 Por tôpico Johann Dirichlet
1)
Basta demonstrar que (n^8+1)(n^8-1) é múltiplo de 17.
Mais isso sai direto de Euler-Fermat: 17 divide n^16-1 se n não é
múltiplo de 17.

2)

(2y+1)^2-4=x^3
Escrevendo z=2y-1:
(z-2)(z+2)=x^3
Veja que z-2 e z+2 não tem fatores comuns (ambos são ímpares
consecutivos), logo ambos são cubos perfeitos. Daqui fica fácil
seguir...

Em 23/09/10, warley ferreiralulu...@yahoo.com.br escreveu:
 Pessoal seria possível ajuda nestas questoes,

 Questão 1)
 Seja n um número natural não divisível por 17. Prove que n^8 + 1 ou n^8 -1 é
 divisível por 17.
 Questão 2)
 Determine, caso existam, as soluções inteiras da equação x^3+3 = 4y(y+1).

 Desde já agradeço,
  Warley Souza





-- 
/**/
Quadrinista e Taverneiro!

http://tavernadofimdomundo.blogspot.com  Quadrinhos, histórioas e afins
http://baratoeletrico.blogspot.com / Um pouco sobre elétrons em movimento
http://bridget-torres.blogspot.com/  Personal! Do not edit!

=
Instru��es para entrar na lista, sair da lista e usar a lista em
http://www.mat.puc-rio.br/~obmlistas/obm-l.html
=


Re: [obm-l] Para mim, 0^0=1

2010-09-22 Por tôpico Johann Dirichlet
Nessas horas eu me pergunto: por que existem tantas arestas
não-aparadas na matemática?

A aresta mais pontuda, na minha opinião, é o paradoxo de
Banach-Tarski: é possível desmontar uma bolinha de gude e juntar os
pedaços de modo a se obter uma bola do tamanho do sol.

Em 16/09/10, Ralph Teixeiraralp...@gmail.com escreveu:
 Eu sou um dos defensores de 0^0=1. Apresento dois motivos:

 i) Se f(x) e g(x) sao analiticas em 0 com f(x),g(x)-0 quando x-a, entao
 f^g - 1 quando x- a (bom, desde que f^g faca sentido em volta de x=a). A
 *unica* excecao a esta regra eh o caso em que f eh identicamente nula,
 quando o limite dah 0 (se f^g faz sentido) ou nao existe (se g0 ali por
 perto de x=a).
 Isto explica porque 99.9% dos exercicios de limite que ficam da forma 0^0
 acabam dando 1 como resposta!

 Acho que isto tambem explica porque eu nao faria 0/0=1 ou algo assim -- nao
 ha teorema semelhante para 0/0.

 ii) Como escrever um polinomio generico de grau 17 usando somatorios? Acho
 que muita gente concorda que uma boa representacao eh:
 p(x) = SUM (n=0 a 17) a_n x^n
 onde os a_n sao coeficientes arbitrarios. Agora eu pergunto -- quanto vale
 p(0)?

 Com a convencao 0^0=1, nada especial precisa ser feito, eh soh substituir
 x=0 no somatorio.

 Com a convencao 0^0 nao existe bom, ai a nossa representacao por
 somatorio ficaria tecnicamente errada. Teriamos que escrever:

 p(x) = SUM (n=0 a 17) a_n x^n, se x0
 p(x) = a_0, se x=0

 ou entao tirar o x^0 do somatorio:

 p(x) = a_0 + SUM (n=1 a 17) a_n x^n

 (e se voce acha que esta ultima eh bem razoavel -- escreva p'(x). Separou o
 a1? Argh!)

 Como eu nao tenho paciencia de ficar escrevendo este a_0 separado toda hora,
 prefiro logo pensar que 0^0=1 e resolvo meus problemas com um somatorio soh.
 :)

 Isto tudo dito, claro que eh soh uma convencao, questao de gosto. Mas eu
 *gosto* de 0^0=1. :)

 Abraco,
  Ralph

 2010/9/16 Jorge Luis Rodrigues e Silva Luis jorgelrs1...@hotmail.com

 Olá, Pessoal! Vale lembrar que o símbolo do nada está entre as mais
 importantes descobertas feita pelo homem. É difícil acreditar que os
 homens
 levaram 5 mil anos entre escrever números e conceber o nosso sistema de
 numeração posicional, ponto crucial num desenvolvimento sem o qual o
 progresso da ciência moderna seria inconcebível. Hoje parece simples, mas
 a
 mentalidade concreta dos antigos gregos, não podia conceber o vazio, o
 nada,
 como um número. Apreciaremos ainda mais a grandeza dessa conquista se
 lembrarmo-nos de que ela escapou ao gênio de Arquimedes e Apolônio, dois
 dos
 maiores homens da antiguidade.

 Existem situações em Análise Combinatória onde há uma certa conveniência
 em
 adotar a regra 0^0=1, a fim de estender um pouco mais o campo de validez
 de
 algumas fórmulas. Nem por isso 0^0 deixa de ser uma expressão
 indeterminada.
 Um caso parecido acontece na Teoria da Medida e da Integral, onde às vezes
 é
 conveniente escrever 0*...=0, a fim de que a fórmula da área de um
 retângulo
 continue válida quando a base do retângulo é toda uma reta e a altura se
 reduz a um ponto. O curioso é que os defensores de 0^0=1 não reivindiquem
 o
 mesmo direito para 0/0. Algum colega saberia o motivo?

 Afinal! Qual das medidas é a mais precisa? E a mais exata?  a)5,6m
 b)560m
 (com aproximação de 10m)   c) 0,056m   d)5600m (com aproximação de 100m)

 Quantos algarismos significativos temos nesta medida? X=(0,009050 + -
 0,02)

 A propósito! Como se escreve zero em algarismos romanos?


 Abraços!




-- 
/**/
Quadrinista e Taverneiro!

http://tavernadofimdomundo.blogspot.com  Quadrinhos, histórioas e afins
http://baratoeletrico.blogspot.com / Um pouco sobre elétrons em movimento
http://bridget-torres.blogspot.com/  Personal! Do not edit!

=
Instru��es para entrar na lista, sair da lista e usar a lista em
http://www.mat.puc-rio.br/~obmlistas/obm-l.html
=


Re: [obm-l] Qual a melhor mailing list internacional de Mathematics

2010-09-20 Por tôpico Johann Dirichlet
Mailing list eu não sei, mas se você aceita um fórum, tem o www.mathlinks.ro.

Em 19/09/10, Rafaelapolo_hiperbo...@terra.com.br escreveu:
 Olá, pessoal.


 Qual a melhor mailing list internacional de Matemática ?



 Regards,
 Rafael


-- 
/**/
Quadrinista e Taverneiro!

http://tavernadofimdomundo.blogspot.com  Quadrinhos, histórioas e afins
http://baratoeletrico.blogspot.com / Um pouco sobre elétrons em movimento
http://bridget-torres.blogspot.com/  Personal! Do not edit!

=
Instru��es para entrar na lista, sair da lista e usar a lista em
http://www.mat.puc-rio.br/~obmlistas/obm-l.html
=


[obm-l] Re: [obm-l] Re: [obm-l] Re: [obm-l] RE: [obm-l] Fato rial via Stirling (confirmação)

2010-09-20 Por tôpico Johann Dirichlet
Em 18/09/10, Bernardo Freitas Paulo da Costabernardo...@gmail.com escreveu:
 2010/9/17 Johann Dirichlet peterdirich...@gmail.com:
 Bem, vou azedar um pouco a coisa: que tal se pudéssemos isolar o r?
 n! = [(2.n.pi)^(1/2)].[(n/e)^n].(e^r) se e somente se
 n!/((2.n.pi)^(1/2).(n/e)^n)=(e^r)
 Passa o log, temos uma expressão em r.
 Se pudermos provar a existência deste monstrinho, fechou

 Eu acho que a fórmula de Euler-MacLaurin é realmente o que é mais
 adaptado para provar esse tipo de horror (expansão assintótica de
 somas finitas, quando a gente passa aos logs). Tem que estudar, mas
 enfim, você não pode querer demonstrar tudo a partir de nada: a
 matemática se constrói passo a passo...

 Enfim, esta observação chata é mais porque, de memória, obter o raiz
 de 2*pi na fórmula do fatorial é bem difícil. Se você dispensar
 essa exatidão toda, acho que até dá, inclusive por indução (Johann: já
 achou como corrigir a tua?). Daí, a fórmula fica
 n! = (n/e)^n*raiz(n) * erro(n)

Na verdade eu nem tentei :)
Creio que você esteja certo no erro da fórmula. No fim das contas
essa constante é difícil de se obter por indução. A bem da verdade não
conheço nenhum problema de limites que use indução.


 onde 0  min  erro(n)  MAX para duas constantes min e MAX (que a
 gente não calculou)

 Em 17/09/10, Guilherme Vieirarjguilhermevie...@hotmail.com escreveu:

 Caro Paulo,
 Continuo pensando que não há possibilidade de se obter demonstração por
 indução finita, pois r depende de n.
 Não sei se há outro modo de confirmar a validade da fórmula.
 Continuemos tentando!
 Um abraço do Guilherme!



 From: argolopa...@hotmail.com
 To: obm-l@mat.puc-rio.br
 Subject: [obm-l] Fatorial via Stirling (confirmação)
 Date: Thu, 16 Sep 2010 20:55:27 +





 Caros amigos,
 Repito a questão a que propus.
 Não sei se as respostas já dadas tratam efetivamente da mesma questão.
 Fiquei em dúvida.

 Gostaria de obter uma demonstração (pode ser por indução finita) do fato
 abaixo, proveniente da fórmula de Stirling.

 Fato:
 Para todo número inteiro positivo n, existe um número real r, com
 1/(12n+1)
  r
  1/(12n), de modo que seja válida a igualdade:
 n! = [(2.n.pi)^(1/2)].[(n/e)^n].(e^r)

 Muito obrigado!
 Paulo Argolo










 --
 /**/
 Quadrinista e Taverneiro!

 http://tavernadofimdomundo.blogspot.com  Quadrinhos, histórioas e afins
 http://baratoeletrico.blogspot.com / Um pouco sobre elétrons em
 movimento
 http://bridget-torres.blogspot.com/  Personal! Do not edit!

 =
 Instru�ões para entrar na lista, sair da lista e usar a lista em
 http://www.mat.puc-rio.br/~obmlistas/obm-l.html
 =




 --
 Bernardo Freitas Paulo da Costa

 =
 Instru�ões para entrar na lista, sair da lista e usar a lista em
 http://www.mat.puc-rio.br/~obmlistas/obm-l.html
 =



-- 
/**/
Quadrinista e Taverneiro!

http://tavernadofimdomundo.blogspot.com  Quadrinhos, histórioas e afins
http://baratoeletrico.blogspot.com / Um pouco sobre elétrons em movimento
http://bridget-torres.blogspot.com/  Personal! Do not edit!

=
Instru��es para entrar na lista, sair da lista e usar a lista em
http://www.mat.puc-rio.br/~obmlistas/obm-l.html
=


[obm-l] Re: [obm-l] RE: [obm-l] Fatorial via Stirling (confi rmação)

2010-09-17 Por tôpico Johann Dirichlet
Bem, vou azedar um pouco a coisa: que tal se pudéssemos isolar o r?
n! = [(2.n.pi)^(1/2)].[(n/e)^n].(e^r) se e somente se
n!/((2.n.pi)^(1/2).(n/e)^n)=(e^r)
Passa o log, temos uma expressão em r.
Se pudermos provar a existência deste monstrinho, fechou

Em 17/09/10, Guilherme Vieirarjguilhermevie...@hotmail.com escreveu:

 Caro Paulo,
 Continuo pensando que não há possibilidade de se obter demonstração por
 indução finita, pois r depende de n.
 Não sei se há outro modo de confirmar a validade da fórmula.
 Continuemos tentando!
 Um abraço do Guilherme!



 From: argolopa...@hotmail.com
 To: obm-l@mat.puc-rio.br
 Subject: [obm-l] Fatorial via Stirling (confirmação)
 Date: Thu, 16 Sep 2010 20:55:27 +





 Caros amigos,
 Repito a questão a que propus.
 Não sei se as respostas já dadas tratam efetivamente da mesma questão.
 Fiquei em dúvida.

 Gostaria de obter uma demonstração (pode ser por indução finita) do fato
 abaixo, proveniente da fórmula de Stirling.

 Fato:
 Para todo número inteiro positivo n, existe um número real r, com 1/(12n+1)
  r
  1/(12n), de modo que seja válida a igualdade:
 n! = [(2.n.pi)^(1/2)].[(n/e)^n].(e^r)

 Muito obrigado!
 Paulo Argolo







   


-- 
/**/
Quadrinista e Taverneiro!

http://tavernadofimdomundo.blogspot.com  Quadrinhos, histórioas e afins
http://baratoeletrico.blogspot.com / Um pouco sobre elétrons em movimento
http://bridget-torres.blogspot.com/  Personal! Do not edit!

=
Instru��es para entrar na lista, sair da lista e usar a lista em
http://www.mat.puc-rio.br/~obmlistas/obm-l.html
=


Re: [obm-l] ajuda

2010-09-15 Por tôpico Johann Dirichlet
Em 14/09/10, Bernardo Freitas Paulo da Costabernardo...@gmail.com escreveu:
 2010/9/14 Johann Dirichlet peterdirich...@gmail.com:
 Não é nenhuma das coisas.O zero é uma espécie de múltiplo universal:
 todo número é múlrtiplo de zero.
 Cuidado, Johann! Além de escrever quase escrever múrtiplo, nenhum
 número, exceto zero, é múltiplo de zero. Ele é realmente um múltiplo
 universal, logo é também divisível por todo número inteiro.


Bem lembrado! Eu sempre esqueço o que é o que: se a | b então a é
divisor de b e b é múltiplo de a.

 Um numero, para ser primo, não pode ser escrito como o produto de dois
 fatores maiores que 1.
 Já um composto é, necessariamente, um produto de dois ou mais naturais
 menores que ele (e maiores que 1).

 O zero e o um caem fora destes dois casos.
 Em geral, você exclui o zero da decomposição porque ele não faz parte
 do grupo multiplicativo (ele não tem inverso). Só tem sentido falar de
 composto e indecomponível (a primeira noção de primo, lá dos
 gregos) dentro de um grupo. Já o um não é primo porque é inversível, e
 a gente também exclui os inversíveis das definições porque isso
 complicaria muito a decomposição (fatoração) única.

 Enfim, dica pro Alberto e todos mais: o importante de entender as
 definições é ver porque elas foram escolhidas assim. Uma resposta boa
 (para mim) é que neste caso a gente tem um enunciado bem simples do
 teorema de decomposição dos números inteiros:
  Todo número inteiro composto n possui uma única decomposição em
 fatores primos, a menos de
 - mudança na ordem dos fatores
 - multiplicações por inversíveis

Esse é semelhante a uma definição do artigo do Guilherme Issao sobre
inteiros de Eisenstein: fatoração única a menos da ordem e a menos de
multiplicação por unidades.


 Note que eu excluí os primos, os irredutíveis (e o zero) do teorema
 porque para eles o resultado é imediato das definições, portanto não é
 tão interessante como o que vale para os compostos.

 Note que esse teorema fica mais simples ainda (mas perdemos um pouco a
 abstração que vale em vários casos) se falarmos de naturais: Todo
 inteiro estritamente positivo pode ser escrito de forma única como
 produto de primos positivos em ordem crescente

Melhor escrever não-decrescente (para deixar mais claro que pode
haver repetições).


 Abraços
 --
 Bernardo Freitas Paulo da Costa

 P.S.: estamos falando dos naturais. Existem estruturas matemáticas nas
 quais o produto de dois não-nulos pode ser nulo.


 Em 14/09/10, Adalberto Dornellesaadornell...@gmail.com escreveu:
 Olá turma,

 Perguntinha rápida:

 O zero é primo? é composto? ou nem uma coisa nem outra?

 Tenho 98,6544% de certeza que a resposta é nem uma coisa nem outra, mas
 ...

 Abraço,
 Adalberto

 =
 Instruções para entrar na lista, sair da lista e usar a lista em
 http://www.mat.puc-rio.br/~obmlistas/obm-l.html
 =



-- 
/**/
Quadrinista e Taverneiro!

http://tavernadofimdomundo.blogspot.com  Quadrinhos, histórioas e afins
http://baratoeletrico.blogspot.com / Um pouco sobre elétrons em movimento
http://bridget-torres.blogspot.com/  Personal! Do not edit!

=
Instru��es para entrar na lista, sair da lista e usar a lista em
http://www.mat.puc-rio.br/~obmlistas/obm-l.html
=


Re: [obm-l] ajuda

2010-09-14 Por tôpico Johann Dirichlet
Não é nenhuma das coisas.O zero é uma espécie de múltiplo universal:
todo número é múlrtiplo de zero.

Um numero, para ser primo, não pode ser escrito como o produto de dois
fatores maiores que 1.
Já um composto é, necessariamente, um produto de dois ou mais naturais
menores que ele (e maiores que 1).

O zero e o um caem fora destes dois casos.

P.S.: estamos falando dos naturais. Existem estruturas matemáticas nas
quais o produto de dois não-nulos pode ser nulo.


Em 14/09/10, Adalberto Dornellesaadornell...@gmail.com escreveu:
 Olá turma,

 Perguntinha rápida:

 O zero é primo? é composto? ou nem uma coisa nem outra?

 Tenho 98,6544% de certeza que a resposta é nem uma coisa nem outra, mas
 ...

 Abraço,
 Adalberto

 =
 Instruções para entrar na lista, sair da lista e usar a lista em
 http://www.mat.puc-rio.br/~obmlistas/obm-l.html
 =



-- 
/**/
Quadrinista e Taverneiro!

http://tavernadofimdomundo.blogspot.com  Quadrinhos, histórioas e afins
http://baratoeletrico.blogspot.com / Um pouco sobre elétrons em movimento
http://bridget-torres.blogspot.com/  Personal! Do not edit!

=
Instru��es para entrar na lista, sair da lista e usar a lista em
http://www.mat.puc-rio.br/~obmlistas/obm-l.html
=


[obm-l] Re: [obm-l] Equação algébrica

2010-09-13 Por tôpico Johann Dirichlet
A maneira que me vem à cabeça é usar o teorema do valor intermediario.

Podemos fazer algumas suposições:
|r|  1. De fato, se |r|1, troque r por R=1/r e x por X=1/x. Assim,
teremos X^n=R, com |R|1, e resolver essa equacao é equivalente
resolver a original.

Caso n ímpar:
Se r  0, podemos trocar x por -x e r por -r. Vamops entao supor r1.

Enfim, existe um valor de x tal que x^n-r0. Isso e relativamente
facil de demonstrar usando limites ou algo que valha.
Igualmente, existe outro valor de x tal que x^n-r0.

Pelo teorema do valor intermediario, existe um cara entre estes dois
extremos tal que x^n=r=0.

O caso par fica por sua conta :)


Em 11/09/10, Guilherme Vieirarjguilhermevie...@hotmail.com escreveu:

 Solicito aos amigos uma demonstração do teorema enunciado a seguir.
 Obviamente, a propriedade é muito conhecida. A demonstração, entretanto,
 parece-me muito difícil.

 Teorema: Se x é uma variável real, n é um número natural (não nulo) e r é
 uma constante real, a equação algébrica x^n = r admite uma única solução
 real quando n é ímpar e admite duas soluções reais quando n é par e r0.


 Obrigado!!!
 Guilherme 


-- 
/**/
Quadrinista e Taverneiro!

http://tavernadofimdomundo.blogspot.com  Quadrinhos, histórioas e afins
http://baratoeletrico.blogspot.com / Um pouco sobre elétrons em movimento
http://bridget-torres.blogspot.com/  Personal! Do not edit!

=
Instru��es para entrar na lista, sair da lista e usar a lista em
http://www.mat.puc-rio.br/~obmlistas/obm-l.html
=


[obm-l] Re: Propriedades dos divisores

2010-09-03 Por tôpico Johann Dirichlet
Ué, mas ela já está aí!

Se você quer algo com linguagem chata, tá bom:

Seja D(N) o conjunto dos divisores de N

1) d pertence a D(N)
acarreta
N/d pertence a D(N)

2) d1d2 acarreta N/d1  N/d2

3) Sejam os conjuntos D(N,d) = {d,N/d}.
Para cada d pertencente a D(N), o conjunto D(N,d) está contido em D(N).
Ademais, D(N,d1)=D(N,d2) se e so se d1=d2 ou d1=N/d2.
Além disso, se d=N/d (ou d^2=N) o conjunto D(N,d) tem somente um elemento.
De quebra, o produto dos elementos de cada D(N,d) é N (excetuando o caso d^2=N).

Portanto, os conjuntos D(N,d), com d variando entre todos os divisores
de N menores que a raiz quadrada de N, particionam o conjunto D(N) (em
outras palavras, são dois a dois disjuntos e a uniao deles é D(n)).

Agora a parte lúdica:
Se colocarmos os divisores de N em uma fila indiana, e a cada um
colocarmos uma etiqueta eu sou elemento do conjunto D(N,d) (em que
escolhemos o d tal que d^2 = N), veremos que os números das pontas
terao etiquetas iguais, e o produto de ambos será N. É justamente isto
o que eu demonstrei acima, ó pá!
Se o numero de divisores for impar, tera um cara que nao tem etiqueta
repetida, justamente a raiz quadrada exata de N.É justamente isto o
que eu demonstrei acima, ó pá!

Esta é a demonstração mais formal que eu consigo escrever, sem ficar enjoado...


Em 02/09/10, enniusenn...@bol.com.br escreveu:

 Caro Torres,

 É exatamente a formalização que desejo obter.

 Um abraço!
 Ennius



-- 
/**/
Quadrinista e Taverneiro!

http://tavernadofimdomundo.blogspot.com  Quadrinhos, histórioas e afins
http://baratoeletrico.blogspot.com / Um pouco sobre elétrons em movimento
http://bridget-torres.blogspot.com/  Personal! Do not edit!

=
Instru��es para entrar na lista, sair da lista e usar a lista em
http://www.mat.puc-rio.br/~obmlistas/obm-l.html
=


[obm-l] Re: [obm-l] combinatória

2010-08-25 Por tôpico Johann Dirichlet
Este e patrecido com um problema da primeira fase da OBM de uns 2 ou 3
anos atras.
Como tem tres As repetidos, chame eles de A1, A2, A3 (A1 e uma coisa
so, nao duas. Pense como se fossem indices numericos)

Primeiro, os caras BTHL ficam nesta ordem. Veja o esquema
_B_T_L_H_

Escolha aonde o O vai ficar nestes espacos. Sao 5 possibilidades. Veja
um exemplo:
_B_T_L_H_O_

Sempre que um cara entra ele gera um espaco. Continuando, onde o A1
quer ir? Sao 6 possibilidades:
_B_A1_T_L_H_O_

E assim por diante.

Agora, lembre-se que nos diferenciamos os As. Como eles sao 3, eles
vem nestas ordens: 123,132,213,231,312,321 (ou o famoso 3!). Basta
dividir o numero obtido antes por este fator.



Em 25/08/10, Fabio Silvacacar...@yahoo.com escreveu:
 Quantos anagramas da palavra BATALHÃO (desconsidere o til como diferença)
 tem as consoantes em ordem alfabética?

 Como pensar?






-- 
/**/
Quadrinista e Taverneiro!

http://tavernadofimdomundo.blogspot.com  Quadrinhos, histórioas e afins
http://baratoeletrico.blogspot.com / Um pouco sobre elétrons em movimento
http://bridget-torres.blogspot.com/  Personal! Do not edit!

=
Instru��es para entrar na lista, sair da lista e usar a lista em
http://www.mat.puc-rio.br/~obmlistas/obm-l.html
=


  1   2   >